Download as pdf or txt
Download as pdf or txt
You are on page 1of 185

Logarithm (Sheet)

Mentor of AIR 1, 12, 18, 24 and thousands of IIT Aspirants

More than 14+ years of teaching experience (2007-Present)

Ex-Maths Faculty at top-notch IIT-JEE Coaching Institute like Career


Point, Bansal Classes, Etoos and Vibrant, Kota

Star Maths Educator on Unacademy Plus. Each course starts from Basic to
JEE Advanced.

Renowned for providing vast, clear cut and well-organized study material
with high-quality questions.

Easy Access to handwritten notes, Workbooks, sheets, DPP’s, PYQ’s and


Question Bank with Solutions.
Maths IIT-JEE ‘Best Approach’ (MCSIR) Contents

Contents
S.No. Topic Page No.
1. Logarithm Key Concept 1–2

2. Logarithm Lecture Notes 3 – 46

3. Logarithm Solved Example, 47 – 60

Exercise -I to V, Question Bank

4. Logarithm DPP (1 to 6) 61 – 68

5. Logarithm Part Test (1, 2) 69 – 72

6. Answer key 73 – 75

7. Revision Planner 76

8. Logarithm Solved Exercise - I 77 – 100

9. Logarithm Solved Exercise - II 101 – 105

10. Logarithm Solved Exercise - III 106 – 115

11. Logarithm Solved Exercise - IV 116 – 124

12. Logarithm Solved Exercise - V 125 – 128

13. Question Bank Solution 129 – 131

14. DPP Solution (1 - 6) 132 – 160

15. Part Test -1,2 Solution 161 – 172

16. Brahmastra (Final Revision) 173 – 179


Maths IIT-JEE ‘Best Approach’ (MCSIR) Logarithm

KEY CONCEPTS (LOGARITHM)


THINGS TO REMEMBER :
1. LOGARITHM OF A NUMBER :
The logarithm of the number N to the base 'a' is the
exponent indicating the power to which the base 'a' must be
raised to obtain the number N. This number is designated
as loga N.

Hence : logaN = x  ax = N , a > 0 , a  1 & N > 0

If a = 10 , then we write log b rather than log10 b .


If a = e , we write ln b rather than loge b .
The existence and uniqueness of the number loga N follows from the properties of
an exponential functions.
From the definition of the logarithm of the number N to the base 'a' , we have an
identity : a loga N = N , a > 0 , a  1 & N > 0
This is known as the FUNDAMENTAL LOGARITHMIC IDENTITY .
NOTE : loga1 = 0 (a > 0 , a  1)
loga a = 1 (a > 0 , a  1) and
log1/a a = - 1 (a > 0 , a  1)

2. THE PRINCIPAL PROPERTIES OF LOGARITHMS :


Let M & N are arbitrary posiitive numbers , a > 0 , a  1 , b > 0 , b  1 and  is any
real number then ;
(i) loga (M . N) = loga M + loga N (ii) loga (M/N) = loga M  loga N
log a M
(iii) loga M = . loga M (iv) logb M =
log a b
NOTE :  logba . logab = 1  logba = 1/logab.   logba . logcb . logac = 1
x
 logy x . logz y . loga z = logax.  eln a = ax

3.* PROPERTIES OF MONOTONOCITY OF LOGARITHM :


(i) For a > 1 the inequality 0 < x < y & loga x < loga y are equivalent.
(ii) For 0 < a < 1 the inequality 0 < x < y & loga x > loga y are equivalent.
(iii) If a > 1 then loga x < p  0 < x < ap
(iv) If a > 1 then logax > p  x > ap
(v) If 0 < a < 1 then loga x < p  x > ap
(vi) If 0 < a < 1 then logax > p  0 < x < ap

1
Maths IIT-JEE ‘Best Approach’ (MCSIR) Logarithm

NOTE THAT :
 If the number & the base are on one side of the unity , then the logarithm is positive ; If
the number & the base are on different sides of unity, then the logarithm is negative.
 The base of the logarithm ‘a’ must not equal unity otherwise numbers not equal to
unity will not have a logarithm & any number will be the logarithm of unity.
 For a non negative number 'a' & n  2 , n  N n
a = a1/n.

* Will be covered in detail in QUADRATIC EQUATION

** In this chapter log x will be treat as log10 x.

2
Logarithm

3.

Logarithm

Meaning of Logarithm:
Logarithm is a rule which is used to solve complex algebraic expression.

Exponential Form:
Every positive real number can be expressed as N = ax, a > 0, a ≠ 1, N>0
a → base, x → exponent

Q.1 Express 16 using exponential form with base 2, base 4 and base 16.
Sol. (i) N = 16,  a = 2,  Suppose x = power
1 1

2   =  16  ⇒  2   =  16
x1 4

(ii) a2 = 4,  power = x2


4x2   = 16  ⇒  42   =  16
(iii) a3 = 16, power = x3
16x3   =  16 ⇒  161   =  16

Q.2 Express 81 using exponential form with base 3, base 9, and base 81.

Sol. (i) N = 81, a1 = 3, x1 = Power


3x1 =  81 ⇒   34   =  81
(ii) a2 = 9, x2 = Power
9x2   =  81 ⇒  92 = 81
(iii) a3 = 81, x3 = Power
81x3   =  81 ⇒  811   =  81

Q.3 Express 16 using exponential form with base ‘3’.

Sol. It means, 3x = 16
We know that, 32 = 9 and 33 = 27 The types of ‘x’ can be find out
⇒ 2 < x < 3 ⇒ x ∈(2, 3) using logarithm.

Note:
Ex.
(i) What if a = 1. 1
Suppose a = –2 and x= − then
2
Express 16 in exponential form with
base a = 1 1

(−2) does not give real number.



1x = 16 ⇒ there does not exist any ‘x’. 2

For which 1x = 16
Logarithm

So we can not take base a = 1.

4.
(ii) What if a is negative.
Then for even root it will not be defined.
Hence for negative base it is not uniquely
defined that with powers it will give real
number or not.

Logarithmic form:
Logarithm of a number to some base is the
exponent by which the base must be raised
in order to get that number.

logaN = x ⇔ ax = N, N > 0, a > 0, a ≠ 1


a → base, x → exponent, N → number

Find values of following:


Q.1 (i) log101000

Sol. Suppose log101000 = x ⇔ 10x = 1000


⇒ 10x = 103 (by comparing)
⇒ x = 3

Q. (ii) log232

Sol. Suppose log232 = y ⇔ 2y = 32


⇒ 2y = 25 (by comparing)
⇒ y = 5

Q. (iii) log5625

Sol. Suppose log5625 = x ⇔ 5x = 625


⇒ 5x = 54 (by comparing)
⇒ x = 4

Q. (iv) log 2 64

Sol. ( 2)
y
Suppose log 2 64  =  y ⇔ =  64
y
⇒ 22   =  26
y
⇒ =  6  (by comparing)
2
⇒ y = 12
Logarithm

5.
Q. (v) log 2 3 1728

Sol. Suppose log 1728 = x ⇔ 2 3 ( )


x
  =  1728  =  123
2 3

( ) 
( ) 
x 2
⇒ 2 3   =    2 3 
 

( ) ( )
x 6
⇒ 2 3 = 2 3 (by comparing)

⇒x=6

Q. (vi) log1632

Sol. Suppose log1632 = y ⇔ 16y = 32


⇒ (24)y = 25
⇒ 24y = 25

⇒ 4y = 5 (by comparing)
5
⇒ y =
4

Q. (vii) log 3 7 2401

Sol. Suppose log 3 7 2401  =  x ⇔ ( 7)


3
x
= 2401
x
⇒ 7 3 =  74
x
⇒ =  4 (by comparing)
3
⇒ x = 12

Q. (viii) log 3 (log 3 (27)3 )

Sol. First we will find the value of log3(27)3 = y


⇒ log3(33)3 = y
⇒ log339 = y
⇒y=9
Then it reduces to log39 and we know that log39 = 2
Hence, log3(log3(27)3) = 2

Q.2 Find ‘x’ for which log(x – 1) (2x + 1) is defined.


 –1 
(A) x ∈ (1, ∞ ) (B) x ∈  , ∞ – {2}
2 
Logarithm

 –1 
(C) x∈ (1, ∞ ) – {2} (D) x ∈  , 1
2 

6.
Sol. (C)
We will check when log(x – 1)(2x + 1) is defined.
x – 1 > 0 and (x – 1) ≠ 1 and (2x + 1) > 0
–1
⇒ x > 1 and x ≠ 2 and x >
2
 –1 
⇒ x ∈ (1, ∞ ) and x ≠ 2 and x ∈
 2 , ∞
Intersection gives x ∈ (1, ∞ ) and x ≠ 2 ⇒ x ∈ (1, ∞ ) – {2}
It can also be written as
x ∈ (1, 2) ∪ (2, ∞)

Q.3 Find complete set of real values of ‘x’ for which log (x2–5x–6) is defined.
(2x–3)

Sol. 2x – 3 > 0 and (2x – 3) ≠ 1 and (x – 5x – 6) > 0


2

3
⇒ x> and x ≠ 2 and (x – 6) (x + 1) > 0
2
3
⇒ x> and x ≠ 2 and x ∈ ( −∞, −1) ∪ (6, ∞ )
2
3 
⇒ x ∈  , ∞  and x ≠ 2 and x ∈ ( −∞, −1) ∪ (6, ∞ )
2 
Intersection gives, x ∈ (6, ∞ )

Q.4 Let a = log1/216


 b = log3(tan30°)
(
  c =  log 2− 3 2 + 3 )
 d = log2(log24)
then a + b + c + d is
7 9 5
(A) − (B) − (C) − (D) –4
2 2 2

Sol. (B)
a = log1/216
a
1
⇒   = 16
2
⇒ 2–a = 24
⇒a=–4
b = log3(tan30°)
1
1 −
⇒ 3b = tan30° = =32
3
1
⇒ b =  −
Logarithm

7.

c =  log 2− (2 + 3 )
3

(2 + 3 )(2 − 3 )
( 3) =
c
⇒ 2−
(2 − 3 )
( 3 ) = ( 4 − 3) (2 − 3 )
c −1
⇒ 2−
c = –1
d = log2(log24)
we know that log24 = 2
so, d= log22 ⇒ d = 1
1 9
then a + b + c + d = −4 − −1+1 = −
2 2

Important Deduction:

(i)  logNN = 1
(ii)  logN1 = 0
(iii)   log1/N N = –1

(i)  Proof : Suppose logNN = x ⇔  Nx  = N 1 ⇒ x = 1


(ii)   Proof : Suppose logN1 = y ⇔  Ny   =  1  =  N0 ⇒ y = 0
z
 1
(iii) Proof : Suppose log 1 N = z ⇔     =  N ⇒  N−z   =  N1  ⇒ z = –1
N
 N

Q.5 Find values of following


(i) logsin30°cos60°
1
Sol. First simplify this, we get log 1  
Know the facts
2 2

1
so log 1   = 1 We know that logNN =1
2 2

Q. (ii) log4/3 1.3

Sol. Let N = 1.3 = 1.3333….


Know the facts
N = 1.3333… ...(i)
10 N = 13.33333....... ...(ii)
We know that logNN =1
Logarithm

8.
Subtracting (i) from (ii), we get
– 9N = –12
–12 4
N =   =   
–9 3
 4
We get log 4   = 1
3 3

Q. (iii) log 5 5 5 5 .  .  .

Sol. Sol. N  =   5


5
5 .  .  .

N

N  =   5 N
N2 = 5N
⇒ N2 –5N = 0
⇒ N(N–5) = 0
⇒ N = 0 or N = 5 but N = 0 is not possible
We get log55 = 1

Q. (iv) log2(sin2x + cos2x)

Sol. We know that sin2x + cos2 x = 1


 log 1 = 0
So we get , log21 = 0 N

Q. (v) log(tan1°).log(tan2°).log(tan3°)…log(tan89°)

Sol. log(tan1°). log(tan2°)… log(tan45°)…log(tan89°)


= log(tan1°). log(tan2°)… log1…log(tan89°)
= 0

Q. (vi) log(tan1°.tan2°.tan3°... tan89°)


Sol. We get
log(tan1°·tan2°·tan3°...tan43°·tan44°·tan45°·tan46°...tan89°)
= log(tan1°·tan2°... tan44°·tan45°·tan(99°–44°)...tan(90–1)°)
= log(tan1°·tan2°... tan44°·1·cot44°·cot43°...cot1°)
= log(tan1°·cot1°·tan2°cot2°...tan44°cot44°)
= log1 = 0
Using tan(90°–θ) = cotθ
and tanθ.cotθ = 1
Logarithm

9.
Q. (vii) log (log (log (27) ))
2 3 3
3

Sol. Let’s assume


log3(27)3 = y
⇒ log3(33)3 = y
⇒ log339 = y
⇒y=9
Then it reduces to log 2 (log 3 9) = log 2 2 = 1

2

Q. (viii)
(log 100 (
10 ) log 2 (log 4 2 ) log 4 log 22 ( 256 )) ( 2
)
log 4 8 + log 8 4


Sol. Let a = log10010 b = log2(log42)
⇒ 100a = 10 1
⇒ (10)2a = 101 ⇒ b = log 2
2
⇒ 2a = 1 ⇒ 2b = 2–1
1 ⇒b=–1
⇒a=
2
2
Let x = log 22 ( 256 ) ⇒ x = log 2 ( 28 )  ⇒ x = (log 2 216 )
2 2 2

 
⇒ x = 162 = 256
     
c = log 4 log ( 2
2 (256)
2
) d = log48
⇒ 4d = 8
e = log84
⇒ 8e = 4
So we get c = log4256 ⇒ (22)d = 23 ⇒ (23)e = 22
⇒ 4c = 44 ⇒ 22d = 23 ⇒ 3e = 2
⇒c=4 ⇒ 2d = 3 2
⇒e=
⇒d= 3 3
2

1
abc   ( −1) ( 4 ) ( −2)·6 12
2
Then given expansion reduces to =  = =−
d+e 3 2 (9 + 4) 13
+
2 3

Q.6 If log (log (log x)) = 0 = log (log (log y)) then find x + y.
2 2 3 2 3 2

Sol. log (log (log x) = 0 ⇔ log (log x) = 2 = 1  (Change it into exponential form)
2 2 3 2 3
0
Logarithm

⇔ log3x = 2 = 2 1

⇔ x = 32 = 9
x=9
10.
Similarly, log2(log3(log2y) = 0 ⇔ log3(log2y) = 20 = 1
(Change it into exponential form)
⇔ log2y = 31 = 3
⇔ y = 23 = 8
Then x + y = 9 + 8 = 17

Fundamental Identify:
alogaN = N
Proof:
Let logaN = x then it changes into
ax = N ⇔ logaN = x
So we get alogaN = N

Q. Find the value of following: 3log3 10

Sol. Using Identify, alogaN = N


We get 3log3 10 = 10

Properties of logarithm:
If m, n are positive real numbers, a > 0, a ≠ 1 then

(i) logamn = logam + logan

Proof:

Suppose logam = x and logan =y


ax = m and ay = n
then mn = ax.ay = ax + y
again change it into logarithmic form, x + y = logamn
we get logamn = logam + logan

Q.1 Solve : log102 + log105

Sol. log102 + log105


= log10(2×5) (using P(1))
= log1010
= 1
Logarithm

11.
Note:

General version:
Suppose a > 0, a ≠ 1, N1, N2, N3, …, Nr > 0
loga(N1.N2.N3 ... Nr) = logaN1 + logaN2 + logaN3 + …. + logaNr

m
(ii) log a   = logam – logan
n

Proof:

Given a > 0, a ≠ 1, m > 0, n > 0


Then suppose logam = x & logan = y
⇒ ax = m & ay = n
m ax
⇒ = = ax – y
n ay
Change it into logarithmic form,
m
x – y = log a  
n
m
⇒ logam – logan = log a  
n

Q.2 Solve : log 10 – log 5


2 2

Sol. log 10 – log 5 = log 10


2 2
5
(using P(2))
2

    = log22
    = 1

(iii) logamn = n logam

Proof:

Let’s say logam = α ⇔ aα = m


then mn = (aα)n = anα
Change it into logarithmic form, then
nα = logamn

nlogam = logamn
Logarithm

12.
Q.3 Q3. Solve log 32 2

Sol. log 32 = log 2


2 2
5

= 5log22
=5

Q.4 Find values of following :


(i) log3855 + log3857 + log38511

Sol. Using property log m + log n + log p = log mnp a a a a


We get log3855.7.11
= log385385
= 1

Q. (ii) log (log (log 625)))


4 2 5

Sol. log (log (log 5 ))


4 2 5
4

= log4(log2(4log55))
= log4(log2(22 × 1))
= log4(2log22)
= log42
= 1/2

Q. (iii) log (11)log 11


11 1331

Sol. log (11)log 1331 11


11
3
= log 11 11log11 11 [Using logamn = nlogam]
= log 11 113log11 11
= log11113
= 3log1111
= 3

Q. (iv) log 10 – log 5 + log 8


2 2 2

Sol. log 10 – log 5 + log 8


2 2 2
10
= log 2 + log 2 8 [Using P(1)]
5
= log22 + log223
= 1 + 3log22
=1+3
=4
Logarithm

13.
Q. (v) log2[log4(log10164 + log10258)]

Sol. log2[log4(log10164 + log10258)]


= log2[log4(log10164.258)]
= log2[log4(log1024×4.52×8)]
= log2[log4(log10(10)16)]
= log2[log4(16log1010)]
= log2[log442]
= log22(log44)
= log22 = 1

Q.5 1023
 1
∑ log  1 +  is equal to:
2
n
n= 1 
(A) 8 (B) 9 (C) 10       (D) 12

Sol. (C)
1023
n + 1
∑ log
n= 1
2 
 n 

2 3 4  1024 


= log 2   + log 2   + log 3   + ... + log 2  
1 2 3  1023 
 2 3 4 1024 
= log 2  . . ... 
 1 2 3 1023 
= log21024
= log2210
= 10log22 = 10

Q.6  16   25 
Find the value of log 10 2 + 16log 10   + 12log 10 
 81 
 + 7 log 10  .
 15   24   80 

Sol. log102 + 16log1016 – log1015 + 12log1025 – 12log1024 + 7log1081 – 7log1080


= log102 + 16log1024 – log10(3×5) + 12log1052 – 12log10(23.3) + 7log1034 – 7log10(24.5
= log102 + 64log102 – log103 – log35 + 24log105 – 36log102
      – 12log103 + 28log103 – 28log102 – 7log105
= (1 + 64 – 36 – 28) log102 + (–16 – 12 + 28) log103 + (–16 – 24 – 7) log105
= log102 + log105
= log1010 = 1
Logarithm

14.
Base Changing theorem:

log c a
logba = , a > 0, b > 0, c > 0, a ≠ 1, b ≠ 1, c ≠ 1
log c b

Proof:

Suppose logba = x ⇔ a = bx
 logca = logcbx
 logca = xlogcb (By taking logarithm)
log c a
  = x = logba
log c b
log c a
⇒ logb a =
log c b

Note:

1
log a b =
logb a

Proof:

log c b 1
=
log c a log c a
(By using base Changing Theorem)
log c b

log c b log c b (Both are equal to each other)


=
log c a log c a
Hence proved.

Q.7 Prove the following: logba . logcb . logdc = logda

Sol. log a . log b . log c = log a log b log c log a


. . =
b c d
log b log c log d log d
= logda (Using Base-Changing Theorem)
Logarithm

15.
Q.8 If log23. log34 . log45...logn(n + 1) = 10. Find ‘n’.

Sol. log23. log34 . log45...logn(n + 1) = 10


log 3 log 4 log 5 log(n + 1)
. . . ... . = 10
log 2 log 3 log 4 log n
log(n + 1)
⇒ = 10 (Using Base-Changing Theorem)
log 2
⇒ log2(n + 1) = 10
⇒ 210 = n + 1
⇒ n = 1024 –1 ⇒ n = 1023

Property of logarithm:

alogbc = clogba

Proof:

log a c

a logbc
=a log ab
 
1
= (a )
log a c log b
a
(Using Base-Changing theorem)
1
= (c ) log ab = clogba
Hence alogbc = clogba

Property of logarithm:
n
log ak mn = ( ) k
log a m   wherever defined

Proof:

log cmn nlog cm n (Using Base-changing theorem)


L.H.S.= = = log a m = R.H.S.
log c a k
klog c a k

Hence Proved.

Q.9 The value of 7log3 5 + 3log5 7 − 5log3 7 − 7log5 3 is equal to


(A) 3 (B) 5 (C) 7       (D) 0

Sol. (A)
7log3 5 + 7log5 3 − 7log3 5 − 7log5 3 = 0 (Using property a log cb = blog c a )
Logarithm

16.
Q.10 Let x
4

1
= log1632, x2 = log625125, x3 = log816, x4 = log 2 8 . Find ∑x .
i= 1
i

5 5
Sol. x 1 = log 24 25 =
4
log 2 2 =
4
3 3
x2 = log 54 53 = log 5 5 =
4 4
4 4
x3 = log 23 24 = log 2 2 =
3 3
x4 = log 21/2 23 = 6log 2 2 = 6
4
5 3 4 28
then ∑x
i= 1
i = x1 + x2 + x3 + x4 = + + +6 =
4 4 3 3

log 3 7 −1

Q.11 Let 1


A=  , B = 2log1/2 7 , C = 8log3 2 , D = 4−log2 6 . Find
1 1 1 1
+ − − .
9 A B C D

Sol. A = (3 ) −2
log 3 7
=3 ( )−2
log 3 7
=
1
49
log
(2−1 )
7
1 −1
B=2 = 2log2 7 =
7
1
C = 23( −log2 3) = 2log2 3 =
−3

27
−2 1
D = 2−2log2 6 = 2log2 6 =
36
1 1 1 1
+ − − = 49 + 7 – 27 – 36 = –7
A B C D

Q.12 If p log 3 7
= 81 , then find value of p(log3 7) .
2

Sol. p log 3 7.log 3 7


= (plog3 7 )
log 3 7
= ( 81)
log 3 7

4
= 34log3 7 = 3log3 7 = 74 = 2401

Q.13 If a, b, c are real positive numbers such that a


log 3 7
= 27,  b
log 7 11 log 11 25
= 49,  c = 11 ,
(log 3 7 )2 (log 7 11)2 (log 11 25)2
then find the value of a +b +c .
(A) 343 (B) 121 (C) 469      (D) 569

Sol. (C)
alog3 7.log3 7 + blog 7 11.log 7 11 + clog11 25.log11 25
= ( alog3 7 ) + (blog 7 11 ) + ( clog11 25 )
log 3 7 log 7 11 log 11 25

Logarithm

( 11 )
log 3 7 log 7 11 log 11 25
= ( 27 ) + ( 49) +

17.
= 33log3 7 + 72log 7 11 + 11½log11 25
1/2
= 3log3 7 + 7log 7 11 + 11log11 (25)
3 2

= 73 + 112 + 5
= 343 + 121 + 5 = 469

Q.14 If log x = b for permissible values of a and x then identify the statement(s)
a
which can be correct?
(A) If a and b are two irrational numbers then x can be rational.
(B) If a rational and b irrational then x can be rational.
(C) If a irrational and b rational then x can be rational.
(D) If a rational and b rational then x can be rational.

Sol. (ABCD)
Change this into exponential form
(A) x = ab
Suppose a = 2,   b = log 2 3
First we will prove that log23 is not rational
Suppose log23 is rational, then log23 = p/q ⇒ 3 = 2p/q
⇒ 3q = 2p
It is not possible for any value of p, q except p = 0 and q = 0
So, log23 is irrational.
1
Hence log 2 3 is irrational
2
So, we can say log 2 3 will be irrational

( 2)
log 3
Then 2
= 3 is rational.
(B) a = 2, b = log23
Then 2log2 3 = 3 is rational.
(C)
a = 3, b = 2

( )
2
3 = 3 is rational.
(D) a = 2, b = 3
Then (2)3 = 8 is rational.

Q.15 B=
1
+
2

3
, C=
log 3 12 log 3 4
− . Find B + C.
log 3 2 log 9 4 log 27 8 log 36 3 log 108 3
(A) 3 (B) 2 (C) 4 (D) 1

Sol. (B)
B = log23 + 2log49 – 3log827
= log 2 3 + 2log 22 32 − 3log 23 33
Logarithm

= log23 + 2log23 – 3log23 = 0

18.
log 3 12 log 3 4
C= −
log 36 3 log 108 3
C = log312 log336 – log34.log3108
C = log3(4×3).log3(4×9) – log34.log3(4×27)
C = (log34 + log33)(log34 + log39) – log34(log34 + log327)
C = (log34 + 1)(log34 + 2) – log34.(log34 + 3)
Put log34 = t,
C = (t + 1)(t + 2) – t(t + 3) = t2 + 3t + 2 – t2 – 3t = 2
So, B + C = 0 + 2 = 2

Q.16 A = 1
+
1
+
1
1 + logba + logbc 1 + log ca + log cb 1 + log ab + log a c
Where a > 0, a ≠ 1, b > 0, b ≠ 1, c > 0, c ≠ 1, abc ≠ 1, then A is :
1
(A) abc (B) (C) 1     (D) 0
abc
Sol. (C)
1 1 1
+ +
logbb + logba + logbc log cc + log ca + log cb log a a + log ab + log a c
1 1 1
= + +
logbabc log cabc log a abc
= logabca + logabcc + logabca
= logabcabc = 1

Q.17 Let a = log 5, b = log


3
25 and c = log51000, d = log72058 then which of the
17
following is/are true?
(A) a > b (B) a < b (C) c > d     (D) c < d

Sol. (AC)
1 2
a = log35 = and b = log1725 = log1752 = 2log175 =
log 5 3 log 5 17
1 1 1
Then = log 5 3 and = log 5 17 = log 5 17
a b 2
1 1
From here log 5 17 > log 5 3 ⇒ >
b a
⇒a>b
c = log51000 and d = log7(2058)
∵ 625 < 1000 < 3125
⇒ log5625 < log51000 < log53125
⇒ log554 < c < log555
⇒ 4<c<5
343 < 2058 < 2401
Logarithm

19.
⇒ log7343 < log72058 < log72401
⇒ log773 < d < log774
⇒ 3 < d < 4
Hence c > d

Q.18 Establish the trichotomy between:


(i) m = (log25)2 & n = log220

Sol. m – n = (log25)2 – (log220)


= (log25)2 – log2(5×4)
= (log25)2 – (log25 + 2log22)
= (log25)2 – log25 – 2
Let log25 = t
then, m – n = t2 – t – 2 = (t –2)(t + 1)
log25 > 2 ⇒ t > 2 hence m – n > 0 ⇒ m > n

Q. (ii) a = logπ2 + log2π and b = 1

Sol. Suppose α = logπ2


1
Then a = α + > 2 and b = 1
α
So a > b

Q. If log615 = α and log1218 = β, then find log2524 in terms of α, β.

Sol. log 3 15 log 3 ( 3 × 5 ) log 3 3 + log 3 5


= α  ⇒ =α⇒ =α
log 3 6 log 3 ( 2 × 3 ) log 3 3 + log 3 2

1 + log 3 5
⇒ = α             ... (i)
1 + log 3 2

log 3 18 log 3 ( 9 × 2 ) log 3 9 + log 3 2


Also = β⇒ = β⇒ =β
log 3 12 log 3 ( 3 × 4 ) log 3 3 + log 3 4

2 + log 3 2
⇒ = β            ... (ii)
1 + 2log 3 2

log 3 24 log 3 ( 8 × 3) 1 + 3log 3 2


Now, log 25 24 = = =
log 3 25 log 3 52 2log 3 5
By equation (ii), 2 + log32 = β + 2βlog32
⇒ (log32)(1 – 2β) = β – 2
β−2
⇒ log 3 2 =
1 − 2β
Logarithm

20.
 β−2 
By equation (i), 1 + log35 = α(1 + log32) = α 1 + 
 1 − 2β 
 1 − 2β + β − 2   −1 − β 
⇒ 1 + log35 = α   = α 
 1 − 2β   1 − 2β 
−αβ − α − 1 + 2β
⇒  log 3 5 =
1 − 2β
 β−2
1 + 3
1 + 3log 3 2  1 − 2β 
Then log 24 25 = =
2log 3 5  2β − 1 − αβ − α 
2 
 1 − 2β

⇒ log 25 = 1 − 2β + 3β − 6 β−5
=
24
(
2 2β − 1 − αβ − α ) (
2 2β − 1 − αβ − α )
Q.20 If log 12 = a and log
7
24 = b then find the value of log54168 in terms of a, b.
12

log 2 12 log 2 ( 4 × 3 ) 2 + log 2 3


Sol. Given, a = log 7 12 =
log 2 7
=
log 2 7
=
log 2 7
        … (i)

log 2 24 log 2 ( 8 × 3) 3 + log 2 3


and b = log1224 = = =         … (ii)
log 2 12 log 2 ( 4 × 3 ) 2 + log 2 3
log 2 168 log 2 ( 7 × 3 × 8) 3 + log 2 7 + log 2 3
log54168 = = =       … (iii)
log 2 54 log 2 ( 27 × 2 ) 1 + 3log 2 3
From equation (ii), 3 + log23 = 2b + b log23
⇒ (1–b)log23 = 2b – 3
2b − 3
⇒ log 2 3 =
1−b
Replace this in equation (i),
2b − 3
2+
1 − b = log 7
2
a
2 − 2b + 2b − 3
⇒ = log 2 7
a ( 1 − b)
−1
⇒ log 2 7 =
a ( 1 − b)
2b − 3 1
3+  −  
1−b a ( 1 − b)
Then from equation (iii), log 54 168 =
 2b − 3
1 + 3 
 1−b 
Logarithm

3a ( 1 − b ) + a ( 2b − 3) − 1
⇒ log 54 168 =
a ( 1 − b ) + 3 ( 2b − 3 ) 
21.
3a − 3ab + 2ab − 3a − 1
⇒ log 54 168 =
a [1 − b + 6b − 9]
−1 − ab
⇒ log 54 168 =
(
a 5b − 8 )
 
Q.21  
If log 7  log 7 7 7 7  = 1 – a log72 and log 15  log 15 15 15 15 15  = 1 – b log152.
   
Find a + b.

Sol.  
Let x = log 7  log 7 7 7 7 
 
1 
x = log 7  log 7 7 7 7 
2 


1
( 
x = log 7  log 7 7 + log 7 7 7 
2 
)
1 1 
x = log 7   1 + log 7 7 7  
2 2 
1 1 1 
x = log 7   1 +  log 7 7 + log 7 7   
2 2 2 
1 1 1 
x = log 7   1 +  1 +   
2 2 2 
1 1 3 
x = log 7   1 + .  
2 2 2 
 1  7  7
x = log 7     = log 7
2
  4 8
x = 1 – log78
x= 1 – 3log72
Compare it with x = 1 – a log72 then a = 3
 
Similarly, suppose y = log 15  log 15 15 15 15 15 
 
1 
then y = log 15  log 15 15 15 15 15 
 2 
1 1 
y = log 15   1 + log 15 15 15 15  
2 2 
1  1 1  
y = log 15   1 +  1 + log 15 15 15   
Logarithm

2  2 2  

22.
1  1 1 1   
y = log 15   1 +  1 +  1 +    
2  2 2 2   
1  1 1 3  
y = log 15   1 +  1 + .   
2  2 2 2  
1  7   1 15 
y = log 15   1 +   = log 15  . 
2  8  2 8 
y = log1515 – log152 4

y = 1 –4 log152, compare it with y = 1 – b log152


Hence b = 4 and a = 3
So a+b=4+3
a+b=7

Q.22 If
log a log b log c
b−c
=
c−a
=
a −b
, show that aa.bb.cc = 1.

log a log b log c


Sol. Assume,
b−c
=
c−a
=
a −b
=k

then log a = k(b –c) ⇒ a log a = k (ab – ac) … (i)


log b = k(c –a) ⇒ b log b = k (bc – ab) … (ii)
log c = k(a –b) ⇒ c log c = k (ac – bc) … (iii)
Adding (i), (ii) and (iii), we get
⇒ log(aa.bb.cc) = k[ab – ac + bc – ab + ac – bc]
⇒ log(aa.bb.cc) = 0 (change it into exponential form)
⇒ aa.bb.cc = 1

Q.23 If a, b, c are positive real numbers other than unity such that
a (b + c − a ) b ( c + a − b ) c ( a + b − c )
= = , prove that abba = bccb = caac.
log a log b log c

Sol. Assume,
a (b + c − a ) b (c + a − b) c (a + b − c )
= = =k
log a log b log c
a (b + c − a )
⇒ log a =   …(i)
k
b (c + a − b)
log b =   …(ii) 
k
c (a + b − c )
log c =   …(iii)
k
ab (b + c − a ) ab ( c + a − b ) c (a + b − c )
⇒ b log a = ,   a log b = ,   log c =
k k k
Logarithm

23.
ab (b + c − a )
⇒ log ab =   …(iv) 
k
ab ( c + a − b )
log ba = …(v)
k
Add equations (iv) & (v)
ab 2abc
log (ab.ba) = [b + c – a + c + a –b] =
k k
Consider the base as ‘e’,
2abc
we get, abba = e k … (A)
Now multiply (ii) by c and (iii) by b, we get
bc ( c + a − b )
c log b = … (vi)
k
bc ( a + b − c )
and b log c = …(vii)
k
Add (vi) and (vii), we get
bc 2abc
log (bc .cb ) = [c +a – b + a + b – c] =
k k
Change this into exponential form, we get
2abc
bc .cb = e k
… (B)
2abc
Similarly we get, ca .ac = e … (C)
k

From equations (A), (B) and (C), we get


ab.ba = bc.cb = ca.ac

Logarithmic Equation

Q.1 Find ‘x’ in following :


(i) x2 + 7log 7 x − 2 = 0

Sol. Using a = N , we get


log aN

x + x – 2 = 0 and x > 0
2

⇒ x2 + 2x – x – 2 = 0
⇒ x(x + 2) – 1(x + 2) = 0
⇒ (x + 2) (x – 1) = 0
⇒ either x = 1 or x = –2
Since x > 0, so we get x = 1
Logarithm

24.
( )
Q.
log 2 x2
(ii) 2 − 3x − 4 = 0

Sol. Using property alogaN = N


We get x2 –3x – 4 = 0
⇒ x2– 4x + x – 4 = 0
⇒ x(x –4) + 1(x –4) = 0
⇒ (x –4)(x + 1) = 0
⇒ x = 4 and x = –1

log 2 ( 9 − 2x )
Q. (iii)
3−x
=1

Sol. log2(9–2x) = 3 – x (change it into exponential form)


⇒ 9 – 2x = 23–x
8
⇒ 9 – 2x = 23.2–x =
2x
⇒ Put 2x = t, we get
8
9–t=
t
⇒ 9t – t = 8
2

⇒ t2 – 9t + 8 = 0
⇒ t2 – 8t –t + 8 = 0
⇒ t(t –8) –1(t –8) = 0
⇒ t = 8, t = 1
⇒ 2x = 23 and 2x = 1
⇒ x = 3 and x = 0, but x ≠ 3, hence x = 0

Q. (iv) ( x + 1)
log 10 ( x + 1)
= 100 ( x + 1)

Sol. By taking logarithm both sides with base 10, we get


log 10 ( x + 1)
log 10 ( x + 1) = log 10 100 ( x + 1)
⇒ log10(x + 1).log10(x + 1) = log10100 + log10(x + 1)
Consider, log10(x + 1) = t, we get
t2 = 2 + t
⇒ t2 – t – 2 = 0
⇒ t2 – 2t + t – 2 = 0
⇒ t(t –2) + 1(t –2) = 0
⇒ (t –2)(t + 1) = 0
⇒ t = 2, t = –1
log10(x + 1) = 2 and log10(x +1) = –1
1
⇒ x + 1 = 100 and (x + 1) =
Logarithm


10

25.
1
⇒ x = 100 – 1 = 99 and x = −1
10
−9
⇒ x = 99 and x =
10
9
⇒ x = 99 and x = −
10

Q. (v) logx–14 = 1 + log2(x –1)

Sol. logx–122 = 1 + log2(x–1)


2
⇒ = 1 + log 2 ( x − 1)
log 2 ( x − 1)
2
Consider log2(x –1) = t, we get =1+t
t
⇒ 2 = t + t2
⇒ t2 + t – 2 = 0
⇒ t2 +2t – t –2 = 0
⇒ t(t + 2) –1(t + 2) = 0
⇒ (t + 2)(t –1) = 0
⇒ t = –2, t = 1
⇒ log2(x –1) = –2 and log2(x –1) = 1
⇒ (x–1) = 2–2 and (x –1) = 21
1
⇒ x=1+ and x = 1 + 2 = 3
4
5
⇒ x = and x = 3
4

1 1 1
Q. (vi) 1 – log 5 = log  + log x + log 5
3 2 3

Sol. Base is given as 10


1 1 1 
log 10 – log 5 = log  + log x + log 5
3 2 3 
10 1
⇒ log  =  −log 2 + log x + log 51/3 
5 3
⇒ 3 log 2 + log 2 = log x + log 51/3
⇒ 4 log 2 – log 51/3 = log x
16
⇒ log x = log 1/3
5
16
⇒ x = 1/3
5
Logarithm

26.
Q. (vii) 3log3x + x(log3x ) = 162
2

Sol. ( 3 )log 3 x log 3 x


+ xlog3x − 162 = 0

⇒ xlog3x + xlog3x − 162 = 0


⇒ 2xlog3x = 162
⇒ xlog3x = 81 
Take logarithm on both sides with base 3, we get
log3x.log3x = log381 = 4
⇒ (log3x)2 = 4
⇒ log3x = ±2
⇒ x = 3±2
⇒ x = 9, 1
9

Q. (viii) 51+log 4x + 5(log1/ 4x)−1 =


26
5

Sol. 51+log 4x + 5−1−log 4x =


26
5
Put 51+log 4x = t , we get
1 26
t+ =
t 5
26t
⇒ t2 + 1 =
5
⇒ 5t + 5 = 26t
2

⇒ 5t2 – 26t + 5 = 0
⇒ 5t2 – 25t – t + 5 = 0
⇒ 5t(t –5) – 1 (t –5) = 0
⇒ (t –5)(5t –1) = 0
⇒ t = 5 or t = 1/5
1+log 4 x 1 1+log 4 x
⇒ 5 = 51  or  5
= 5−1 =
5
⇒ 1 + log4x = 1 or 1 + log4x = –1
⇒ log4x = 0 or log4x = –2
⇒ x = 1 or x = 4–2
1
⇒ x = 1 or x =
16
Logarithm

27.
Q. (ix) log4(2log3(1 + log2(1 + 3log2x))) =
1
2

Sol. Change it into exponential form,


2log (1 + log (1 + 3log x)) = 4 = 2 1/2
3 2 2
Again, 1 + log2(1 + 3log2x) = 3
log2(1 + 3log2x) = 2
Again, 1 + 3log2x = 4
⇒ 3log2x = 3
⇒ log2x = 1
⇒x=2

Q. (x) log 5 ( 51/x + 125 ) = log 5 6 + 1 +


1
2x

Sol. log (5
5
1/x 
+ 125) = log56 +  1 +
1 
 log55
2x 

 1 
 1+ 
⇒ log5(5 1/x
+ 125) = log56 + log 5 5  2x 

⇒ log5(5 + 125) = log5(6×5×51/2x)


1/x

⇒ 51/x + 125 = 30×51/2x


Put 51/2x = t, then t2 + 125 = 30t
⇒ t2 – 30t + 125 = 0
⇒ t2 – 25t – 5t + 125 = 0
⇒ t(t –25) – 5(t –25) = 0
⇒ (t–25)(t –5) = 0
Either t = 25 or t = 5
⇒ 51/2x = 52 or 51/2x = 51
1 1
⇒ = 2 or =1
2x 2x
1 1
⇒ x = or x =
4 2

Q. (xi) x2+log x 4 = x2 + 27

Sol. x2 .xlog x 4 = x2 + 27
⇒ x2.4 = x2 + 27
⇒ 3x2 = 27
⇒ x2 = 9
⇒ x = +3 or x = –3
As x > 0 and x ≠ 1, so we get x = 3
Logarithm

28.
Q. (xii) a2log2x = 5 + 4xlog2a

Sol. (a )
log 2 x 2
= 5 + 4 ( alog2x ) (Using property alogcb = blogca )

Put alog2x = t , we get


t2 = 5 + 4t
⇒ t2 – 4t – 5 = 0
⇒ t2 – 5t + t – 5 = 0
⇒ t(t –5) + 1(t–5) = 0
⇒ (t–5)(t + 1) = 0
⇒ t = 5 or t = –1
⇒ alog2x = 5  or alog2x = −1  (not possible)
⇒ log5a.log2x = log55
⇒ log2x.log5a = 1
1
⇒ log 2 x =
log 5a
⇒ log2x = loga5
⇒ x = 2log 5 or 5log
a a 2

Common logarithm and Natural logarithm:


log10N is referred as a common logarithm.
logeN is called as a natural logarithm. It is also
written as lnN.

Note:
e is an irrational quantity lying between 2.7 to
2.8. Also elnx = x Ex. (i) 3.4 = 3 + 0.4
R = 3.4, I = 3 and f = 0.4
Characteristics and Mantissa (ii) 5.1 = 5 + 0.1
R = 5.1, I = 5 and f = 0.1
Real number = Integer + Fraction (iii) –1.2 = –1 –0.2
logaN = Characteristics + Mantissa –1.2 = –1 – 1 + 1 –0.2
R=I+f –1.2 = –2 + 0.8
where R is a real number R = –1.2, I = –2, f = 0.8
I is an integer
(iv) –3.8 = –3 – 0.8
f is a fraction i.e. 0 ≤ f < 1
–3.8 = –3 –1 + 1 –0.8
–3.8 = –4 + 0.2
Here, characteristics is an integer and R = 3.8, I = –4, f = 0.2
mantissa is fraction, so (v) –6.2 = –7 + 0.8
0 ≤ mantissa < 1 R = –6.2, I = –7, f = 0.8
Logarithm

29.
Note:

(i) Characteristic can be zero, positive and negative.


(ii)   Mantissa is always non-negative and less than 1.

Finding Number of digits

Number of digits = Characteristics + 1

Observation:

Range Taking log Characteristics Number of digits

1 ≤ N < 10 0 ≤ log10N < 1 0 1

10 ≤ N < 100 1 ≤ log10N < 2 1 2

100 ≤ N < 1000 2 ≤ log10N < 3 2 3

By observation, we can say that


Number of digits = Characteristics + 1

Remember the following values

log102 = 0.3010
log103 = 0.4771
log105 = 0.6989
log107 = 0.8450

Q.1 Find number of digits in


(i) N = 650

Sol. Taking logarithm on both sides, we get


log10N = log10650 = 50 log106
Logarithm

30.
   = 50[log102 + log103]
   = 50[0.3010 + 0.4771]
   = 50[0.7781] = 38.905
log10650 = 38 + 0.905
Characteristics = 38, So Number of digits = 39

Q. (ii) N = 525

Taking logarithm on both sides, we get


Sol. log N = log10525 = 25log10
10
= 25[log1010 – log102]
10
2
log10N = 25[1–0.3010]
= 25[0.699]
log10525 = 17.475 = 17 + 0.475
Here characteristics = 17
Hence, Number of digits = 18

Number of zeroes after decimal before a significant digit starts:

Number of zeroes = |Characteristics + 1|

Observation:

Range Taking log Characteristics Number of zeroes

0.1 ≤ N < 1 –1 ≤ log10N < 0 –1 0

0.01 ≤ N < 0.1 –2 ≤ log10N < –1 –2 1

0.001 ≤ N < 0.01 –3 ≤ log10N < –2 –3 2

Hence by observation we can say that


Number of zeros = |Characteristics + 1|
Logarithm

31.
Q.2 Find number of zeroes after decimal point before a significant digit starts in
(i) N = 3–50

Sol. Taking logarithm on both sides, we get


log10N = log103–50 = (–50)log103
⇒ log10N = (–50) × (0.4771) = –23.855
⇒ log10N = –23 – 0.855 + 1 – 1 = –24 + 0.145
Here characteristics = –24
Hence Number of zeros = |–24 +1| = |–23| = 23

−100
9
Q. (ii) N= 
8

Sol. Taking logarithm on both sides with base ‘10’


−100
9 9
log10N = log 10   = −100log 10    = –100[2log3 – 3log2]
8 8
⇒ log10N = –100[2 × 0.4771 – 3 × 0.3010]
⇒ log10N = –100[0.9542 – 0.9030]
⇒ log10N = –5.12
Characteristics = –6
Number of zeros = |–6 + 1| = |–5| = 5

Q.3 Find ‘N’ for which characteristics in 2 and base is 10


(A) N ∈ [1000, 10000) (B) N ∈ [10, 100)
(C) N ∈ [100, 1000) (D) N ∈ [1, 10)

Sol. (C)
According to given condition,
⇒ 2 ≤ log10N < 3
⇒ 100 ≤ N < 1000
⇒ N ∈ [100, 1000)

Q.4 Find the number of integral values of N, for which characteristic is 5 under
base 3.

Sol. 5
According to given condition,
≤ log N < 6
3
⇒ 35 ≤ N < 36
⇒ 243 ≤ N < 729
Number of integral values of N = 729 – 243 = 486

Q.5 Find ‘N’ for which characteristics is –5 under base 3.

Sol. According to given statement,


Logarithm

–5 ≤ log N < –4
3
⇒ 3–5 ≤ N < 3–4

32.
1 1
⇒ ≤N< 4
3 5
3
1 1
⇒ ≤N<
243 81
 1 1 
⇒ N ∈  , 
 243 81 

Significance of Modulus function in Logarithmic Equation:

Q.1 Solve : log4x2 =2

Sol. xx =42
2

= 16
2

x = ±4
Wrong method:
log4x2 = 2
⇒ 2log4x = 2
⇒ log4x = 1
⇒ x = 41
⇒x=4
Incomplete solution
That’s why
log(x2) = 2log |x|
Now it can be solved as
log4x2 = 2 ⇒ 2log4|x| = 2
⇒ log4|x| = 1
⇒ |x| = 41
⇒ x=±4

Note:

(i) logx2 = 2 log|x|
(ii)   logx2m = 2m log|x|, m ∈ N
(iii)  logx2m+1 = (2m +1) logx, m ∈ N
(iv) x2 = x
y
Absolute value function/modulus function
y = –x y=x
 x ; x  0

Y x  x  0 ; x 0
2
x
x ; x0 (0, 0)

Logarithm

33.
Q.2 Solve : |x–5| = 10. Find ‘x’

Sol. x – 5 = ± 10
⇒ x – 5 = 10 or x – 5 = –10
⇒ x = 15 or x = –5

Q.3 Find ‘x’ if |3x – 2| + x = 11



Sol. Case-(i) When x<
2
3
–∞
– +

Then, –(3x –2) + x = 11 2
⇒ –3x + 2 + x = 11 3
⇒ –2x = 9
⇒ x = –9/2
2
Case-(ii) When x ≥
3
(3x –2) + x = 11
⇒ 4x = 13
⇒ x = 13/4
Hence, x = –9/2 and x = 13/4 are the solution.

Q.4 Find ‘x’ if |x – 3| +2|x + 1| = 4

Sol. –∞
–1 3

Case-(i) When x <–1, then


–(x –3) –2(x + 1) = 4
⇒ –x + 3 – 2x – 2 = 4
⇒ –3x = 3
⇒ x = –1, but x < –1 hence no solution.
Case-(ii) When –1 ≤ x < 3, then
–(x –3) + 2(x + 1) = 4
⇒ –x + 3 + 2x + 2 = 4
⇒ x = –1 is a solution.
Case-(iii) When x ≥ 3, then
(x –3) +2(x +1) = 4
⇒ x –3 + 2x + 2 = 4
⇒ 3x = 5
5
⇒ x= but x ≥ 3
3
Hence no solution
So, x = –1 is the only solution.
Logarithm

34.
Q.5 Find ‘x’
4
(i) 2log8(2x) + log8(x2 + 1 – 2x) =
3
Sol. 2log 8  2x   log 8  x  1 
2 4
3
4
 2log 8  2x   2log
x1  (Change it into exponential form)
8
3
2
 log 8 2x x  1 
3
2
 2x x  1   8 3  4
x x1 2

Case (i): When x < 1, then


–x (x – 1) = 2
⇒ x2 – x + 2 = 0
It’s discriminant D = (–1)2 – 4 × 2 = 1 – 8 = –7 < 0
Hence, no real roots exist, so no solution.
Case (ii): When x ≥ 1, then
x (x –1) = 2
⇒ x2 – x = 2
⇒ x2 – x – 2 = 0
⇒ (x –2)(x + 1) = 0
⇒ x = – 1 or x = 2, but x ≥ 1
So, x = 2 is the only solution.

Q. (ii) 2log 3 ( x − 2 ) + log 3 ( x − 4 ) = 0


2

Sol. 2log 3 ( x − 2 ) + 2log 3 x − 4 = 0

⇒ log 3 ( x − 2 ) x − 4 = 0 (Change it into exponential form)


⇒ (x – 2) |x – 4| = 1
Case (i): When x < 4, then –∞ ∞

− ( x − 2 )( x − 4 ) = 1
4

⇒ x2 − 6x + 8 = −1
⇒ x2 − 6x + 9 = 0
⇒ (x – 3)2 = 0 ⇒ x = 3
Logarithm

35.
Case (ii): When x ≥ 4

( x − 2)( x − 4 ) = 1
⇒ x2 − 6x + 8 = 1
⇒ x2 − 6x + 7 = 0

6± 8
⇒x=
2

⇒ x = 3 + 2,   x = 3 − 2
but x ≥ 4 So, x = 3 + 2
Hence, x = 3 or x = 3 + 2 is the solution.

Q. (
(iii) log 4 x2 − 1 − log 4 x − 1 ) ( )
2
= log 4 (4 − x)
2

Sol. log ( x 2
)
− 1 − log 4 ( x − 1) = log 4 4 − x
2
4

⇒ log 4
(x 2
−1 ) = log 4 − x = log 4 x − 4
( x − 1)
2 4


(x 2
−1 ) = x−4
( x − 1)
2


( x − 1)( x + 1) = x − 4
( x − 1)
2


( x + 1) = x − 4
( x − 1) –∞ ∞
4
Case (i): When x < 4
x+1
= − ( x − 4)
x−1
⇒ −x − 1 = ( x − 1)( x − 4 )

⇒ –x – 1 = x2 – 5x + 4
⇒ x2 – 4x + 5 = 0, Since D < 0
Hence no real roots.

Case(ii): When x ≥ 4
x+1
 
x−1
= x−4 ( )

Logarithm

36.

⇒ x+1 = x−4 x−1 ( ) ( )( )

(
⇒ x + 1 = x2 − 5x + 4 )
⇒ x2 − 6x + 3 = 0

⇒ x =3± 6

⇒ x = 3 + 6  , 3 − 6   
Since x ≥ 4 so x = 3 + 6

Q. (iv) x − 2
10x2 − 1
=x−2
3x

Sol. Comparing the powers,


10x2 − 1 = 3x
⇒ 10x2 − 3x − 1 = 0

⇒ 10x2 − 5x + 2x − 1 = 0
 5x ( 2x − 1) + ( 2x − 1) = 0

⇒ ( 2x − 1)( 5x + 1) = 0

1 or 1
⇒x= x=−
2 5
When base is equal to 1, we get
|x –2| = 1
⇒x–2=±1
⇒ x = 3 and x = 1

Q. (v) | x − 3 |3x
2
− 10x + 3
=1

Sol. | x − 3 | 3x2 − 10x + 3 0


= x − 3 (Comparing the powers)
⇒ 3x – 10x + 3 = 0
2

⇒ 3x2 – 9x – x + 3 = 0
⇒ 3x(x –3) –1(x –3) = 0
⇒ (x – 3)(3x – 1) = 0
1
⇒x= ,3
3
When base is equal to 1,
|x – 3| = 1
⇒x–3=±1
⇒ x = 4, 2
When x = 3, base is 0, so we reject this.
Logarithm

1
So x = 4, 2, is the solution.
3
37.
Graphs of logarithm:

( )
f x = log a x,   x > 0,   a ≠ 1,   a > 0
Case-1:
a>1 y
()
Ex.: f x = log 2 x = y

x=
1 , y = –1 y = logax, when a > 0, a  0
2
1 x
x= , y = –2 (1, 0)
4
x = 1, y = 0
x = 2, y = 1
x = 4, y = 2
x = 8, y = 3
x = 16, y = 4
y
Case-2: y =f(x) = logax
0<a<1 where 0 < a < 1
Ex.: f (x ) = log 1 x = y
2
(1, 0)
x
x = 2, y = – 1
x = 4, y = – 2
x = 8, y = – 3
x = 16, y = – 4

Note:

1. W
 henever the number and base are on 2. W
 henever the number and base are on
the same side of unity then logarithm of the opposite side of unity then logarithm
that number to the same base is positive. of that number to the base is negative.

( )
(i)  y = f x = log a x > 0 ⇒ x,  a are on the same side of unity.

(ii)  y = f ( x ) = log a
x < 0 ⇒ x,  a are on the opposite side of unity.
Logarithm

38.
  17
Q.1 ( )
Solve log 4 log 3 x + log 1  log 1 y  = 0 and x + y =
2 2
.
  4
4  3 

 
Sol. log 4 (log 3 x ) + log 1  log 1 y  = 0
  .... (i)
4  3 

17
x2 + y 2 = ....(ii)
4
 
log 4 (log 3 x ) − log 4  log 1 y  = 0
 
 3 

 
⇒ log 4 (log 3 x ) = log 4  log 1 y 
 
 3 

log 3 x = log 1 y 
3

 log 3 x = − log 3 y
1
log 3 x = log 3
y
1
 x =
y
1 17
Equation (ii), x2 + = , put x2 = t
x 2
4
1 17
t+ =
t 4
t 2 + 1 17
⇒ =
t 4
⇒ 4t 2 + 4 = 17t
⇒ 4t 2 − 17t + 4 = 0
1
⇒ t = 4,
4
1
⇒ x2 = 4,   
4
1 1
⇒ x = ±2,   ± but x > 0 ⇒ x = 2  and y =  
2 2
Logarithm

39.
Graphs of Exponential
f ( x ) = ax   a > 0, a≠1

Case-1: a > 1
y

f(x) = ax
a>1
(0, 1)
x

Ex.: ( )
y =  f x = 2
x

x = 1, y=2
x = 2, y=4 Increasing
x = 3, y = 8

Case-2: 0<a<1
y
y = ax = f(x)
0<a<1

(0, 1)

x
 1
Ex.:  f ( x ) =  
2
1
x = 1, y=
2
1
x = 2, y=
4
1
x = 3, y= Decreasing
8

()
x → ∞  ⇒ f x → 0

x → −∞  ⇒ f (x ) → ∞

Logarithm

40.
Logarithmic Inequalities
1.
f ( x ) = log a x
(i)  When a > 1 and x1 > x2 ⇔ log a x 1 > log a x2
(ii)   When 0 < a < 1 and x1 > x2 ⇔ log a x 1 < log a x2

2.
f ( x ) = ax
x1 x2
(i)  When a > 1 and x1 > x2 ⇔ a >a
x1 x2
(ii)  When 0 < a < 1 and x1 > x2 ⇔ a <a

Find ‘x’
Q.1 (
(i) log 1 2x + 1 > 0 )
2

Sol. log 1 ( 2x + 1) > log 1 1


2 2

⇒ ( 2x + 1) < 1
⇒x<0

and
( 2x + 1) > 0
1
⇒x>−
2
 1 
By Intersection, x ∈  − , 0 
 2 

Q. (ii) log 2 ( x + 2 ) > 0

Sol. log 2 ( x + 2 ) > log 2 1


⇒ (x + 2) > 1
⇒x>–1
and x + 2 > 0
⇒x>–2
By intersection, we get x > – 1 or x ∈ (–1, ∞).
Logarithm

41.
Q. (iii) 23−6x > 1

Sol. 23−6x > 20


⇒ 3 – 6x > 0
⇒ 6x < 3
⇒x<
3
6
1
⇒x<
2
 1
 x ∈   –∞, 
 2

2x + 1
 1
Q. (iv)   > 125
5

Sol. 5
− ( 2x + 1)
> 53
⇒ −2x − 1 > 3
⇒ −2x > 4
⇒ x < −2
⇒ x ∈ ( −∞, −2 )

Q. (v) log 0.3 x − 2 > 0

Sol. log 0.3 x − 2 > log 0.3 1


⇒ |x – 2| < 1
⇒–1<x–2<1
⇒1<x<3
and |x –2| = 1 ⇒ x – 2 = ± 1 ⇒ x = 3 or 1
and |x – 2| ≠ 0 ⇒ x ≠ 2
( ) ( ) ( ) {}
By Intersection, x ∈ 1, 2 ∪ 2, 3 or x ∈ 1, 3  – 2

Q. (
(vi) log 8 x2 − 4x + 3 ≤ 1 )
Sol. (
log 8 x2 − 4x + 3 ≤ log 8 8)
⇒ x2 − 4x + 3 ≤ 8 and x2 − 4x + 3 > 0

⇒ x2 − 4x − 5 ≤ 0 and x − 4x + 3 > 0
2
Logarithm

⇒ x2 − 5x + x − 5 ≤ 0 and x2 − 3x − x + 3 > 0

42.
⇒ x (x-5) + 1 (x-5) ≤ 0 and x ( x − 3) − 1 ( x − 3) > 0
⇒ ( x − 5 )( x + 1) ≤ 0 and ( x − 3 )( x − 1) > 0

( ) ( )
⇒ x ∈  −1,  5 and x ∈ −∞, 1 ∪ 3, ∞
By intersection, x ∈  −1, 1) ∪ ( 3, 5

  x2 + x  
Q. (vii) log 
1 
log 6    < 0
2   x + 4 
0
 x2 + x   1 
Sol. (i) log 6  > 
 x + 4  2
x2 + x
>6
x+4
 x2 + x 
(ii) log 6 >0
 x+4 
x2 + x
>1
x+4
x2 + x
(iii) >0
x+4
By intersection of (i), (ii) and (iii) we can clearly state that
x2 + x
>6
x+4
x2 + x
⇒ −6 >0
x+4
x2 + x − 6x − 24
⇒ >0
x+4
x2 − 5x − 24
⇒ >0
( x + 4)

( x − 8)( x + 3) > 0
( x + 4)
⇒ x ∈ ( −4, −3 ) ∪ ( 8, ∞ )
Logarithm

43.
Q. ( )
(viii) 0.3 3
  3x +6  
log 1  log 2 
2  
 x +2   >1

Sol. (0.3)   3x +6  
log 1  log 2  0
  3x + 6 
> ( 0.3 ) and log 2  2
3x + 6
3   x2 + 2  
 > 0 and x2 + 2 > 0
x +2
  3x + 6   3x + 6 3x + 6
So, log 1  log 2  2   < 0 and 2 > 1 and 2 >0
3   x + 2   x + 2 x + 2

 3x + 6 
⇒ log 2  2 >1
x +2
3x + 6
⇒ >2
x2 + 2
By intersection, we get
3x + 6
>2
x2 + 2
⇒ 3x + 6 > 2(x2 + 2)
⇒ 2x2 + 4 < 3x + 6
⇒ 2x2 – 3x – 2 < 0
⇒ 2x2 – 4x + x – 2 < 0
⇒ 2x(x –2) + 1(x – 2) < 0
⇒ (x –2)(2x + 1) < 0
 1 
⇒ x ∈  − , 2
 2 

3
  1 x    1 x 
Q.2 Find the solution of the equation, 2log 9  2   − 1  = log 27    − 4  . Also
 2   4  
   
state whether the solution is rational or irrational.
x x
 1  1
Sol. Suppose   = t so   = t 2
2 4
2 3
2
log 3 ( 2t − 1) = log 3 t 2 − 4
3
( )

⇒ 2t – 1 = t2 – 4
⇒ t2 – 2t – 3=0
⇒ (t – 3) (t + 1) = 0
⇒ t = 3, t = – 1
x x
 1  1
⇒   = 3 or   =  – 1 (Not possible)
Logarithm


2 2

44.
⇒ 2− x = 3
⇒ – x log2 = log3
log 3
⇒x= −
log 2
( )
⇒ x  =  − log 2 3   which is irrational.

Q.3 ( 2020) x =
( 2020)
log x 2021
If the product of all solutions of equation can be
2021
m
expressed in the lowest form as . (m,  n ∈ I) then find the value of (m + n).
n
 2020x 
Sol. log   = log x ( 2021)·log ( 2021) ( 2020 )
 2021 
2021

⇒ log 2021 2020 + log 2021x − 1 = log x 2021·log 2021 2020

Suppose log 2021x t then log 2021 = 1


x
t
1
Then, log 2021 2020 + t − 1 =  log 2021 2020
t
t ·log 2021 2020 + t 2 − t − log 2021 2020 = 0
⇒ (t – 1) log2021(2020) + t(t – 1) = 0
⇒ (t – 1) (log20212020 + t) = 0
⇒ t = 1 or t = –log20212020
⇒ log2021x = 1 or log2021x = log2021(2020)–1
1
⇒ x = 2021 or x=
2020
Product of solutions
m 2021
=
n 2020
m + n = 2021 + 2020 = 4041

Q.4 The sum of the integral values(s) of a ∈ [–9, 9] so that equation


(x–2)log133 + log13(3x – 7a) = log132 + 2log13 a has integral solution.

Sol. log 13 ( ) (
3x −2 · 3x − 7a = log 13 2·a 2 )
(
⇒ 3x −2 3x − 7a = 2a 2 )
Put 3 = t, we get
x

t
(
t − 7a = 2a 2 )
Logarithm

9
⇒ t 2 − 7at = 18a 2
45.
⇒ t 2 − 9at + 2at − 18a 2 = 0
⇒ (t – 9a) (t + 2a) = 0
⇒ t = 9a or t = – 2a
⇒ 3x = 9a or 3x = – 2a
30, 31, 32, 33, ... = – 2a or
1, 3, 9, 27, ... = –2a or

30, 31, 32, ... = 9a


1, 3, 9, ... = 9a
1 3 9
a = ,    ,    , …  
No integral value of a 9 9 9
1 1
a= ,    , 1,   3,   9, …
9 3

a = 1, 3, 9
Sum = 1 + 3 + 9 = 13

 or i = 1 to 6, let loga(logb(logcxi))=0, where a, b and c represent every possible


F
Q.5 different arrangement of 2, 4 and 8. The product x1x2x3x4x5x6 can be expressed
in the form 2N. The value of N, is
(A) 20 (B) 28 (C) 33       (D) 50

Sol. (D)
loga(logb(logcxi)) = 0
⇒ logb(logcxi) = a = 1
0

⇒ logcxi = b
⇒ xi = cb
a = 2 a = 4 a=8
x = 48, 84 x = 28, 82 x = 24, 42
x1x2x3x4x5x6 = 82 · 48 · 84 · 28 · 42 · 24 = 216 · 212 · 28 · 26 · 24 · 24 = 250
N = 50
Logarithm

46.
Maths IIT-JEE ‘Best Approach’ (MCSIR) Logarithm
SOLVED EXAMPLES
1
1. If log 8 b  3 , then find the value of b.
3

1 2 10
Sol. log 8 b  3  log 2 b   log 2 b  5  b  25  32
3 3 3

1 1 1 1
2. If n > 1, then prove that =   .....  
log 2 n log3 n log53 n log53! n

Sol. The given expression is equal to logn2 + logn3 + ..... + logn53 = logn(2.3 ...... 53)

1
= logn 53! =
log53! n

3. Prove that number log2 7 is an irrational number


Sol. Let log27 is a rational number
p
 log27 = q  7 = 2p/q  7q = 2p which is not possible for any integral values of p and q.

Hence, log2 7 is irrational.

1 1 1
4. Simplify  
1  log a bc 1  log b ca 1  log c ab

1 1 1
Sol.  
1  log a bc 1  log b ca 1  log c ab

log a log b log c


   1
log a  log b  log c log a  log b  log c log a  log b  log c

5. Solve log2 (3x – 2) = log1/2 x.


log 2 x
Sol. log2(3x – 2) = log1/2 x =  log 2 x 1
log 2 21
 3x – 2 = x–1  3x2 – 2x = 1  x = 1 or x = –1/3. But log2 (3x – 2) and log1/2 x are meaningful
only if x > 2/3.
Hence, x = 1.
  1 2  1 
6. If log 25 = a and log 225 = b, then find the value of log      log   in terms of a and b (base
 9    2250 
 
of the log is 10).
Sol. log 25 = a ; log 225 = b
2 log 5 = a ; log (25 . 9) = b or log 25 + 2 log 3 = 3  2 log 3 = b – a
2
Now 1  1 
log    log  
9  2250 

Get 10% Instant Discount On Unacademy Plus [Use Referral Code: MCSIR] 47
Maths IIT-JEE ‘Best Approach’ (MCSIR) Logarithm

= –2 log 9 – log 2250


= –4 log 3 – [log 225 + log 10]
= –2 (b – a) – [b + 1]
= –2b + 2a – b – 1 = 2a – 3b – 1

1 1
7. Compute the value of  .
log 2 36 log3 36

1 1 1
Sol.   log36 2  log 36 3  log36 6  .
log 2 36 log3 36 2

8. If logx–3 (2x – 3) is a meaningful quantity then find the interval in which x must lie.
Sol. x – 3 > 0, x – 3  1 and 2x – 3 > 0
x > 3 ; x  4 and x > 3/2 Hence x  (3, 4)  (4,  )
9. Compute log6 16 if log12 27 = a.
Sol. The chain of transformations
4 4
log6 16 = 4 log6 2 = 
log 2 6 1  log 2 3
Shows us that we have to know log2 3 in order to find log6 16.
We find it from the condition log12 27 = a :

3 3 3 3log 2 3
a  log12 27  3log12 3    
log 3 12 1  2log3 2 1  2 2  log 2 3
log 2 3

2a
which means that log 2 3  (Note that, obviously, a  3).
3a

4(3  a)
We finally have log6 16  .
3a
10. Let x = (0.15)20. Find the characteristic and mantissa in the logarithm of x, to the base 10. Assume
log102 = 0.301 and log103 = 0.477.

 15 
Sol. log x = log (0.15)20 = 20 log  
 100 
= 20 [log 15 – 2] = 20 [log 3 + log 5 – 2]
= 20 [log 3 + 1 – log 2 – 2]
= 20 [– 1 + log 3 – log 2]
= 20 [– 1 + 0.477 – 0.301]
= –20 × 0.824 = – 16.48 = 17.52
Hence characteristic = – 17 and mantissa = 0.52

Get 10% Instant Discount On Unacademy Plus [Use Referral Code: MCSIR] 48
Maths IIT-JEE ‘Best Approach’ (MCSIR) Logarithm
11. Find the number of positive integers which have the characteristic 3, when the base of the logarithm is 7.
Sol. log7 N = x where 3  x < 4
 73  N < 74
 number of integers are 2058

12. How many digits are contained in the number 275?


Sol. Computing log 275, we have log 275  75. log 2 = 75. 0.3010 = 22.5750.
Consequently, the characteristic of this common logarithm is equal to 22.
Therefore, 275 = a. 1022. where 1  a < 10, a is an integer, and, hence the number 275 has 23 digits.

13. Solve the equation log3(x2 – 3x – 5) = log3 (7 – 2x).


Sol. The equation is equivalent to the following mixed system :

 x 2  3x  5  7  2x
 2
 x  3x  5  0
7  2x  0

Solving the equation of this system, we get : x1 = 4, x2 = –3.


Of these two values only x = –3 satisfies both inequalities of system (that is, the value x = 4 does not
belong to the domain of equation).
Therefore, x = –3 is the solution.
1
14. Prove that log log3 2 0
2
Sol. SInce 0 < log3 2 < 1 and 1/2 < 1, it follows that the inequality is valid.

15. What can be said about the number x if it is known that for ever real a  0, logx(a2 + 1) < 0 ?
Sol. For every a  0 the number 1 + a2 > 1.
Since the logarithm of a number greater than unity is negative only to a base less than unity, it follows that
0 < x < 1.
1 1
16. Solve the inequality  1
log 2 x log 2 x  1
Sol. Reducing the fractions on the left hand side to a common denominator, we find
1 1  log 2 x(log 2 x  1)
1 0
log 2 x(log 2 x  1) log 2 x(log 2 x  1)
The numerator of the last expression is positive since we have
2
2  1 3
1  log x  log 2 x   log 2 x   
2
 2 4
 log2x (log2x – 1) > 0,
which is fulfilled for x > 2 and 0 < x < 1.

Get 10% Instant Discount On Unacademy Plus [Use Referral Code: MCSIR] 49
Maths IIT-JEE ‘Best Approach’ (MCSIR) Logarithm
17. If x6 – y6 = z6 , (x, y, z are all positive and z  1), then

 1 
logz(x2 – y2) + logz(x2 + y2 – xy) – logz  2 2 
 x  y  xy 
Ans. 6
Sol. logz (x2 – y2) (x2 + y2 – xy) (x2 + x2 + xy)
= logz (x + y) (x – y) (x2 + y2 – xy) (x2 + y2 + xy)
= logz (x + y) (x2 + y2 – xy) · (x – y) (x2 + y2 + xy)
= logz (x3 + y3). (x3 – y3)

18. Let N = 0.00000165. Characteristic of log10 N is :


Ans. –6
Sol. N = 1.65 × 10–6
log N = log (1.65 × 10–6)
= log(1.65)
 
 (
6)
mantissa characteristic

19. a, b, c are 3 numbers, Characteristic of thier logarithms to the base 10 are 5, –3, 2 respectively. The
maximum numbers of digits in N = abc, could be
Ans. 7
Sol. Given
log a = 5 + 1
log b = –3 + 2
log c = 2 + 3
where 1 , 2 , 3 are mantissas.
   [0, 1)
Adding the above equations
log abc = 4  
1  2  3

can vary from [0,3)

 log (abc)  [4, 7)  log N  [4, 7)


 Maximum possible characteristic of log N = 6
 Maximum no. of digits = 6 + 1 = 7
20. Number of solutions of the equation.

 7x  3x 2  4 
log10  2   log10 (0.9) is/are
 5x  2x  3 
Ans. 0
Sol. 0.9  1

7x  3x 2  4
 on solving 1
5x  2x 2  3
We get x = 1
0
For x = 1 we get   which is not defined.
0  

Get 10% Instant Discount On Unacademy Plus [Use Referral Code: MCSIR] 50
Maths IIT-JEE ‘Best Approach’ (MCSIR) Logarithm
EXERCISE–I
Solve the following equations
(Note: whenever log x appear it will be treat as logarithm of x with base 10) :
Q.125 logx – 13 = 2
1
Q.126 log4(2log3(1 + log2(1 + 3log3x))) = .
2

Q.127 log3(1 + log3(2x – 7)) = 1

Q.128 log3(3x – 8) = 2 –x

log 2 (9  2x )
Q.129 1
3 x

Q.130 log5 – x(x2 – 2x + 65) = 2

 1 
Q.131 log 3  log 9 x   9 x   2x
 2 

Q.132 log3(x + 1) + log3(x + 3) = 1

Q.133 log7(2x – 1) + log7(2x – 7) = 1

Q.134 log5 + log(x + 10) – 1 = log(21x – 20) – log(2x – 1)

1 1 1 
Q.135 1  log 5   log  log x  log 5 
3 2 3 

1  1  1 1  1
Q.136 log x  log  x    log  x    log  x  
2  2  2 2  8

Q.137 3log3 log x


 log x  log 2 x  3  0

2
(x  2)  log(x  2)5 12
Q.138 (x  2)log  102log(x 2)

Q.139 9log3 (12x)  5x 2  5

Q.140 x1+ logx = 10x

Q.141 x2logx = 10x2

log x  5
Q.142 x 3  105 log x
Q.143 x log3 x  9

Get 10% Instant Discount On Unacademy Plus [Use Referral Code: MCSIR] 51
Maths IIT-JEE ‘Best Approach’ (MCSIR) Logarithm
log5 x 1
Q.144  x 5

Q.145 xlogx + 1 =106

log x  7
Q.146 x 4  10log x 1

Q.147 log x (x  2)
x 9

log 2 x  log x 2  2
 log x 
Q.148  2   log x
 

Q.149 3 log 2 x  log 2 8x  1  0

Q.150 log2x – 3logx = log(x2) – 4

Q.151 log1/3 x  3 log1/3 x  2  0

2
Q.152 
2 log x 5   3log x 5  1  0

Q.153 log 22 x  2log 2 x  2  0

log b x 2
Q.154 a   5x logb a  6  0

1
Q.155 log2 (100x) + log2 (10x) = 14 + log  
x

Q.156 log4 (x +3) – log4 (x – 1) = 2 – log48

Q.157 log4(x2 – 1) – log4(x – 1)2 = log 4 (4  x) 2

x 3 x 3
Q.158 2 log 3  1  log 3
x7 x 1

Q.159 2log4(4 – x) = 4 – log2(–2 – x)

Q.160 3 + 2logx + 13 = 2log3(x + 1)

Q.161 logx (9x2). log32 x = 4

Get 10% Instant Discount On Unacademy Plus [Use Referral Code: MCSIR] 52
Maths IIT-JEE ‘Best Approach’ (MCSIR) Logarithm

2  x2 
Q.162 log1/2 (4x)  log 2    8
 8 

Q.163 log 0.5x x 2  14log16x x 3  40log 4x x  0

4 2log 3 3
Q.164 6  (1  4  9 )  log 7 x  log x 7 , x  Q

Q.165 log3(4.3x– 1) = 2x + 1

Q.166 log3(3x – 6) = x – 1

Q.167 log3(4x – 3) + log3(4x – 1) = 1

Q.168 log 3  log1/2 2 x  3log1/2 x  5   2

 2x   2 
Q.169 log 5    log 5  
 10   x 1

Q.170 1+ 2 log(x +2)5 = log5(x +2)

Q.171 log 4 24x  2log2 4

x 15
Q.172 log 2   
 4  log x  1
2
8

1  2(log x) 2
Q.173 1
log x  2(log x) 2

Q.174 log2(4.3x –6) – log2(9x – 6) = 1

1
Q.175 log(5x  4)  log x  1  2  log 0.18
2

Q.176 log4(2.4x–2 –1) + 4 =2x

Q.177 log x 5  log x (5x)  2.25  (log x 5) 2

Q.178 log(log x) + log(log x4– 3) = 0

Q.179 log3x – 2 log1/3x = 6

Get 10% Instant Discount On Unacademy Plus [Use Referral Code: MCSIR] 53
Maths IIT-JEE ‘Best Approach’ (MCSIR) Logarithm

2 log x
Q.180 1
log(5x  4)

4
Q.181 2 log8 (2x)  log8 (x 2  1  2x) 
3

1 1
Q.182 log 2 (x  2)   log1/8 3x  5
6 3

Q.183 2 log3(x – 2) + log3(x – 4)2 = 0

Q.184 log 2 (2x 2 )  log 4 (16x )  log 4 x 3

3log x  19
Q.185  2log x  1
3log x  1

Q.186
log  x 1 1 3
log 3 x  40

Q.187 log 32 6  log 32 2 = (log2 x – 2) log312


EXERCISE–II
1. Which is the correct order for a given number d, d > 1
(A) log2 d < log3 d < loge d < log8 d
(B) log8 d < log3 d < loge d < log2 d
(C) log8 d < loge d < log2 d < log3 d
(D) log3 d < loge d < log2 d < log8 d

2. If log 27 = 1.431 then the value of log 9 is

3. If x = log2a a, y = log3a 2a, z = log4a 3a, then the value of x y z + 1 is


(A) yz (B) 2yz (C) y + z (D) y – z

4. Let 3a = 4, 4b = 5, 5c = 6, 6d = 7, 7e = 8, and 8f = 9, then find the value of the product abcdef :

5. If log5 a . loga x = 2, then x is equal to :

81 25 16
6. Value of 3 log + 5 log + 7 log is
80 24 15

Get 10% Instant Discount On Unacademy Plus [Use Referral Code: MCSIR] 54
Maths IIT-JEE ‘Best Approach’ (MCSIR) Logarithm

2
7. f(x) = log10 x . The set of all values of x for which f(x) is real is :
log x alog a ylog y z
8. Value of x :

9. If log2 x + log2 y  6, then the least value of x + y is :

10. A rational number which is 50 times its own logarithim to the base 10 is :

11. If x = log5 (1000) and y = log7 (2058) then which is greater ?

12. If 4log9 3  9log 2 4  10log x 83 then x is equal to :

 a  b  na  nb a b
13. If n   then  is equal to :
 3  2 b a

14. If log 2 = 0.30103 the no. of digits in 264 is :


15. The number of zeroes coming immediately after the decimal point in the value of (0.2)25 is :
(Given log10 2 : 0.30103)

16. Simplify : 7 log3 5  3log5 7  5log3 7  7 log5 3


17. The solution set of log2 |4 – 5x| > 2 is :

1
18. If  log0.1 x  2 then 'x' belongs to :
2

19. The solution set of log11 log7  


x  5  x = 0 is :

20. Solve for 'x' in the equation : ln(x – 3) + ln(x – 2) = ln(2x + 24) :

EXERCISE–III

 ab  (ab) 2  4(a  b)   2 
1. Let A denotes the value of log10   + log  ab  (ab)  4(a  b) 
  10  
2 2
   
when a = 43 and b = 57
and B denotes the value of the expression 2log6 18 · 3log6 3 .  
Find the value of (A · B).
2. (a) If x = log34 and y = log53, find the value of log310 and log3(1.2) in terms of x and y.
log2 5 2
(b) If k = 16, find the value of k (log2 5) .

Get 10% Instant Discount On Unacademy Plus [Use Referral Code: MCSIR] 55
Maths IIT-JEE ‘Best Approach’ (MCSIR) Logarithm
Solve for x (3 to 5)
3. (a) If log10 (x2  12x + 36) = 2 (b) 91+logx  31+logx  210 = 0 ; where base of log is 3.

log b log b N 
log b a
4. Simplify : (a) log1/3 4 729.3 9 1.27 4 / 3 ; (b) a

5. (a) If log4 log3 log2 x = 0 ; (b) If loge log5 [ 2 x  2  3] = 0

6. (a) Which is smaller? 2 or (log2 + log2 ). (b) Prove that log35 and log27 are both irrational.

7. Let a and b be real numbers greater than 1 for which there exists a positive real number c, different
from 1, such that
2(logac + logbc) = 9logabc. Find the largest possible value of logab.

8. Find the square of the sum of the roots of the equation


log3x · log4x · log5x = log3x · log4x + log4x · log5x + log5x · log3x.
2 3
9. Find the value of the expression 6
 .
log 4 (2000) log5 (2000) 6
5log
4 2
3 6 6log8  3 2 
10. Calculate : 4
1 3
log 9 log 3
5 6 2
81 3  
11. Simplify :
409
.  7 log 25 7
125log 25 6 
 

12. Simplify : 5
 log
log1/ 5 12 4
log1 / 2
1
.
2 7 3 10 2 21
2 2
13. Find 'x' satisfying the equation 4log10 x 1 – 6log10 x – 2.3log10 x = 0.
2 a 2 b5
14. Given that log2a = s, log4b = s2 and log 2 (8) = 3 . Write log2 4 as a function of 's'
c s 1 c
(a, b, c > 0, c  1).
1log 7 2   log 5 4
15. Find the value of 49 +5 .
16. Given that log2 3 = a , log3 5 = b, log7 2 = c, express the logarithm of the number 63 to the base
140 in terms of a, b & c.
log 2 24 log 2192
17. Prove that  = 3.
log96 2 log12 2

18. Prove that ax – by = 0 where x = loga b & y = logb a , a > 0 , b > 0 & a , b  1.
log3 7 log7 11 log11 25
19. If a, b, c are positive real numbers such that a = 27 ; b = 49 and c = 11 . Find the
 (log 7 )2
value of  a 3  b 7
(log 11) 2 (log 25)2 
 c 11 .
 
log10 ( x 3) 1
20. (a) Solve for x , 
log10 x 21  2
 2
(b) log (log x) + log (log x3  2) = 0 ; where base of log is 10 everywhere.

Get 10% Instant Discount On Unacademy Plus [Use Referral Code: MCSIR] 56
Maths IIT-JEE ‘Best Approach’ (MCSIR) Logarithm
(c) logx2 . log2x2 = log4x2 (d) 5logx + 5 xlog5 = 3 (a > 0) ; where base of log is a.
10 xy
21. If x, y > 0, logyx + logxy = and xy = 144, then = N where N is a natural number, find the
3 2
value of N.

22. Solve the system of equations:


log a x log a ( xyz)  48
log a y log a ( xyz)  12 , a > 0, a  1.
log a z log a ( xyz)  84

23. (a) Given : log1034.56 = 1.5386, find log103.456 ; log100.3456 & log100.003456.

(b) Find the number of positive integers which have the characteristic 3, when the base of the
logarithm is 7.
(c) If log102 = 0.3010 & log103 = 0.4771, find the value of log10(2.25).

(d) Find the antilogarithm of 0.75, if the base of the logarithm is 2401.

24. If log102 = 0.3010, log103 = 0.4771. Find the number of integers in :

(a) 5200 (b) 615 & (c) the number of zeros after the decimal in 3100.

25. Let 'L' denotes the antilog of 0.4 to the base 1024.
and 'M' denotes the number of digits in 610 (Given log102 = 0.3010, log103 = 0.4771)
and 'N' denotes the number of positive integers which have the characteristic 2, when base of the
logarithm is 6.
Find the value of LMN.
EXERCISE–IV
Note : From Q.1 to Q.7, solve the equation for x :
1. xlogx+4 = 32, where base of logarithm is 2.

2. logx+1 (x² + x  6)2 = 4 3. x + log10(1 + 2x) = x . log105 + log106.

4. 5logx – 3logx-1 = 3logx+1 – 5logx-1, where the base of logarithm is 10.


1  log 2 ( x  4)
5. =1
log ( x  3  x  3)
2
6. log5 120 + (x  3)  2 . log5 (1  5x-3) = log5(0.2  5x-4)
 1 
7. log 4 + 1   log 3 = log
 2x 
x 3  27  .
x
8. If 'x' and 'y' are real numbers such that, 2 log(2y – 3x) = log x + log y, find .
y
9. The real x and y satisfy log8x + log4y2 = 5 and log8y + log4x2 = 7, find xy.

10. If a = log1218 & b = log2454 then find the value of ab + 5 (a  b).

Get 10% Instant Discount On Unacademy Plus [Use Referral Code: MCSIR] 57
Maths IIT-JEE ‘Best Approach’ (MCSIR) Logarithm
11. If x = 1 + logabc , y = 1 + logbca, z = 1 + logcab, then prove that xyz = xy + yz + zx.

12. If p = loga bc, q = logb ca, r = logc ab, then prove that pqr = p + q + r + 2.

13. If logba . logca + logab . logcb + logac . logb c = 3 (Where a, b, c are different positive real numbers  1),
then find the value of abc.

14. Let y = log 2 3 ·log 2 12 ·log 2 48 ·log 2 192  16 – log212 · log248 + 10. Find y  N.

3
15. Solve the equation log4(x + 2)2 + 3 = log4(4 – x)3 + log4(6 + x)3.
2
16. Find the product of the positive roots of the equation (2008) ( x ) log2008 x  x 2 .

4 4  2 2
17. Find x satisfying the equation log 2 1    log 2 1    2 log 

 1 .
 x  x4  x 1 

18. Solve : log3  x  


x  1 = log9 4 x  3  4 x 1 
 
 log 4 ab  log 4 ab  log 4 b  log 4 a  · log b
 a b a a b b a  2 if ba 1
19. Prove that : 2  =  loga b
2 if 1ba

 1  1
20. Solve for x : log2 (4  x) + log (4  x) . log  x    2 log2  x   = 0.
 2  2

EXERCISE–V
1
1. Solve the following equations for x & y : log100|x + y| = [REE 96, 6]
2
log10 y – log10|x| = log100 4.
2. Find all real numbers x which satisfy the equation, [REE 96, 6]


2 log2(log2 x) + log1/2 log2 2 2 x = 1
3. log3/4log8 (x2 + 7) + log1/2 log1/4 (x2 + 7)1 =  2. [REE 2000]
4. Number of solutions of log4(x – 1) = log2(x – 3) is
(A) 3 (B) 1 (C) 2 (D) 0 [JEE 2001 (Screening)]
5. Let (x0, y0) be solution of the following equations
(2x)ln2 = (3y)ln3 [JEE 2011]
3lnx = 2lny
1 1 1
Then x0 is : (A) (B) (C) (D) 6
6 3 2

 
 1 1 1 1
6. The value of 6 + log 3 4 4 4 ...  is [JEE 2012]
2
 3 2 3 2 3 2 3 2 
 

Get 10% Instant Discount On Unacademy Plus [Use Referral Code: MCSIR] 58
Maths IIT-JEE ‘Best Approach’ (MCSIR) Logarithm
1 1

7. The value of  (log 2 9)2  log2 (log2 9)  ( 7) log4 7 is [JEE Adv. 2018]

8. Consider the statement : "P(n) : n2 – n + 41 is prime,. " then which one fo the following is
true ? [Jee main 2019 (10-01-2019-shift-1)]
(A) Both P(3) and P(5) are true (B) P(3) is false but P(5) is true
(C) Both P(3) and P(5) are false (D) P(5) is false but P(3) is true.

9. The number of solutions of the equation log4(x – 1) = log2(x – 3) is :


[Jee main 2021 (27-02-2021-shift-1)]

10. The number of solutions of the equation


log(x +1) (2x2 + 7x + 5) + log(2x + 5) (x +1)2 – 4 = 0, x > 0 ,is____.[Jee main 2021 (20-07-2021-shift-2)]

Get 10% Instant Discount On Unacademy Plus [Use Referral Code: MCSIR] 59
Maths IIT-JEE ‘Best Approach’ (MCSIR) Logarithm

QUESTION BANK

[STRAIGHT OBJECTIVE TYPE]


1a  b
2(1 b )
Q.1 If 60a = 3 and 60b = 5 then the value of 12 equals
(A) 2 (B) 3 (C) 3 (D) 12

log b c a  log cb a


Q.2 Let ABC be a triangle right angled at C. The value of log a ·log a (b + c  1, c – b  1) equals
b c cb
(A) 1 (B) 2 (C) 3 (D) 1/2

 x  8 2  x
Q.3 The set of values of x satisfying simultaneously the inequalities  0 and
log 0.3  10
7 log 2 5  1
2x  3  31 > 0 is :
(A) a unit set (B) an empty set
(C) an infinite set (D) a set consisting of exactly two elements .

Q.4 If log0.3(x – 1) < log0.09 (x – 1) , then x lies in the interval


(A) (2 , ) (B) (1 , 2) (C) (1, ) (D) none of these

2 x  2007 
Q.5 Number of integral values of x the inequality log10    0 holds true, is
 x 1 
(A) 1004 (B) 1005 (C) 2007 (D) 2008

[MULTIPLE OBJECTIVE TYPE]


Q.6 If y = log7–a (2x2 + 2x + a + 3) is defined
 x  R, then possible integral value(s) of a is/are
(A) – 3 (B) – 2 (C) 4 (D) 5

Get 10% Instant Discount On Unacademy Plus [Use Referral Code: MCSIR] 60
Maths IIT-JEE ‘Best Approach’( MCSIR ) Logarithm

DPP-1
Time : 35 Min.

Straight Objective Type

1. If a4 · b5 = 1 then the value of loga(a5b4) equals


(A) 9/5 (B) 4 (C) 5 (D) 8/5

2. If log175 5x = log343 7x, then the value of log42 (x4–2x2+7) is


(A) 1 (B) 2 (C) 3 (D) 4

3. The sum of all the solutions to the equation 2 log x – log(2x – 75) = 2
(A) 30 (B) 350 (C) 75 (D) 200

4. 10
log p (log r x ))
= 1 and logq (log r (logp x)) = 0, then 'p' equals
(A) rq/r (B) rq (C) 1 (D) r r/q

5. The value of log10  3 5  3 5  is


(A) 1/ 2 (B) 1/4 (C) 3/2 (D) 3/4

B
6. If log4 A = log6 B = log9 (A +B) then the value of is
A

5 1 5 1
(A) (B)
4 4

5 1 5 1
(C) (D)
2 2

Multiple Objective Type


x
7. If p, q  N satisfy the equation x x
  x then p & q are
(A) relatively prime (B) twin prime
(C) coprime (D) if logqp is defined then logpq is not & vice versa

8. Which of the following when simplified, reduces to unity ?


2 log 2  log 3 1  64 
(A) log105 · log1020 + log102 2 (B) (C)  log5 log3 5
9 (D) log 3  
log 48  log 4 6 2  27 

61
Maths IIT-JEE ‘Best Approach’( MCSIR ) Logarithm
1  2 log3 2
9. The number N = 2
 log26 2 when simplified reduces to
1  log3 2
(A) a prime number (B) an irrational number
(C) a real which is less than log3 (D) a real which is greater than log76

DPP-2
Time : 35 Min.

Straight Objective Type

1. The greatest integer less than or equal to the number log2 15 × log1/6 2×log3 1/6 is
(A) 4 (B) 3 (C) 2 (D) 1

2. Given that log23 = a, log35 = b, log72 = c, then the value of log140 63 is equal to
2  ac 1  ac
(A) (B)
2c  1  abc c  2  abc

1  2ac 2  ac
(C) (D)
2c  1  abc c  2  abc

3. The real value of x for which the statement log69 – log927 + log8x = log64x – log64 holds true, is
(A) 1/2 (B) 1/4 (C) 1/8 (D) 1/16

log 2 x log 2 y log 2 z


4. If   and x2y2z =1 , then k is equla to
4 6 3k
(A) –8 (B) – 4 (C) 0 (D) 4

5. A line x = k intersects the graph of y = log4x and y =log4 (x + 4). The distance between the points of
intersection is 0.5, then the value of k is
(A) 1 (B) 2 (C) 3 (D) 4

3 log 1353 log 5


6. The value of log 3  log 3 is
15 405

(A) 1 (B) 2 (C) 3 (D) 4

7. There exist positive integers A,B and C with no common factors greater than 1, such that
A log200 5+B log 2002 = C. The sum A+B+C equals
(A) 5 (B) 6 (C) 7 (D) 8
8. The value of  2 log 18  .  3log
6 6 3
 is
(A) 5 (B) 6 (C) 7 (D) 8

62
Maths IIT-JEE ‘Best Approach’( MCSIR ) Logarithm
Multiple Objective Type
log8 (8 x 2 )
9. The equation = 3 has :
(log8 x ) 2
(A) no integral solution (B) one natural solution (C) two real solutions (D) one irrational solution

10. The expression, logp logp ...... p p where p  2, p  N, when simplified is


p p p

 
n radical sign
(A) independent of p, but dependent on n (B) independent of n, but dependent on p
(C) dependent on both p & n (D) negative.

Match The Column

11. Column-I Column-II


Following expressions simplifies to
log 2 32
(A) log3 243 (P) positive integer

2 log 6
(B) (Q) negative integer
log12  log 3

2
(C) log1/4  1  (R) rational but not integer
 16 

log5 16  log5 4
(D) (S) prime
log5 128

DPP-3
Time : 35 Min.

Straight Objective Type

1. Which one of the following is the smallest ?


3
 1   1 
 
(A) log10 (B) log10  2
(C)  
 (D)  
log
 10  log
 10  

2. If log 2 log 3 log 4 ( x )   0 and log3 log 4 log 2 ( y)   0 and log 4 log 2 log3 (z)   0 then the sum
of x, y and z is
(A) 89 (B) 58 (C) 105 (D) 50

1
63
Maths IIT-JEE ‘Best Approach’( MCSIR ) Logarithm
c
3. If log2 b = 2, logbc = 2 and log3c = 3 + log3a , then the value of is
ab
(A) 1 (B) 3 (C) 9 (D) 27

log e 10
4. If log2 10 = p : log 7 = q and (11)r = 10, then which one of the following expression is equivalent ot
e

log10154 ?
1 pqr pq  qr  rp
(A) pqr (B) pqr (C) pqr (D) pqr

5. If 3(log37)x =7 (7log73)x , then the value of x will be


1 1
(A) (B)
2 4

1
(C) (D) 1
3

6. The value of 45log  3 6   6 log 


4 2
8 3 2  is

(A) 3 (B) 6 (C) 9 (D) 27

1 3
3
log59 log 6
7. The value of 81 3  log2257 log 6 
 7  125 25 
409  
(A) 0 (B) 1 (C) 2 (D) 3

8. The value of 6log 10


40
.5 log1036 is
(A) 200 (B) 216 (C) 432 (D) none of these

Multiple Objective Type

9. Consider the quadratic equation, (log108)x2 – (log105)x = 2(log210)–1 – x. Which of the following
quantities are irrational.
(A) sum of the roots (B) product of the roots
(C) sum of the coefficients (D) discriminant

10. In which of the following case(s) the real number 'm' is greater than the real number 'n'?
(A) m = (log25)2 and n = log220 (B) m = log102 and n = log10 3 10
(C) m = log105 · log1020 + (log102)2 and n=1 (D) m = log1/2 1 3 and n = log1/3 1 2

64
Maths IIT-JEE ‘Best Approach’( MCSIR ) Logarithm
Match The Column
11. Column-I Column-II
(A) The value of K, (P) 1
where log (log 4) + log (log 25) = log K + log (log 2) + log (log 5) is
(B) Number of values of x  N, for which x4 + 4 is prime, is (Q) 2
(C) If b is a positive real number different from 1, let logbx denotes the base b
logarithm of x. Let n be the number of solutions of x to the equation, (R) 3
logbx = logxb where x is a positive real different from 1. Then n equals
(D) The expression log0.52 8 has the value equal to (S) 4

DPP-4
Time : 35 Min.

Straight Objective Type

1. If x = 2  5 , then log
5 2
10x =  
A  B (log102) + B . The value of (A + B) equals
(A) 7 (B) 9 (C) 11 (D) 13

2. Let u = (log2x)2 – 6 log2x + 12 where x is a real number. Then the equation xu = 256 has
(A) no solution for x (B) exactly one solution for x
(C) exactly two distinct solutions for x (D) exactly three distinct solutions for x

 
1 2 1 1
3. The value of log 9  6 6 6 ..... 

4 2 3 2 3 2 3 2 3 
 
(A) – 2 (B) – 1 (C) – 1/2 (D) none of these

4. Number of real values of x satisfying the equation log x 2


 6 x 8
(log log 2x 2
 2x  3
(x2–2x)) = 0 is equal to
(A) 3 (B) 2 (C) 1 (D) 0

5. If logab + logbc + logca vanishes where a, b and c are positive reals different than unity then the value
of (logab)3 + (logbc)3 + (logca)3 is
(A) an odd prime (B) an even prime
(C) an odd composite (D) an irrational number

Subjective

6. Solve the equation, log( x )  log x 2 (base is 10)

65
Maths IIT-JEE ‘Best Approach’( MCSIR ) Logarithm

7. If log a  log b  log c , show that aa . bb . cc = 1.


bc ca a b

8. Find the value of x satisyfing the equations log3 (log2x) + log1/3 (log1/2y) = 1 and xy2 =9

3
9. Solve : log4 (x+2)2 + 3 = log4 (4–x)3 + log4 (6 + x)3
2
10. Solve : log3/4 log8 (x2 +7) + log1/2 log1/4 (x2+7)–1 = –2

11. If 5log x – 3 log x–1 = 3log x+1 –5 log x–1 , where the base of logarithm is 10, then find the value of x.

12. Slove : 4 | x  3 |x 1  3 | x  3 |x  2
13. Solve log x 16 + log2x + 64 = 3
2

DPP-5
Time : 40 Min.

Straight Objective Type


1. If logab = 2; logbc = 2 and log3c = 3 + log3a then (a + b + c) equals
(A) 90 (B) 93 (D) 102 (D) 243

2. Let 3a = 4, 4b = 5, 5c = 6, 6d = 7, 7e = 8 and 8f = 9. The value of the product (abcdef), is


(A) 1 (B) 2 (C) 6 (D) 3

3. If log9x + log4y = 7 2 and log9x – log8y = – 3 2 , then x + y equals


(A) 35 (B) 41 (C) 67 (D) 73

4. Let A denotes the real value of x satisfying the equation x3 + 3x2 + 3x + 4 = log12(1728) and

B = 132  132  132......... then (A – B) equals


(A) 1 (B) 13 (C) – 12 (D) – 13

Multiple Objective Type

5. If log45 = x and log56 = y then


1 1
(A) log46 = xy (B) log64 = xy (C) log32 = (D) log23 =
2 xy  1 2 xy  1

66
Maths IIT-JEE ‘Best Approach’( MCSIR ) Logarithm
6. If log1/2 (4–x)  log1/2 f2–log1/2 (x –1) , then x can belong to
(A) (1,2] (B) [1,3] (C) [3,4] (D) [2,3]

9 1
7. The values of x satisfying 2 log 1 (x  5) 4 log 3 3  9   log x 5 (2) is / are
4

(A) (–5,– 4) (B) (–3, –1) (C) (–4, –1) (D) (–5, –2)
3
8. If log x – (log3 x)2  log 4, then x can belong to
2 1/ 2 2 
(A) (–, 1/3) (B) (9, )
(C) (1,6) (D) (–,0)

Match The Column

9. Column-I Column-II
log10 x
(A) If x1 and x2 satisfy the equation x = 100x (P) irrational
then the value of x1x2 equals
(B) Sum of the squares of the roots of the equation log2(9 – 2x) = 3 – x is (Q) rational
(C)   
If log1 8 log1 4 log1 2 x = 1/3 then x is (R) prime

(D) If logba = 3, logbc = – 4 and if the value of x satisfying the equation, a3x =cx – 1 (S) composite
is expressed in the form p/q, where p & q are relatively prime then (p+q) is

Subjective
10. Establish tricotomy in each of this following pairs of numbers
log 27 3 log 2
(i) 3 2 4and (ii) log45 and log1/16(1/25)
(iii) 4 and log310 + log1081 (iv) log1/5(1/7) and log1/7 (1/5)

11. Compute the value of


1 4
log 5 3 log 9 36 log 7 9
81  27  3

12. Let a and b real number greater than 1 for which there exists a positive real numbers c. different from 1,
such that 2(logac +logbc) = 9 logabc. Find the largest possible value of logab.

13. Solve : log3x. log4x. log5x = log3x. log4x + log4x. log5x+ log5x. log3x.

14. Solve the system of equations :


(logax) (loga (xyz)) = 48
(logay) (loga(xyz)) = 12
(logaz) (loga(xyz)) = 84

67
Maths IIT-JEE ‘Best Approach’( MCSIR ) Logarithm
DPP-6
Time : 40 Min.

Straight Objective Type


1. Let B, C, P and L be positive real numbers such that
log (B · L) + log (B · P) = 2; log (P · L) + log (P · C) = 3; log (C · B) + log (C · L) = 4
The value of the product (BCPL) equals (base of the log is 10)
(A ) 102 (B) 103 (C) 104 (D) 109

1
2. If x = logkb = logbc = logcd then logkd equals
2

x3
(A) 2x3 (B) (C) 2x8 (D) 6x
2

3. If log2 (log2(log2(log2x) = 2 , then the number of digits in x, is (log102 = 0.3010)


(A) 7 (B) 6 (C) 5 (D) 4

4. The number of integers satisfying the inequality log 0.9 log5  


x 2  2  x > 0 is

(A) 6 (B) 7 (C) 8 (D) 9

1  log 4 x 1
5. The smallest integra x satisfying the inequality 1  log x  2 x is
2

(A) 2 (B) 2 (C) 3 (D) 4

6. The number of integral solutions of log9 (x + 1) log2 (x +1) – log9 (x +1) – log 2 (x +1) + 1 < 0 is

(A) 4 (B) 5 (C) 6 (D) 7

 2  x  2 
7. The number of integers satisfying log log 1   x  1 x  5    1 is
x  
(A) 0 (B) 0 (C) 2 (D) 3

68
Maths IIT-JEE ‘Best Approach’ (MCSIR) Logarithm

PART TEST – 1
Single Correct Answer Type

 1 
1. Find the value of x satisfying the equation log3  log9 x   9x   2x is
 2 
1 1 1
(A) (B) (C) (D) none of these
2 3 4
log x 7
2. Number of real values of x satisfying the equation x 4  10log x 1 is equal to
(A) 1 (B) 2 (C) 3 (D) 4

x 3 x 3
3. Find the value of x satisfying the equation 2log3  1  log3 is
x 7 x 1
(A) 5 (B) 4 (C)  4 (D)  5

4. If log2 x + log2 y  6, then the least value of x + y is :

(A) 8 (B) 16 (C) 32 (D) 12

5. Let a and b be real numbers greater than 1 for which there exists a positive real number c,
different from 1, such that 2(logac + logbc) = 9logabc. Find the largest possible value of logab.
(A) 0 (B) 1 (C) 2 (D) 3

x
6. If 'x' and 'y' are real numbers such that, 2 log(2y – 3x) = log x + log y, find .
y

4 4 9 3
(A) 1, (B) 1, (C) 1 , (D) 1,
3 9 4 4

7. Sum of all values satisfying the equation log( x)  log x 2 is (when base is 10)
(A)  9 (B)  10 (C)  1 (D)  11

8.  
If log1 8 log1 4  log1 2 x  = 1/3 then x is

(A) rational (B) irrational (C) integer (D) prime

69
Maths IIT-JEE ‘Best Approach’ (MCSIR) Logarithm

Integer Type
10 x y
9. If x, y > 0, logyx + logxy = and xy = 144, then = N where N is a natural number,
3 2
find the value of N.

10. If logx log18  


2  8 =–
1
2
. Then the value of 'x' is equal to

11. The value of K, where log (log 4) + log (log 25) = log K + log (log 2) + log (log 5) is

log a log b log c


12. If   then value of aa . bb . cc is equal to
b c c a a b

13. The equation log1/3(x2 + 8) = – 2 has


(A) One real solutions. (B) Two real solutions.

(C) More then two solutions (D) None of these

1
14. Let N = log215 · log1/62 · log3   . The greatest integer which is less than or equal to N is
6

log 64  log 9
15. Let A = log32 2 8 ; B = 5log5 2log5 3 ; C = ; D = 2log2 301
2 log 24
find the value of ( A + B + C)  D.

70
Maths IIT-JEE ‘Best Approach’ (MCSIR) Logarithm

PART TEST – 2
Single Correct Answer Type

1. Find the value of x satisfying the equation 9log3 (12x)  5x 2  5 is

(A) – 2 – 10 (B) – 2 + 10 (C) 2 + 10 (D) 2 – 10

2. Given log102 = a and log103 = b. if 3x + 2 = 45. The value of x in terms of a and b is


a 1 1 a 1 a b
(A) (B) (C) (D)
b b b 1 a

3. Number of real values of x satisfying the equation log 22 x  2log 2 x  2  0 is equal to


(A) 1 (B) 2 (C) 3 (D) 4

4. Find the value of x satisfying the equation log3(4x – 3) + log3(4x – 1) = 1 is


(A) 0 (B) 1 (C) 2 (D) 3

 a  b  na  nb a b
5. If n   then  is equal to :
 3  2 b a

(A) 5 (B) 6 (C) 7 (D) 8

6. Find the value of x satisfying the equation ln(x – 3) + ln(x – 2) = ln(2x + 24) is
(A) 3 (B) 6 (C) 9 (D) 12

7. The real value of x for which the statement log69 – log927 + log8x = log64x – log64 holds true,
is
(A) 1/2 (B) 1/4 (C) 1/8 (D) 1/16

8. If x = 2  5  , then log
5 2
10x =  
A  B (log102) + B . The value of (A + B) equals
(A) 7 (B) 9 (C) 11 (D) 13

9. If logab = 2; logbc = 2 and log3c = 3 + log3a then (a + b + c) equals


(A) 90 (B) 93 (D) 102 (D) 243

10. If log9x + log4y = 7 2 and log9x – log8y = – 3 2 , then x + y equals


(A) 35 (B) 41 (C) 67 (D) 73

71
Maths IIT-JEE ‘Best Approach’ (MCSIR) Logarithm

Integer Type
1 3

3
.   7 
log 9 log 6 3
81 2

 125
5
log25 7 log 25 6
11. find the value of 
409  

12. Let 'L' denotes the antilog of 0.4 to the base 1024.
and 'M' denotes the number of digits in 610 (Given log102 = 0.3010, log103 = 0.4771)
and 'N' denotes the number of positive integers which have the characteristic 2, when base of
the logarithm is 6.
Find the value of LMN.
1 4

 
log 9 36
13. Find the value of 81log 5 3 27 3log 7 9
14. If log8a + log8b = (log8a)(log8b) and logab = 3, then the value of 'a' is

15. If a = log1218 & b = log2454 then find the value of ab + 5 (a  b).

72
Maths IIT-JEE ‘Best Approach’ (MCSIR) Logarithm

ANSWER KEY
EXERCISE–I

Q.125 1  3  Q.126 {3} Q.127 {4} Q.128 {2}

Q.129 {0} Q.130 {–5} Q.131 {1/3} Q.132 {0}

Q.133 {3} Q.134 {3/2, 10} 


Q.135 16 / 3 5  Q.136 {1}

Q.137 {100} Q.138 {2 + 10–7, 3, 102} Q.139 2  10


Q.140 {10–1, 10} Q.141  10 1 3
, 101 3
 Q.142 {10–5, 103}

Q.143 3  2
,3 2
 Q.144 {1/5, 25} Q.145 {10–3, 102} Q.146 {10–4, 10}

Q.147 {5} Q.148 {10–3,10, 100} Q.149 {2, 16} Q.150 {10,104}

Q.151 {1/81, 1/3} Q.152  5,5 Q.153 {1/4, 2}

Q.154 2 log a b

,3loga b a > 0, a  1, b > 0, b  1 Q.155  10 ,10
9
Q.156 {5}

Q.157 3  6 Q.158 {–5} Q.159 {–4} Q.160 (3  


3) / 3,8

Q.161 {1/9, 3} Q.162 {2–7, 2} 


Q.163 1/ 2,1, 4  Q.164 {7}

Q.165 {–1, 0} Q.166 {2} Q.167 {1} Q.168 {1/16,2}


Q.169 {3} Q.170 {–9/5, 23} Q.171 {2} Q.172 {1/2,128}


Q.173 1/ 10, 3 10  Q.174 {1} Q.175 {8} Q.176 {2}

Q.177  5
5,5  Q.178 {10} Q.179 {9} Q.180 {4}

Q.181 {2} Q.182 {3} Q.183 3,3  2 Q.184 {16}

Q.185 {10–5/3, 102} Q.186 {48}


 3
Q.187 10 ,10 3

EXERCISE–II
1. B 2. 0.954 3. B 4. 2 5. x = 25
6. log 2 7. (–, –1]  [1, ) 8. z 9. 16 10. 100
11. x>y 12. 10 13. 7 14. 20 15. 17
 1 1  484
16. 0 17. (–, 0)  (8/5, ) 18.  ,  19. x= 20. x=9
100 10  49

73
Maths IIT-JEE ‘Best Approach’ (MCSIR) Logarithm
EXERCISE–III
xy  2 xy  2 y  2
1. 12 2. (a) , ; (b) 625 3. (a) x = 16 or x =  4 (b) x = 5
2y 2y

4. (a)  1 (b) logb N 5. (a) 8 (b) x = 3 6. (a) 2 7. 2 8. 3721

1
9. 1/6 10. 9 11. 1 12. 6 13. x =
100

1  2ac
14. 2s + 10s2 – 3(s3 + 1) 15. 25 16. 19. 469
2 2c  abc  1
2
20 (a) x=5 (b) x=10 (c) x = 2 or 2  2
(d) x = 2–loga where base of log is 5.

 1 1 1 
21. 507 22. (a4, a, a7) or  4 , , 7 
a a a 

23. (a) 0.5386; 1.5386 ; 3.5386 (b) 2058 (c) 0.3522 (d) 343
24. (a) 140 (b) 12 (c) 47 25. 23040

EXERCISE–IV
1
1. x = 2 or 2. x = 1 3. x = 1 4. x = 100 5. x = 5 6. x = 1 7. x 
32
8. 4/9 9. xy = 29 10. 1 13. abc = 1 14. y = 6 15. x = 2 or 1 – 33
 7 3  24 
16. (2008)2 17. x  2 or 6 18. [0, 1]  {4} 20. 0 , , 
 4 2 

EXERCISE–V

1. {–10, 20}, {10/3, 20/3} 2. X=8 3. X = 3 OR - 3 4. B


5. C 6. 4 7. 8 8. A 9. 1
10. 1

QUESTION BANK

Q.1 A Q.2 B Q.3 A Q.4 A Q.5 B


Q.6 BCD

DPP -1

1 A 2 A 3 D 4 A 5 A
6 D 7 ACD 8 ABC 9 CD

74
Maths IIT-JEE ‘Best Approach’ (MCSIR) Logarithm

DPP -2
1 C 2 C 3 C 4 A 5 D
6 C 7 B 8 A 9 BC 10 AD
11. A-P S:B-P :C-Q : D-R

DPP -3
1 B 2 A 3 B 4 D 5 A
6 C 7 B 8 B 9 CD 10 AD
11. (A - S: B - P : C- Q :D - R)

DPP -4
1 A 2 B 3 C 4 C 5 A
6 x  1, 10 8 x = 729 9 x = 2,1 – 33 10 x= 3
11 x = 100 12 x = 4, 2 or x = 11 13 x = 2–1/3 or x = 4

DPP -5
1 B 2 B 3 C 4 D 5 AC
6 AC 7 AB 8 AB
9 (A)  Q, S; (B)  Q, S; (C)  P; (D)  Q, R
11 891 12 2 13 x = 60 14 x = a4 , y = a and z = a7

DPP -6
1 B 2 A 3 C 4 C 5 B
6 C 7 A

PART TEST -1
1 B 2 B 3 D 4 B 5 C
6 C 7 D 8 B 9 507 10 4
11 4 12 1 13 2 14 2 15 1

PART TEST -2
1 A 2 B 3 B 4 B 5 C
6 C 7 C 8 A 9 B 10 C
11 1 12 23040 13 891 14 16 15 1

75
Maths IIT-JEE ‘Best Approach’ (MCSIR) Logarithm

Logarithm
REVISION PLANNER FOR MAINS

EXERCISE–I
126. 131. 140. 152. 159. 167. 172.
179. 182.

EXERCISE–II
1. 12. 13. 17. 19. 20.

EXERCISE–III
1. 4. 9. 11. 18. 19. 23((b)
24.

EXERCISE–IV
10. 12.

EXERCISE–V
1. 5. 7.

REVISION PLANNER FOR ADVANCED

EXERCISE–I
151. 184.

EXERCISE–III
8. 22.

EXERCISE–IV
5. 6. 7. 14. 17. 18. 19.
20.

EXERCISE–V
3. 6.

76
Maths IIT-JEE ‘Best Approach’ (MCSIR) Logarithm

Logarithm Solved Exercise-I


125. logx – 13 = 2
Sol. logx – 13 = 2

x 1  0
 x 1
Domain : x  1  1   x  2  
 x  [1, ] – {2}
log(x–1)3 = 2 (x – 1)2 = 3
 x=1+ 3 ,1– 3

 x=1+ 3

1
126. log4(2log3(1 + log2(1 + 3log3x))) = .
2

1
Sol. log4(2log3 (1 + log2(1 + 3log3x))) = ....(1)
2
 2log3(1 + log2 (1 + 3 log3x)) = 41/2 = 2
 1 + log2(1 + 3log3x) = 3
 1 + 3log3x = 22 = 4
 log3x = 1  x = 3 : Put in equation (1)
 x = 3 satisfy equation (1)
 x = 3 Ans.

127. log3(1 + log3(2x – 7)) = 1


Sol. log3(1 + log3(2x – 7)) = 1 ......(1)
 1 + log3(2x – 7) = 3
 (2x – 7) = 32 = 9
 2x = 16 x = 4 ; Put in equation (1)
 x = 4 satisfy equation (1) ,  x = 4 Ans.

128. log3(3x – 8) = 2 –x
Sol. log3(3x – 8) = 2 – x .....(1)

9
 3x – 8 = 32–x = ; Put 3x = t
3x
 t2 – 8t – 9 = 0
 t = 9 or t = – 1
 3x = 9 or 3x = – 1 (Reject)
 x = 2 ; Put in equation (1)
x = 2 satisfy eqn (1) , x = 2 Ans.
Maths IIT-JEE ‘Best Approach’ (MCSIR) Logarithm

log 2 (9  2x )
129. 1
3 x

log 2 (9  2 x )
Sol. 1 ....(1)
3 x
 log2(9 – 2x) = 3 – x

8
 9 – 2x = 23–x = ; Put 2x = t
2x
 t2 – 9t + 8 = 0  t = 1, 8
 2x = 1, 8 x = 0 , 3 ; Put in equation (1)
 x = 3 does not satisfy eqn (1)
 x = 0 Ans.

130. log5 – x(x2 – 2x + 65) = 2


Sol. log(5 – x) (x2 – 2x + 65) = 2

x 2  2 x  65  0  xR

  x  5 
Domain : 5  x  0  
  x  4 
5 x 1 

x (–, 5) – {4}


 log(5–x) (x2 – 2x + 65) = 2
 x2 – 2x + 65 = (5 – x2)
 (x = – 5)  Domain
 x = – 5 Ans.

 1 
131. log 3  log 9 x   9 x   2x
 2 

1
Sol. log3  log 9 x   9 x  = 2x ....(1)
 2 

1
 log9x + + 9x = 32x = 9x
2

1 1
 log9x = –  x = 9–1/2 = ; Put in eqn (1)
2 3

1 1
x= satisfy eqn (1) ;  x = Ans.
3 3
Maths IIT-JEE ‘Best Approach’ (MCSIR) Logarithm
132. log3(x + 1) + log3(x + 3) = 1
Sol. log3(x + 1) + log3(x + 3) = 1

x 1  0 
Domain :   x > – 1
x  3  0

 log3((x + 1) × (x + 3)) = 1
 x2 + 4x + 3 = 3
 (x = 0, – 4) Domain
 x = 0 Ans.

133. log7(2x – 1) + log7(2x – 7) = 1


Sol. log7(2x – 1) + log7(2x – 7) = 1 ....(1)
x x
 log7[(2 – 1) (2 – 7)] = 1 ; Put 2x = t
 (t – 1) (t – 7) = 7
 t2 – 8t + 7 = 7
 t = 0 or t = 8
 2x = 0 or 2x = 8

 x = 3 ; Put in eqn (1)
x = 3 satisfy eqn (1) ;  x = 3 Ans.

134. log5 + log(x + 10) – 1 = log(21x – 20) – log(2x – 1)

x  10  0 
20
Sol. Domain : 21x  20  0 x >
21
2 x  1  0 

log5 + log(x + 10) – 1 = log(21x – 20) – log(2x – 1)


 log5 + log(x + 10) – log10 = log(21x – 20) – log(2x – 1)

 5  ( x  10)   21x  20 
 log   = log  
 10   2x  1 

x  10 21x  20
 
2 2x  1
 2x2 – 23x + 30 = 0
 (x = 10 or x = 3/2)  Domain

3
 x = 10, Ans.
2
Maths IIT-JEE ‘Best Approach’ (MCSIR) Logarithm

1 1 1 
135. 1  log 5   log  log x  log 5 
3 2 3 
Sol. Domain : x > 0

1  1 1 
1 – log5 =  log  log x  log 5 
3  2 3 

1 1
 3 (log10 – log5) = log + log x + log 5
2 3

 x 1/ 3 
 log(2)3 = log  .5 
2 

x 1/ 3  16 
 8= .5  x    Domain
2  3 5

16
 x= 3
Ans.
5

1  1  1 1  1
136. log x  log  x    log  x    log  x  
2  2  2 2  8

x0 
1 
x   0
2  1
Sol. Domain : 1   x > 2
x   0
2

1
x   0
8 

1  1  1 1  1
logx – log  x   = log  x   – log  x  
2  2  2 2  8

   x 1 
 x   2 
 log   = log  
 x 1   x 1 
 2  8 

x x 1
 = 2
x 1 x 1
2 8
Maths IIT-JEE ‘Best Approach’ (MCSIR) Logarithm
Square,
2
 1  1  1
 x2  x   = x   x  
 8  2  2

 3x2 – 2x – 1 = 0

 1 
  x   , 1  Domain
 3 


 x=1 Ans.

137. 3log3 log x


 log x  log 2 x  3  0

 x0
 x  0
 
x 0   x  0
Sol. Domain :  x > 1
  x  1 
log x  0

3log 3 (log x)
– logx + ((logx)2 – 3 = 0

 log x – log x + (log x)2 – 3 = 0

1
 logx – logx + (logx)2 – 3 = 0 ; Put log x = t
2

3
 2t2 – t – 6 = 0  t=  ,2
2

3
 logx =  , 2
2
 x = (10)–3/2 , (10)2

 1 
 x  3/ 2
, 100   Domain
 (10) 


 x = 100 Ans.

2
(x  2)  log(x  2)5 12
138. (x  2)log  102log(x 2)
Sol. Domain : x – 2 > 0 x > 2
2
( x  2 )  log( x  2 ) 5 12
( x  2) log  10 2 log( x 2 )

Take log both sides,


Maths IIT-JEE ‘Best Approach’ (MCSIR) Logarithm
 [log2(x – 2) + log(x – 2)5 – 12] × log(x – 2) = 2 log(x – 2) × log10
 [(log(x – 2))2 + 5 log(x – 2) – 12] × log (x – 2) = 2log(x – 2)
Put log (x – 2) = t
 (t2 + 5t – 12) × t = 2t
 t[t2 + 5t – 14] = 0
 t = 0 or (t + 7) (t – 2) = 0
 t = 0 or t = – 7, 2
 log(x – 2) = 0 , 2, – 7
 x – 2 = 1 , 100, 10–7

 1 
  x  3, 102, 2  7   Domain
 10 

1
 x = 3, 102, 2  Ans.
107

139. 9log3 (12x)  5x 2  5

1
Sol. Domain : 1 – 2x > 0  x <
2

9log3 (12 x )  5x 2  5
2 log (1 2 x )
 3 3  5x 2  5
 (1 – 2x)2 = 5x2 – 5
 x2 + 4x – 6 = 0

 (x = – 2 + 10 , – 2 – 10 )  Domain
 

 x = – 2 – 10 Ans.

140. x1+ logx = 10x


Sol. Domain : x > 0
x(1+logx) = 10x
Take log both sides.
 (1 + logx) × logx = log10 + logx
Put log x = t
 (1 + t)t = (1 + t)
 (1 + t) (t – 1) = 0 t = 1 , – 1

 1
 log x = 1, – 1   x  10,   Domain
 10 

1
 x = 10, Ans.
10
Maths IIT-JEE ‘Best Approach’ (MCSIR) Logarithm

141. x2logx = 10x2


Sol. Domain : x > 0
x2logx = 10x2
Take log both sides,
 2logx × logx = log10 + 2logx
Put log x = t
 2t2 – 2t – 1 = 0

 t=1± 3

 logx = 1 ± 3  ( x  101 3 )  Domain

 x  101 3
Ans.

log x  5
142. x 3  105 log x
Sol. Domain : x > 0
log x 5
x 3  105log x
Take log both sides.

log x  5
 × logx = (5 + logx) × log10
3
 Put log x = t

t 
 (t + 5)  1 = 0  t=–5,3
3 
 log x = – 5 , 3 (x = 10–5, 103) Domain
 x = 10–5 , 103 Ans.

142. x log3 x  9
Sol. Domain : x > 0

x log 3 x  9
Take log3 both sides
 log3x × log3x = log39 = 2

 log3x = ± 2  ( x  3 2
)  Domain

 x  3 2
Ans.
Maths IIT-JEE ‘Best Approach’ (MCSIR) Logarithm

log5 x 1
144.  x 5

Sol. Domain x > 0

( x ) lo g 5 x  1 = 5
Take log5 both sides.
 (log5x – 1) × log5 x = log55

1
 (log5x – 1) × × log5x = 1 ; Put log5x = t
2
 t2 – t – 2 = 0 t = 2, – 1

 1
 log5x = 2, –1   x  25,   Domain
 5

1
 x = 25, Ans.
5

145. xlogx + 1 =106


Sol. Domain : x > 0
xlogx+1 = 106
Take log both sides
 (logx + 1) × log x = 6 × log10 ; Put log x = t
 t2 + t – 6 = 0  t = – 3 , 2
 log x = – 3 , 2
 (x = 10–3 , 102) Domain
 x = 10–3 , 102 Ans.

log x  7
146. x 4  10log x 1
Sol. Domain : x > 0
log x  7
x 4  10log x 1
Take log both sides.
log x  7
 × logx = (logx + 1) × log10 ; Put log x = t
4
 t2 + 3t – 4 = 0  t = – 4 , 1
 log x = – 4, 1
 (x = 10–4, 10)
Domain
 x = 10–4 , 10 Ans.
Maths IIT-JEE ‘Best Approach’ (MCSIR) Logarithm

147. log x (x  2)
x 9

x  2  0

Sol. Domain : x  0   x > 2

x 1 

log x ( x  2 )
x 9  x 2 log x ( x 2)  9
2
 x log x ( x 2)  9
 (x – 2)2 = 9 x – 2 = ± 3
 (x = 5 – 1) Domain
 x=5

log 2 x  log x 2  2
 log x 
148.  2   log x
 

Sol. Domain : x > 0

(log 2 x  log x 2  2 )
 log x 
   log x
 2 

Take log both sides

 log x 
 (log2x + 2 logx – 2) × log  = log (log x )
 2 

 log x   log x 
 (log2x + 2logx – 2) × log  – log  0
 2   2 

 log x 
 log   × [log2x + 2logx – 3] = 0
 2 

 log x 
 log   × (logx + 3) (log x – 1) = 0
 2 

log x
  1 , logx = – 3, 1
2
 (x = 102 , 10–3, 10)  Domain
 x = 102, 10, 10–3 Ans.
Maths IIT-JEE ‘Best Approach’ (MCSIR) Logarithm

149. 3 log 2 x  log 2 8x  1  0

x0   x  0
Sol. Domain     x 2
log2 x  1  x  2

3 log 2 x  log 2 (8x )  1  0

 3 log 2 x  3  log 2 x  1  0
Put log2x = t2
 3|t| – t2 – 2 = 0
 |t|2 – 3|t| + 2 = 0
 |t| = 1, 2  t = ± 1, ± 2 t2 = 1, 4
 log2x = t2 = 1, 4
 (x = 2, 16) Domain
 x = 2, 16 Ans.

150. log2x – 3logx = log(x2) – 4

x0 
Sol. Domain :   x  0
x 2  0

log2x – 3logx = 2logx – 4


 log2x – 5logx + 4 = 0
 logx = 1, 4 (x = 10, 104) Domain
 10, 104 Ans.

151. log1/3 x  3 log1/3 x  2  0

x0  x  0
Sol. Domain :    x  (0, 1]
log1 / 3 x  0 x  1 

log1/3x – 3 log1 / 3 x  2  0

Put log1/3x = t2
 t2 – 3 |t| + 2 = 0
 |t|2 – 3|t| + 2 = 0
 |t| = 1, 2
 log1/3x = t2 = 1, 4

 1 1
  x  ,   Domain
 3 81 
Maths IIT-JEE ‘Best Approach’ (MCSIR) Logarithm

1 1
 x , Ans.
3 81

2
152. 
2 log x 5   3log x 5  1  0

x  0
Sol. Domain :    x  (0, )  {1}
x  1

  
2

2 log x 5  3 log x 5  1 = 0 ; Put log x 5 = t
 2t2 – 3t + 1 = 0

1
 t = 1,
2

1
 log x 5  1 ,  5 = x or 5 = x1/2
2


 x  5 , 5  Domain 
 x  5 , 5 Ans.

153. log 22 x  2log 2 x  2  0

x  0
Sol. Domain :    x  (0, )
x  0

log222x + 2log2 x – 2 = 0
 (log2x)2 + log2x – 2 = 0 ; Put log2x = t
 t2 + t – 2 = 0
 (t + 2) (t – 1) = 0
 t = – 2, 1 log2x = – 2 , 1
 1 
  x  , 2  Domain
 4 
1
 x , 2 Ans.
4

log b x 2
154. a   5x logb a  6  0

Sol. (a log b x ) 2  5x log b a  6 ; x > 0, a > 0 , b > 0


a1 , b1

 (a log b x ) 2  5(a log b x )  6  0

 a log b x  2, 3  logbx = loga2 , loga3


Maths IIT-JEE ‘Best Approach’ (MCSIR) Logarithm

 x = b log a 2 , b log a 3

 x  2log a b , 3log a b

1
155. log2 (100x) + log2 (10x) = 14 + log  
x


100 x  0

Sol. Domain : 10 x  0   x > 0
1 
0 
x 

log2 (100x) + log2(10x) = 14 + log(1/x)


 (2 + logx)2 + (1 + logx)2 = 14 – logx ; Put logx =t
 (2 + t)2 + (1 + t)2 = 14 – t
 2t2 + 7t – 9 = 0

9
 t=– ,1
2

9
 log x = – , 1 (x = 10–9/2 , 10)  Domain
2
 x = 10–9/2 , 10 Ans.

156. log4 (x +3) – log4 (x – 1) = 2 – log48

x  3  0
Sol. Domain :    x 1
x 1  0 

log4(x + 3) – log4(x – 1) = 2 – log48

 x 3 3 1
 log4   = 2 
 x 1  2 2

x3
 = 41/2 = 2  x + 3 = 2x – 2
x 1
 (x = 5)  Domain
 x = 5 Ans.

157. log4(x2 – 1) – log4(x – 1)2 = log 4 (4  x) 2


Maths IIT-JEE ‘Best Approach’ (MCSIR) Logarithm

x 2 1  0 

Sol. Domain ( x  1) 2  0    x (–, – 1)  (1, ) – {4}

(4  x ) 2  0

log4(x2 – 1) – log4(x – 1)2 = log4 |4 – x|

 x 2 1 
 log4  
2  = log4 |4 – x|
 ( x  1) 

x 1
 = |4 – x| = |x – 4|
x 1
Case (i) x – 4 > 0 x > 4

x 1
 = x – 4  x2 – 6x + 3 = 0
x 1

 x=3+ 6,3– 6 (x > 4)


Case(ii) x–4<0  x<4

x 1
 = – (x – 4)
x 1
 x2 – 4x + 5 = 0 (Reject)
 x=3+ 6 Ans.

x 3 x 3
158. 2 log 3  1  log 3
x7 x 1

x 3 
 0
x7   x  (, 3)  (7, )
Sol. Domain :      x  1 x  7
x 3  x  (–  , 1)  (3,  ) 
 0 
x 1 

 x 3  x 3
2 log 3    1  log 3  
 x 7  x 1 
Maths IIT-JEE ‘Best Approach’ (MCSIR) Logarithm

  x  3 2 
  
 x 7  ( x  3) 2 ( x  1) 1
 log3   1   
 x 3  ( x  7) 2 ( x  3) 3
  
  x  1  
 

 3(x – 3) (x – 1) = (x – 7)2
 x2 + x – 20 = 0
 (x = – 5, 4) Domain
 x = – 5 Ans.

159. 2log4(4 – x) = 4 – log2(–2 – x)

4x  0 
Sol. Domain :   x  2
 2  x  0

2log4(4 – x) = 4 – log2(–2 – x)
 log2(4 – x) + log2(–2 – x) = 4
 log2 [(4 – x) (–2 – x)] = 4
 (x – 4) (x + 2) = 24 = 16
 x2 – 2x – 24 = 0
 (x = 6, –4)  Domain
 x = – 4 Ans.

160. 3 + 2logx + 13 = 2log3(x + 1)

x  1  0
Sol. Domain :    x  (1, )  {0}
x 1  1 

3 + 2logx+13 = 2log3(x + 1)

2
 3 + log ( x  1) + 2log3(x + 1) ; Put log3 (x + 1) = t
3

2
 3  2 t  2t2 – 3t + 2 = 0
t

1
t = 2 , –
2

1
 log3(x + 1) = 2, –
2
 x + 1 = 32 , 3–1/2

 1 
  x  8,  1    Domain
 3
Maths IIT-JEE ‘Best Approach’ (MCSIR) Logarithm

1
 x = 8, – 1 + Ans.
3

161. logx (9x2). log32 x = 4

9 x 2  0

Sol. Domain : x  0    x  (0, )  {x}
x  1 

logx(9x2) × log32x = 4
 (2logx3 + 2) × (log3x)2= 4 ; Put log3x = t

2 
   2  t2 = 4  t2 + t – 2 = 0
t 
 t = – 2, 1
 log3x = – 2, 1

 1 
  x  , 3   Domain
 9 

1
 x  , 3 Ans.
9

2  x2 
162. log (4x)  log 2    8
1/2
 8 

4x  0 

Sol. Domain : x 2    x  (0, )
 0
8 

 x2 
(log1/24x)2 + log2  8
 =8

 

 x2 
 (log24x)2 + log2  8  = 8
 
 (2 + log2x)2 + 2log2x – 3 = 8 ; Put log2x = t
 (2 + t)2 + 2t – 11 = 0 t = – 7, 1
 log2x = – 7, 1 (x = 2–7, 2)  Domain
 x = 2–7 , 2 Ans.

163. log 0.5x x 2  14log16x x 3  40log 4x x  0


Maths IIT-JEE ‘Best Approach’ (MCSIR) Logarithm

x2  0 

x3  0 
0.5x  0

16x  0   1 1
Sol. Domain :  x  (0, ) – 2, 16 , 4 
4x  0   
0.5x  1 

16x  1 
4x  1 

log0.5xx2 – 14log16xx3 + 40 log4x x = 0


Change base to 2

log 2 x 2 log 2 x 3 log 2 x1 / 2


  14  40 =0
x log 2 16 x log 2 4 x
log 2
2

2 log 2 x log 2 x 40 log 2 x  0


 log x  1 – 42 4  log x +
2 2 2 2  log 2 x

Put log2x = t

2t 42t t
 –  20 0
t 1 4  t t2

1 21 10
 t = 0 or   0
t 1 4  t t  2
 t = 0 or (4 + t) (t + 2) – 21 (t – 1) (t + 2) + 10(t – 1) (t + 4) = 0
 t = 0 or 2t2 – 3t – 2 = 0

1
 t = 0 or t = 2, –
2

1
 log2x = 0 , 2, –
2
 1 
  x  1, 4,   Domain
 2

1
 x = 1, 4, Ans.
2

4 2log 3 3
164. 6  (1  4  9 )  log 7 x  log x 7 , x  Q
Maths IIT-JEE ‘Best Approach’ (MCSIR) Logarithm

Sol. Domain :
x0
x 1 
  x  (0, )  {1} & x  Q (Given)

6 – 1  4  9   lo g
4  2 log 3
3
7 x  log x 7

1
 6 – (1 + 4  94–4) × log7x = log x ; Put log7x = t
7

1
 6–5t=  5t2 – 6t + 1 = 0  t = 1, 1/5
t

1
log7x = 1, (x = 7, 71/5)  Domain
5
 x=7

165. log3(4.3x– 1) = 2x + 1
Sol. log3(4  3x – 1) = 2x+ 1 ......(1)
 4  3x – 1 = 32x+1 = 3  32x
Put 3x = t 3t2 – 4t + 1 = 0
 t = 1/3, 1

1
 3x = , 1  x = – 1, 0 , Put in (1)
3
 x = – 1, 0 satisfy eqn (1)
 x = – 1, 0 Ans.

166. log3(3x – 6) = x – 1
Sol. log3(3x – 6) = x – 1 .....(1)

3x
 3x – 6 = 3x –1 =
3

2
 . 3x = 6
3
 3x = 9  x = 2 ; Put x = 2
 x = 2 satisfy eqn (1)
 x = 2 Ans.

167. log3(4x – 3) + log3(4x – 1) = 1


Sol. log3(4x – 3) + log3(4x – 1) = 1 .....(1)
x x
 log3[(4 – 3) (4 – 1)] = 1 ; Put 4 = t x

 (t – 3) (t – 1) = 3 t2 – 4t = 0
 t = 0, 4
 4x = 0, 4  x = 1 ; Put in (1)
 x = 1 satisfy eqn (1) ,  x = 1 Ans.
Maths IIT-JEE ‘Best Approach’ (MCSIR) Logarithm

168. log 3  log1/2 2 x  3log1/2 x  5   2

Sol. log3(log1/22x – 3log1/2x + 5) = 2 .....(1)


2 2
 (log2x) + 3log2x + 5 = 3 = 9 ; Put log2x = t
 t2 + 3t – 4 = 0
 t = 1, – 4
 1
 log2x = 1, – 4   x  2,  ; Put in (1)
 16 
1
 x = 2, satisfy eqn (1)
16

1
 x = 2, Ans.
16

 2x   2 
169. log 5    log 5  
 10   x 1

2x 
 0
   x  2
Sol. Domain : 10   x > – 1
2  x  1
 0 
x 1 

2x   2 
log5   = log5  
 10   x 1 

2 x 2
 =  (x + 1) (x + 2) = 2 = 0
10 x 1
 x2 + 3x – 18 = 0
 (x = – 6, 3) Domain
 x = 3 Ans.

170. 1+ 2 log(x +2)5 = log5(x +2)

x  2  0
Sol. Domain :   x (–2, ) – {–1}
x  2  1

1 + 2 log(x+2)5 = log5(x + 2)
Maths IIT-JEE ‘Best Approach’ (MCSIR) Logarithm

2
 1 + log ( x  2) = log5(x + 2) Put log5(x + 2) = t
5

2
 1+ =t  t2 – t – 2 = 0
t
 t = 2, – 1
 log5(x + 2) = 2, – 1 (x = 23, –9/5) Domain
 x = 23, –9/5 Ans.

171. log 4 24x  2log2 4

Sol. log4(24x) = 2 log 2 4

4x
 =4  x=2 Ans.
2

x 15
172. log 2   
 4  log x  1
2
8

x 
0 
8 
x  x0 
Sol. Domain :  0     x (0, ) – {16}
4   x  16
x 
log 2    1  0
8 

x 15
log2   = log ( x / 8)  1
4 2

15
 log2x – 2 = log x  3  1 ; Put log2x = t
2

15
 t–2=  t2 – 6t – 7  t = 7, –1
t4
 log2x = 7, – 1 (x = 27, 2–1) Domain
 x = 27 , 1/2 Ans.

1  2(log x) 2
173. 1
log x  2(log x) 2
Maths IIT-JEE ‘Best Approach’ (MCSIR) Logarithm

1  2(2 log x) 2
Sol. 1 .....(1)
log x  2(log x) 2

Put log x = t

1  8t2
 1
t  2t2

 6t 2  t  1  0
 t = 1/3 , – 1/2

1 1
 logx = ,  x = 101/3 , 10–1/2 ; Put in (1)
3 2
 x = 101/3 , 10–1/2 Satisfy eqn (1)
 x = 101/3 , 10–1/2

174. log2(4.3x –6) – log2(9x – 6) = 1


Sol. log2(4  3x – 6) – log2(9x – 6) = 1 ....(1)

 4  3x  6 
 log2  x  1
 9 6 

4  3x  6
 2  Put 3x  t
9x  6
 4t – 6 = 2 (t2 – 6)
 t2 – 2t – 3 = 0
 t=3,–1
 3x = 3, – 1  x = 1 ; Put in (1)
 x = 1 satisfy eqn (1) ,  x = 1 Ans.

1
175. log(5x  4)  log x  1  2  log 0.18
2

5x  4  0 4
Sol. Domain :   x 
x 1  0  5

1
log(5x – 4) + log x  1 = 2 + log 0.18
2

1 1
 log (5x – 4) + log (x + 1) = log (100 × 0.18)
2 2
 log (5x – 4) + log (x + 1) = 2 log(18)
 (5x – 4) (x + 1) = (18)2
 5x2 + x – 328 = 0
Maths IIT-JEE ‘Best Approach’ (MCSIR) Logarithm

1  81  82 80 
 x=– =  ,  Domain
25  10 10 
x 

176. log4(2.4x–2 –1) + 4 =2x


Sol. log4(2  4x–2 – 1) + 4 = 2x ....(1)
 2  4x–2 – 1 = 4(2x–4)

2  4x 42x
 1 = ; Put 4x = t
16 256
 t2 – 32t + 256 = 0
 (t – 16)2 = 0 t = 16
 4x = 16 x = 2 ; Put in eqn (1)
 x = 2 satisfy eqn (1)
x = 2 Ans.

177. log x 5  log x (5x)  2.25  (log x 5) 2

x0 

5x  0
Sol. Domain :  x – {1}
x  1 

logx 5 + logx(5x) – 2.25 = (logx 5 )2

2
1 1 
 logx5 + logx5 + 1 – 2.25 =  log x 5 
2 2 

Put logx5 = t

t 1
 + t – 1.25 = t2
2 4
 t2 – 6t + 5 = 0 t = 1, 5
 logx5 = 1, 5 5 = x , x5
 (x = 5, 5 1/5) Domain
 x = 5, 51/5 Ans.

178. log(log x) + log(log x4– 3) = 0


Sol. log(logx) + log(logx4 – 3) = 0 .....(1)
4
 log[(logx)·(logx – 3)] = 0
 (log x) (4logx –3) = 1 ; Put log x = t
Maths IIT-JEE ‘Best Approach’ (MCSIR) Logarithm
 t(4t – 3) = 1

1
 4t2 – 3t – 1 = 0 t = – , 1
4

1
 log x = – , 1
4
 x = 10–1/4 , 10 ; Put in eqn (1)
 x = 10, 10–1/4 Ans.

179. log3x – 2 log1/3x = 6


Sol. Domain : x > 0
log3x – 2 log1/3x = 6 log3x + 2 log3x = 6
 log3x = 2 (x = 9) Domain
 x = 9 Ans.

2 log x
180. 1
log(5x  4)

2 log x
Sol. 1 ......(1)
log(5x  4)

 2logx = log(5x – 4)
 logx2 = log(5x – 4)
 x2 – 5x + 4 = 0
 x = 1, 4 ; Put in (1)
 x = 4 Ans.

4
181. 2 log8 (2x)  log8 (x 2  1  2x) 
3

2x  0  x0 
Sol. Domain :    x (0, ) – {1}
x 2  1  2 x  0 ( x  1) 2  0

4
2log8(2x) + log8(x – 1)2 =
3

4
 log8((2x)2 (x – 1)2) =
3
 (2x(x – 1))2 = 84/3 = 16
 2x(x – 1) = 4 or 2x(x – 1) = – 4
 x2 – x – 2 = 0 or x2 – x + 2 = 0
 (x = 2, – 1) Domain
 x = 2 Ans.
Maths IIT-JEE ‘Best Approach’ (MCSIR) Logarithm

1 1
182. log 2 (x  2)   log1/8 3x  5
6 3
x20 
Sol. Domain :   x > 2
3x  5  0

1 1
log2(x – 2) – = log1/8 3x  5
6 3

1 1 1
 log2(x – 2) – = – log2 (3x – 5)
6 3 6
 log2(x – 2) – 2 = – log2 (3x – 5)
 log2[(x – 2) . (3x – 5)] = 2
 (x – 2)(3x – 5) = 4
 3x2 – 11x + 6 = 0

 2 
  x  , 3  Domain
 3 

 x = 3 Ans.

183. 2 log3(x – 2) + log3(x – 4)2 = 0

x20 
Sol. Domain :   x (2, ) – {4}
2
( x  4)  0 

2log3(x – 2) + 2 × log3 |x – 4| = 0
 (x – 2) |x – 4| = 1
Case (i) x – 4 > 0 x > 4
 (x – 2) (x – 4) = 1
 x2 – 6x + 7 = 0

 x=3+ 2 , 3– 2
Case (ii) x < 4
 –(x – 2) (x – 4) = 1
 x2 – 6x + 9 = 0 x = 3

 x = 3, 3 + 2 Ans.

184. log 2 (2x 2 )  log 4 (16x )  log 4 x 3

Sol. log 2 ( 2 x 2 ). log 4 (16 x ) = log4x3 ....(1)


Maths IIT-JEE ‘Best Approach’ (MCSIR) Logarithm

2
1 3 
Square , (1 + 2log2x) . (4 + log2x) =  log 2 x 
2 2 

Put log2x = t

9 2
 (1 + 2t) (4 + t) = t
2
 5t2 – 18t – 8 = 0

2
 t = 4, –
5

2
 log2x = 4, – x = 16, (2)–2/5 ; Put in (1)
5
 x = 16 Ans.

3log x  19
185.  2log x  1
3log x  1

3 log x  19
Sol. = 2 logx + 1 .....(1)
3 log x  1

Put log x = t
 3t + 19 = (3t – 1) (2t + 1)
 3t2 – t – 10 = 0

5
 t = 2, –
3

5
 log x = 2, –
3
 x = 102, 10–5/3 ; Put in eqn (1)
 x = 100 , 10–5/3 Ans.

186.
log  x 1 1  3
3
log x  40

log( x  1  1)
Sol. 3 ....(1)
log(3 x  40 )

 log ( x  1  1) = log(x – 40)

 x  1 + 1 = x – 40

 x  1 = x – 41
Square  x + 1 (x – 41)2
Maths IIT-JEE ‘Best Approach’ (MCSIR) Logarithm

Logarithm Solved Exercise-II


1. Which is the correct order for a given number d, d > 1
(A) log2 d < log3 d < loge d < log8 d
(B) log8 d < log3 d < loge d < log2 d
(C) log8 d < loge d < log2 d < log3 d
(D) log3 d < loge d < log2 d < log8 d
Sol.  8 > 3 > e > 2 & d > 1
 logd8 > logd3 > logde > logd2
1 1 1 1
   
log8 d log3 d log e d log 2 d
 log8d < log3d < loged < log2d

2. If log 27 = 1.431 then the value of log 9 is


1.431
Sol. log27 = 1.431 log3 = = 0.477
3
 log9 = 2 log3 = 2 × 0.477 = 0.954

3. If x = log2a a, y = log3a 2a, z = log4a 3a, then the value of x y z + 1 is


(A) yz (B) 2yz (C) y + z (D) y – z
Sol. x = log2aa , y = log3a2a , z = log4a3a
 xyz = log2aa . log3a2a . log4a3a = log4aa
 xyz + 1 = log4aa + 1 = log4aa + log4a4a
= log4a4a2 = log4a(2a)2 = 2 log4a2a
xyz + 1 = 2yz {yz = log3a2a . log4a3a = log4a2a}

4. Let 3a = 4, 4b = 5, 5c = 6, 6d = 7, 7e = 8, and 8f = 9, then find the value of the product abcdef :


Sol. 3a = 4, 4b = 5, 5c = 6 , 6d = 7, 7e = 8 and 8f = 9
 a = log34 , b = log45 , c = log56 , d = log67 , e = log78 & f = log89
 abcdef = log39 = 2

5. If log5 a . loga x = 2, then x is equal to :


Sol. log5a . logax = 2
change base to "e"
na nx nx
 × = 2  =2
n5 na n5
 log5x = 2 x = 25

81 25 16
6. Value of 3 log + 5 log + 7 log is
80 24 15
81 25 16
Sol. 3.log + 5. log + 7.log
80 24 15
101
Maths IIT-JEE ‘Best Approach’ (MCSIR) Logarithm

 81 3  25 5  16 7 
= log   .   .    {Prime factorize}
 80   24   15  
 312 510 228 
= log  12 3  15 5  7 7 
 2 .5 2 .3 3 .5 
= log 2 Ans.

7. f(x) = log10 x 2 . The set of all values of x for which f(x) is real is :
Sol. f(x) = log10 x 2
x2  0   x  R  {0}
Domain :    
log10 x  0 
2
 x2  1 

 x  R  {0}
 x  1 or x  1 
 x (–, – 1] [1, ) Ans.

log alog ylog y z


8. Value of x x a :
log a.log y.log z
Sol. x x a y
change base to "e"
na ny nz nz
. .
 x nx na ny
x nx

 x log x z
 z Ans.

9. If log2 x + log2 y  6, then the least value of x + y is :


Sol. log2x + log2y  6 ; x, y > 0
 log2 (xy)  6 xy 26 xy 64
 x & y are positive
 x + y = ( x )2  ( y)2 = ( x  y) 2 + 2 xy
0

 x + y  2 xy  2 × 8  x + y 16
 least value of x + y = 16 Ans.

10. A rational number which is 50 times its own logarithim to the base 10 is :
Sol. Let the rational no. be x
 According to the conditions,
x = 50 × log10x
x
 log10x =
50
 By hit & trial, x = 100

102
Maths IIT-JEE ‘Best Approach’ (MCSIR) Logarithm
11. If x = log5 (1000) and y = log7 (2058) then which is greater ?
Sol. x = log5(1000) and y = log7(2058)
 54 = 625  73 = 343
55 = 3125 74 = 2401
 x     y (3, 4)

12. If 4log9 3  9log2 4  10logx 83 then x is equal to :


Sol. 4log9 3  9log2 4  10logx 83
 2log3 3  92log2 2  10logx 83
 2 + 81 = 10logx 83
 83 = 83logx 10  logx10 = 1  x = 10

ab na  nb a b
13. If n   then  is equal to :
 3  2 b a
ab na  nb
Sol. n 
 3  2
ab
 2n    n(ab)
 3 
ab
2

    ab  a2 + b2 + 2ab = 9ab
 3 
 a2 + b2 = 7ab
a b
  7
b a

14. If log 2 = 0.30103 the no. of digits in 264 is :


Sol. log2 = 0.30103 (Given)
Let y = 264 ; Take “log” both sides
 log(y) = 64n2 = 64 × 0.30103
 log(y) = 19.26592
 Characteristic = 19
 Characteristic = (No. of digits) – 1
 19 = No. of digits – 1
 No. of digits = 20

103
Maths IIT-JEE ‘Best Approach’ (MCSIR) Logarithm
15. The number of zeroes coming immediately after the decimal point in the value of (0.2)25 is :
(Given log10 2 : 0.30103)
Sol. log102 = 0.30103
Let y = (0.2)25 ; Take “log” both sides
 log(y) = 25 × log(0.2) = 25 × [log2 – log10]
= – 17.47425
Characteristic = – 18
Characteristic = – [No. of zeroes immediately after decimal + 1]
 – 18 = – (No. of zeroes + 1)
 No. of zeroes = 17

16. Simplify : 7log3 5  3log5 7  5log3 7  7log5 3
a loge b .(log a 2 b).(log b2 a)
Sol. 6
eloge a ·log e b
1 1
a loge b    log a b.log b a
= 6× 2 2
(eloge a )loge b
6 a loge b 3
=  
4 a loge b 2

17. The solution set of log2 |4 – 5x| > 2 is :


Sol. log2|4 – 5x| > 2 ; Domain : |4 – 5x| > 0
4
x  R –  
5
 |4 – 5x| > 4
 4 – 5x > 4 or 4 – 5x < – 4
8
 (x < 0 or x  )  Domain
5
8 
 x  (–, 0)   ,   Ans.
5 

1
18. If  log0.1 x  2 then 'x' belongs to :
2
Sol. 1  log0.1x  2 ; Domain : x > 0
 (0.1)1  x  (0.1)2 { 0 < Base < 1}
 1 1
  x   Domain
 100 10 
 1 1
 x  ,
100 10 

104
Maths IIT-JEE ‘Best Approach’ (MCSIR) Logarithm

19. The solution set of log11 log7  


x  5  x = 0 is :

x 5 0 

Sol. Domain : x  0    x0

x  5  x  0

 
log11 log7 x 5  x   0
 log7 ( x  5  x ) = 1
 x 5  x  7
 x 5  7 – x
Square ,  x + 5= (7 – x )2
xx – 14 x
 484 
  x   Domain
 49 
484
x = Ans.
49

20. Solve for 'x' in the equation : ln(x – 3) + ln(x – 2) = ln(2x + 24) :
x 3  0

Sol. Domian : x  2  0    x > 3
2x  24  0 

n(x – 3) + n(x – 2) = n (2x + 24)
 (x – 3) (x – 2) = 2x + 24
 x2 – 7x – 18 = 0
 (x = 9, – 2)  Domain
 x = 9 Ans.

105
Maths IIT-JEE ‘Best Approach’ (MCSIR) Logarithm

Logarithm Solved Exercise-III


 ab  (ab) 2  4(a  b)   ab  (ab) 2  4(a  b) 
1. Let A denotes the value of log10   + log10  
 2   2 
   
when a = 43 and b = 57
and B denotes the value of the expression  2log 18 · 3log 3  . 6 6

Find the value of (A · B).


 ab  (ab)2 – 4(a  b)   ab – (ab) 2 – 4(a  b) 
Sol. A = log10   + log10  
 2   2 
   
 (ab)  ((ab)  4(a  b)) 
2 2
 A = log10  
 4 
= log10(a + b) = log10100 = 2 { a = 43 ; b = 57}

B = 2log6 18 ·3log 6 3  2log 6 (6 3) ·3log 63


= 21log6 3 ·3log6 3
= 2 · 2log6 3 ·3log6 3  2  (2·3)log6 3
= 2 × (6)log6 3  2  3  6
 A × B = 2 × 6 = 12

2. (a) If x = log34 and y = log53, find the value of log310 and log3(1.2) in terms of x and y.
2
(b) If k log2 5 = 16, find the value of k (log2 5) .
Sol. (a) x = log34 = 2log32 , y = log53 (given)
x 1
 log310 = log3(2 × 5) = log32 + log35 = 
2 y
xy  2
log310 = Ans.
2y
&
12
log31.2 = log3 = log312 – log310 = log3(22 × 3) – log310
10
= 2log32 + 1 – log310
 xy  2 
=x+1–  
 2y 
xy  2y – 2
 log31.2 = Ans.
2y
(b) k log2 5  16 (Given)
k (log2 5)  (k log2 5 )log2 5  16log2 5  24log2 5
2

= 54 = 625

Solve for x (3 to 5)
3. (a) If log10 (x2  12x + 36) = 2
(b) 91+logx  31+logx  210 = 0 ; where base of log is 3.

106
Maths IIT-JEE ‘Best Approach’ (MCSIR) Logarithm
Sol. (a) Domain : x2 – 12x + 36 > 0
 (x – 6)2 > 0  x  R – {6}
Log10(x2 – 12x + 36) = 2  x2 – 12x + 36 = 100
 x2 – 12x – 64 = 0
 (x = 16, – 4)  Domain
 x = 16 , –4 Ans.

(b) Domain : x > 0


91log x – 31log x – 210  0
 9·9logx – 3·3logx – 210 = 0
 9·32logx – 3.3logx – 210 = 0 ; Put 3logx = t
 9t2 – 3t – 210 = 0
14
 t=– , 5
3
14
 3 log3 x
 , 5 x 5
3
log b  log b N 
4 3 1  4/3 log b a
4. Simplify : (a) log1/3 729 . 9 . 27 ; (b) a
Sol. (a) log1/3 4 729. 3 9–1.27 –4/3
= – log3(36·(3–2·3–4)1/3)1/4
= – log3(36·3–2)1/4 = – log3(34)1/4
= – 1 Ans.
 log b (log b N) 
 
 
 a loga (logb N)
log b a
(b) a
 log b N

5. (a) If log4 log3 log2 x = 0 ; (b) If loge log5 [ 2x  2  3] = 0


Sol. (a) log4(log3(log2x)) = 0 …..(1)
 log3(log2x) = 1  log2x = 3
 x = 23 = 8 ; Put in eqn (1)
 x=8
(b) 
loge log5 ( 2x  2  3)  0  …..(1)
 log5 ( 2x  2  3)  1
 2x  2  3  5
 2x – 2 = 4  x = 3 ; put in eqn (1)
 x=3

6. (a) Which is smaller? 2 or (log2 + log2 ).


(b) Prove that log35 and log27 are both irrational.
Sol. (a) If x > 0
1 1
x + = ( x )2 +
x ( x )2
2
1  1  1
x+ =  x  2  x+ 2 & Equality holds when x = 1
x  x x
0

107
Maths IIT-JEE ‘Best Approach’ (MCSIR) Logarithm
1
 log2 + log2 = log2 + 2
log  2

(b) Assume that log35 is a rational no.


p
 log35 = ; p, q  I & q  0
q
 5 = 3p/q
 5q = 3p
 Both 5 and 3 are prime nos.
 For no. integral values of p & q.
5q can be equal to 3p
 Our assumption is wrong
 Log35 is an irrational number hence proved
Similarly we can prove that log27 is also an irrational number.

7. Let a and b be real numbers greater than 1 for which there exists a positive real number c,
different
from 1, such that
2(logac + logbc) = 9logabc. Find the largest possible value of logab.
Sol. a > 1, b > 1 , c > 0 (c  1)
2(logac + logbc) = 9logabc
Change base to ‘a’
 log a c  log a c
 2 log a c   =9×
 log a b  log a ab
 1  1
 2 1    9 ; Put logab = t
 log a b  1  log a b
 1 9
 2 1   =  2(t + 1)2 = 9t
 t  1  t
1
 2t – 5t + 2 = 0 
2
t = 2,
2
1
 logab = 2 or logab =
2
 largest value of logab = 2

8. Find the square of the sum of the roots of the equation


log3x · log4x · log5x = log3x · log4x + log4x · log5x + log5x · log3x.
Sol. log3x·log4x·log5x = log3x·log4x + log4x·log5x + log5x·log3x
Case (i) x = 1
 LHS = RHS = 1
 x = 1 is a solution
Case(ii) x  1
Divide by log3x·log4x·log5x both sides.
1 1 1
 1=  
log5 x log3 x log 4 x
 1 = logx5 + logx3 + logx4
 1 = logx(60)  x = 60
 x = 1, 60

108
Maths IIT-JEE ‘Best Approach’ (MCSIR) Logarithm

2 3
9. Find the value of the expression  .
log 4 (2000) log5 (2000)6
6

Sol. 2  log(2000)6 4  3  log(2000)6 5


1
= log (2000)6 [42  53 ]  log 2000 2000
6
1
=
6
5log4 2 3  6   6log8  3  2 
10. Calculate : 4
5log4 (3 6 ) 6log8 ( 3  2 )
Sol. 4 2

5log (3 6 ) 6log 3 ( 3  2 )


25/2
=4 2

2 6
5· log2 (3 6 )  log 2 ( 3  2 )
5 3
=4
  3 6  
2 log 2    ; Rationalise
  3  2  
=4
= 42log2 ( 3)
 24log2 3

= ( 3)4
=9
1 3

3
.    
log 9 log 6 3
81 2

 125
5
log25 7 log 25 6
11. Simplify : 7 
409  

81log9 5  33.log3 6 
Sol.  ( 7)2.log7 25  (125)log25 6 
409
92.log9 5  33log3 6  log7 25 3log52 6 
=  7 5
409  
25  ( 6) 3
=  (25  (6)3/2 )
409
(25)2  63 625  216
=  1
409 409

log1/5  12 4 1
12. Simplify : 5  log log1/2 .
7 3 2
10  2 21
log (2) –1  4   1 
Sol. 5 5–1  log 21/2    log 2–1  
 7 3  10  2 21 
 4   1 
= 5log5 2  2log 2    log  
 7 3  10  2 21 
= 2 + 2log2 ( 7  3)  log 2 ( 7  3)2
= 2 + 2 [log 2 ( 7  3)  log 2 ( 7  3)]
= 2 + 2 × log3(7 – 3) = 2 + 2 × log24
=2+2×2
=6

109
Maths IIT-JEE ‘Best Approach’ (MCSIR) Logarithm
2
2
13. Find 'x' satisfying the equation 4log10 x 1 – 6log10 x – 2.3log10 x = 0.
x0 
Sol. Domain : 2   x  0
x  0
4log10 x 1  6log10 x  2·3log10 x 2  0
2

 4log10 x ·4  6log10 x – 2·3log10 x ·32  0


2

 4·22log10 x  (2·3)log10 x  18·32log10 x  0


 4·22log10 x  2log10 x ·3log10 x –18·32log10 x  0
Put 2log10 x  p and 3log10 x  q
 4p2 – pq – 18q2 = 0
 (p + 2q) (4p – 9q) = 0
 p = – 2q or 4p = 9q
 2log10 x  2  3log10 x (Reject)
Or
4 2 log10 x
 9  3log10 x
log x 2
2 9 2
10

     
3 4 3
 log10x = – 2  (x = 100–2)  Domain
1
 x=
100
2 a 2 b5
14. Given that log2a = s, log4b = s2 and log c2 (8) = . Write log as a function of 's'
s3  1
2
c4
(a, b, c > 0, c  1).
Sol. log2a = s , log4b = s2  log2b = 2s2
2 3 2 4
and log c2 8  3  log c 2  3  log c 2  3
s 1 2 s 1 3(s  1)
a b 
2 5
 log2  4  = 2log2a + 5log2b – 4log2c
 c 
3
= 2s + 10s2 – 4 × (s3 + 1)
4
= 2s + 10s2 – 3(s3 + 1)

15. Find the value of 491log7 2 + 5 log5 4 .


Sol. 49(1log7 2)  5 log5 4
–1 –2 –1
5log5 ( 4)
= 49  7 –2log7 2  5log5 (4)  49  7log7 (2)
49 1 50
=    12.5
4 4 4

16. Given that log2 3 = a , log3 5 = b, log7 2 = c, express the logarithm of the number 63 to
the base 140 in terms of a, b & c.
Sol. log23 = a , log35 = b , log72 = c
  
3 = 2a 5 = 3b 2 = 7c
 5 = 2ab
 log14063 = log(7×20)(9×7) = log (22 57) (32  7)

110
Maths IIT-JEE ‘Best Approach’ (MCSIR) Logarithm

1
2a 
= log(22 ·2ab ·21/c ) (22a ·21/c ) = c
1
2  ab 
c
2ac  1
=
abc  2c  1

log 2 24 log 2 192


17. Prove that  = 3.
log96 2 log12 2
log 2 24 log 2 192
Sol. LHS = 
log96 2 log12 2
= log224 × log296 – log2192 × log212
= log2(23·3) × log2(25.3) – log2(26·3) × log2(22·3)
= (3 + log23) × (5 + log23) – (6 + log23) × (2 + log23)
Let log23 = t
= (3 + t) (5 + t) – (6 + t)(2 + t)
= (15 +8t + t2) – (12 + 8t + t2)
= 3 = RHS Ans.

18. Prove that ax – by = 0 where x = log a b & y = log b a , a > 0 , b > 0 & a , b  1.
1
Sol. x= log a b & y = log b a =
log a b
LHS = ax – by
1

b
loga b log a b
=a
Put logab = t  b = at
1 1

= a t b t
 a t  (a t ) t

 a t  a t  0  RHS
Hence proved.

19. If a, b, c are positive real numbers such that a log3 7 = 27 ; blog7 11 = 49 and clog11 25 = 11 . Find
 2
the value of a (log3 7)  b(log7 11)  c(log11 25)
2 2

.
Sol. a log3 7  27, blog7 11  49, clog11 25  11
 a (log3 7)  b(log7 11)  c(log11 25)
2 2 2

= (a log3 7 )log3 7  (blog7 11 )log7 11  (clog11 25 )log11 25


= 27log3 7  49log7 11  ( 11)log11 25
1
log11 25
= 33log3 7  72log7 11  112
= (3)3 + (7)2 + (25)1/2
= 469 Ans.

log10 (x  3) 1
20. (a) Solve for x , 
log10  x  21 2
2

(b) log (log x) + log (log x3  2) = 0 ; where base of log is 10 everywhere.


(c) logx2 . log2x2 = log4x2 (d) 5logx + 5 xlog5 = 3 (a > 0) ; where base of log is a.

111
Maths IIT-JEE ‘Best Approach’ (MCSIR) Logarithm
log10 (x  3) 1
Sol. (a)  …….(1)
log10 (x  21) 2
2

 2log10 (x – 3) = log10(x2 – 21)


 (x – 3)2 = x2 – 21
 x = 5 ; Put in (1)
 x = 5 Ans.

log(logx) + log(logx3 – 2) = 0
(b) …….(1)
 log((logx) × (logx3 – 2)) = 0
 logx × (3logx – 2) = 1 ; Put log x = t
 3t2 – 2t – 1 = 0
1
t=– , 1
3
1
 logx = – , 1  (x = 10–1/3, 10) ; Put in (1)
3
 x = 10 Ans.

(c) logx2 · log2x2 = log4x2


1 1
Domain : x > 0 and x  ,
2 4
1 1 1
  
log 2 x log 2 2x log 2 4x
1 1 1
   ; Put log2x = t
log 2 x 1  log 2 x 2  log 2 x
1 1
   t2  2
t(1  t) 2  t
 t=± 2
 log2x = ± 2  (x  2 2 , 2 2 )  Domain
 x  2 2 , 2 2 Ans.
(d) Let log a x  z
5z  5.5z  3
Let 5  p
z

p(6)  3
1
p
2
1
5z 
2
z log5   log 2
log 2
z
log 5
log a x   log5 2
x  a  log5 2

112
Maths IIT-JEE ‘Best Approach’ (MCSIR) Logarithm
10 xy
21. If x, y > 0, logyx + logxy = and xy = 144, then = N where N is a natural number,
3 2
find the value of N.
x  0
y  0 
Sol. Domain :  
x 1
y  1 
10
logyx + logxy = & xy = 144
3
Put logyx = t
1 10
 t+   3t2 – 10t + 3 = 0
t 3
1
 t=3 or t=
3
1
 logyx = 3 or logyx =
3
 x = y3 or x = y1/3
 xy = 144  xy = 144
 y = 144
4
 y4/3 = 144
1/2
y = (12) y = (12)3/2
 x = (12) 3/2
 x = (12)1/2
x  y (12)1/2  (12)3/2
 
2 2
1/2
(12)
= (1  12)
2
13
= 12  507
2
 N = 507 Ans.

22. Solve the system of equations:


log a x log a (xyz)  48
log a y log a (xyz)  12 , a > 0, a  1.
log a z log a (xyz)  84
Sol. logax · loga(xyz) = 48 …..(1)
logay · loga(xyz) = 12 …..(2)
logaz · loga(xyz) = 84 ..…(3)
(1) + (2) + (3)
 loga(xyz) [logax + logay + logaz] = 144
 loga(xyz) × loga(xyz) = 144
 loga(xyz) = ± 12
 from eqn (1), (2) and (3)
logax = ± 4, logay = ± 1 , logaz = ± 7
 x = a±4, y = a±1, z = a±7
 x = a4, y = a , z = a7
Or
x = a–4 , y = a–1, z = a–7

113
Maths IIT-JEE ‘Best Approach’ (MCSIR) Logarithm
23. (a) Given : log1034.56 = 1.5386, find log103.456 ; log100.3456 & log100.003456.
(b) Find the number of positive integers which have the characteristic 3, when the base of
the logarithm is 7.
(c) If log102 = 0.3010 & log103 = 0.4771, find the value of log10(2.25).
(d) Find the antilogarithm of 0.75, if the base of the logarithm is 2401.
Sol. (a) log10(34.56) = 1.5386
 Mantissa = 0.5386
 log103.456 = 0.5386
log100.3456 = 1 .5386
log100.003456 = 3 .5386
 If number > 1,
then characteristic = No. of digits – 1
Or
 If 0 < Number < 1
then, characteristic = – (No. of zeroes immediately before decimal + 1)

(b) Characteristic of log7N = 3


 N  [343, 2401)
 No. of +ve integers = 2058 Ans.

(c) log102 = 0.3010 & log103 = 0.4771


225
 log10(2.25) = log10 = log10(15)2 – 2
100
= 2[log103 + log105] – 2
10
= 2[log103 + log10 ] – 2
2
= 2[log103 + 1 – log102] – 2
= 2[0.4771 + 1 – 0.3010] – 2
= 0.3522 Ans.

(d) Antilog of 0.75 to the base 2401 can be written as,


log2401x = 0.75
 x = (2401)3/4 = (74)3/4
x = 343 Ans.

24. If log102 = 0.3010, log103 = 0.4771. Find the number of integers in :


(a) 5200 (b) 615 & (c) the number of zeros after the decimal in 3100.
Sol. log102 = 0.3010 & log103 = 0.4771
(a) let y = 5200
Take log10 both sides
 log10y = 200 × log105 = 200 × log10(10/2)
log10y = 200 × [1 – log102] = 139.8
 Characteristic = No. of digits – 1
 139 = No. of digits – 1
 No. of digits = 140

(b) Let y = 615


take log10 both sides
 log10y = 15log106 = 15 [log102 + log103]
= 11.6715
Characteristic = No. of digits – 1
 11 = No. of digits – 1
 No. of digits = 12
114
Maths IIT-JEE ‘Best Approach’ (MCSIR) Logarithm

(c) Let y = 3–100


Take log10 both sides
 log10y = – 100 log(3) = –100 × 0.4771
log10y = – 47.71
Characteristic = – (No. of zeroes immediately before decimal + 1)
 – 48 = – (No. of zeroes immediately before decimal + 1)
 No. of zeroes immediately before decimal
= 47

25. Let 'L' denotes the antilog of 0.4 to the base 1024.
and 'M' denotes the number of digits in 610 (Given log102 = 0.3010, log103 = 0.4771)
and 'N' denotes the number of positive integers which have the characteristic 2,
when base of the logarithm is 6.
Find the value of LMN.
Sol. log10242 = 0.4  L = (1024)0.4 = (210)0.4 = 16
 L = 16
= Let y = 610 , Take log10 both sides
 log10y = 10 × log106 = 6 × (log102 + log103)
Log10y = 10 × (0.3010 + 0.4771) = 7.781
 Characteristic = No. of digits – 1
 7 = No. of digits – 1
 No. of digits = 8
 M=8
Characteristic of log6x = 2
 x  [36, 216)
 No. of +ve integers = 180
 N = 180
 LMN = 16 × 8 × 180
= 23040 Ans.

115
Maths IIT-JEE ‘Best Approach’ (MCSIR) Logarithm

Logarithm Solved Exercise-IV

Note : From Q.1 to Q.7, solve the equation for x :


1. xlogx+4 = 32, where base of logarithm is 2.
Sol. Domain : x > 0
x log2 x + 4 = 25
Take log2 both sides
 (log2x + 4) × log2x = 5 ; Put log2x = t
 t2 + 4t – 5 = 0  t = – 5, 1
 log2x = – 5, 1
 1 
  x = , 2   Domain
 32 
1
 x= ,2
32

2. logx+1 (x² + x − 6)2 = 4


x +1  0 

Sol. Domain : x + 1  1    x > – 1 – {0, 2}
(x 2 + x – 6) 2  0 
log(x+1)(x2 + x –6)2 = 4
 (x2 + x – 6)2 = (x + 1)4
 x2 + x – 6 = ± (x + 1)2
 x2 + x – 6 = (x + 1)2 or x2 + x – 6 = – (x + 1)2
 x2 + x – 6 = x2 + 2x + 1 or x2 + x – 6 = – x2 – 2x – 1
 x=–7 or 2x2 + 3x – 5 = 0
5
 x = 1, –
2
 5
  x = −7,1, –   Domain
 2
 x = 1 Ans.

3. x + log10(1 + 2x) = x . log105 + log106.


Sol. x + log10(1 + 2x) = x.log105 + log106 …….(1)
 log1010x + log10(1 + 2x) = log10 (5x . 6)
 10x.(1 + 2x) = 6.5x
 5x [2x(1 + 2x) – 6] = 0 ; Put 2x = t
 5x = 0 or t2 + t – 6 = 0  t = – 3 , 2
 2x = – 3, 2
 x = 1 ; Put is eqn (1)
 x = 1 Ans.
116
Maths IIT-JEE ‘Best Approach’ (MCSIR) Logarithm

4. 5logx – 3logx-1 = 3logx+1 – 5logx-1, where the base of logarithm is 10.


Sol. Domain : x > 0
5log x – 3log x −1 = 3log x +1 − 5log x −1
 5log x – 3log x ·3–1 = 3log x ·3 – 5log x ·5–1
 1  1
 5log x 1 +  = 3log x  3 + 
 5  3
6  10 
 5log x   = 3log x  
5  3
log x 2
5 5
   =   logx = 2  (x = 100)  Domain
3  3
 x = 100 Ans.

1 + log 2 (x − 4)
5. =1
log 2 ( x + 3 − x − 3)
1 + log 2 (x − 4)
Sol. =1 ….(1)
log 2 ( x + 3 – x − 3)

 1 + log2(x – 4) = 2log2 ( x + 3 − x − 3)
 log22 + log2(x – 4) = log2(2x – 2 x 2 – 9 )
 2(x – 4) = 2x – 2 x 2 – 9
 x2 – 9 = 4
 x2 – 9 = 16  x = 5, –5 ; Put is eqn (1)
 x = 5 Asn.

6. log5 120 + (x − 3) − 2 . log5 (1 − 5x-3) = −log5(0.2 − 5x-4)


Sol. log5120 + (x – 3) – 2 . log5(1 – 5x–3) = – log5(0.2 – 5x–4) …(1)
 log5120 + log5(0.2 – 5x–4) – log5 (1 – 5x–3)2 = –(x – 3)
 120  (0.2 – 5x –4 ) 
 log5   = – (x – 3)
 (1 – 5x –3 ) 2 
24 – 24  5x −3
 = 5–(x–3) ; Put 5x–3 = t
(1 − 5x −3 )2
(1 – t) 2
 24 – 24t =  25t2 – 26t + 1 = 0
t
1
 t = 1,
25
 5 = 1, 5–2  x – 3 = 0, –2
x–3

 x = 3, 1 ; Put is eqn (1)


 x = 1 Ans.
117
Maths IIT-JEE ‘Best Approach’ (MCSIR) Logarithm

 1 
7. log 4 + 1 +
 2 x
 log 3 = log

( x 3 + 27 ) .
Sol. Domain : x  N – {1}
 1 
log4 +  1 +  log3 = log ( 3 + 27)
x

 2x 
 1 
1+ 
 log4 + log 3  2x 
= log(31/x + 27)
 1+ 1 x  
 log  4·3 2   = log(31/ x + 27)
 
 
 4 · 3 · 31/2x = 31/x + 27 ; Put 31/2x = t
 12t = t2 + 27
 t2 – 12t + 27 = 0
 t = 3, 9  31/2x = 3, 9
1  1 1
 = 1, 2   x = ,   Domain
2x  2 4
 x   (No solution) Ans.

x
8. If 'x' and 'y' are real numbers such that, 2 log(2y – 3x) = log x + log y, find .
y

x0 

Sol. Domain : y  0  
2y − 3x  0 
2log(2y – 3x) = logx + logy
 (2y – 3x) = logx + logy
 4y2 + 9x2 – 13xy = 0
Divide by y2 and Put x/y = t
4
 9t2 – 13t + 4 = 0  t = 1,
9
x 4
 = 1, Ans.
y 9

9. The real x and y satisfy log8x + log4y2 = 5 and log8y + log4x2 = 7, find xy.
1 2
Sol. log8x + log4y2 = 5  log2x + log2y = 5
3 2
 x1/3y = 25 ……(1)
and
1 2
log8y + log4x2 = 7  log2y + log2x = 7
3 2
118
Maths IIT-JEE ‘Best Approach’ (MCSIR) Logarithm

 y1/3 x = 27 ……(2)
(1) × (2)  (xy) = 212  xy = 29 Ans.
4/3

10. If a = log1218 & b = log2454 then find the value of ab + 5 (a − b).


Sol. a = log1218 and b = log2454
Change base to 3
log 3 18 log3 54
 a= and b=
log 3 12 log 3 24
log 3 (32  2) log 3 (33  2)
 a= and b=
log 3 (22  3) log 3 (23  3)
2 + log 3 2 3 + log 3 2
a= and b=
2 log 3 2 + 1 3log 3 2 + 1
 2alog32 + a = 2 + log32 and 3blog32 + b = 3 + log32
Eliminate log3 2
2− a 3−b
 =
2a − 1 3b − 1
 (2 − a)(3b −1) = (3 − b)(2a −1)
 6b − 2 − 3ab + a = 6a − 3 − 2ab + b
 ab + 5a − 5b = 3 − 2
 ab + 5(a − b) = 1

11. If x = 1 + logabc , y = 1 + logbca, z = 1 + logcab, then prove that xyz = xy + yz + zx.


Sol. x = 1 + logabc  x = logaa + logabc = logaabc
y = 1 + logbca  y = logbb + logbca = logbabc
1 1 1
 logabca = , logabcb = , logabcc =
x y z
Add all,
1 1 1
 logabca + logabcb + logabcc = + +
x y z
xy + yz + zx
 logabcabc =
xyz
 xy + yz + zx = xyz Hence proved.

12. If p = loga bc, q = logb ca, r = logc ab, then prove that pqr = p + q + r + 2.
Sol. p = logabc, q = logbca , r = logcab
Add 1 both sides is,
 p + 1 = 1 + logabc = logaabc
 q + 1 = 1+ logbca = logbabc
 r + 1 = 1 + logcab = logcabc

119
Maths IIT-JEE ‘Best Approach’ (MCSIR) Logarithm

1 1 1
 logabca = , logabcb = , logabcc =
p +1 q +1 r +1
Add all,
1 1 1
 logabcabc = + +
p +1 q +1 r +1
 (p + 1) (q + 1) (r + 1) = (q + 1) (r + 1) + (p + 1)(r + 1) + (p + 1)(q + 1)
On simplifying, we get
pqr = p + q + r + 2
Hence proved.

13. If logba . logca + logab . logcb + logac . logb c = 3 (Where a, b, c are different positive real
numbers  1), then find the value of abc.
Sol. Change base to “e”
na na nb nb nc nc
 . + . + . =3
nb nc na nc na nb
 (na)3 + (nb)3 + (nc)3 = 3 na. nb . nc
 only possible when,
na = nb = nc or na + nb + nc = 0

 a=b=c or n(abc) = 0
(Reject)  abc = 1 Ans.
 a, b, c are distinct

14. Let y = log 2 3·log 2 12·log 2 48·log 2 192 + 16 – log212 · log248 + 10. Find y  N.
Sol. y= log 2 3·log 2 12·log2 48·log2 192 + 16 – log212 · log248 + 10
 y= log 2 3·(2 + log 2 3)·(4 + log 2 3)·(6 + log 2 3) – (2 + log 2 3).(4 + log 2 3) + 10
Put log23 = t
 y = t(2 + t)(4 + t)(6 + t) – (2 + t)(4 + t) + 10

= (t 2 + 6t + 8)(t 2 + 6t – (t2 + 6t + 8) + 10

= (t 2 + 6t + 4) 2 – (t2 + 6t + 8) + 10
= (t2 + 6t + 4) – (t2 + 6t + 8) + 10
= 6 Ans.

3
15. Solve the equation log4(x + 2)2 + 3 = log4(4 – x)3 + log4(6 + x)3.
2
(x + 2) 2  0 

Sol. Domain : (4 − x)3  0    x  (–6, 4) – {2}
(6 + x)3  0 

120
Maths IIT-JEE ‘Best Approach’ (MCSIR) Logarithm
3
log4(x + 2)2 + 3 = log4(4 – x)3 + log4(6 + x)3
2
 3log4|x + 2| + 3 = 3 log4(4 – x) + 3log4(6 + x)
 log4|x + 2| – log4(4 – x) – log4(6 + x) = – 1
 | x +2| 
 log4   = −1
 (4 − x)(6 + x) 
| x +2| 1
 = 4 –1 =
(4 − x)(6 + x) 4
Case (i) x + 2 > 0  x > – 2
–(x + 2) 1
 =
(4 – x)(6 + x) 4
 x2 + 6x – 16 = 0
 x = –8, 2
Case (ii) x + 2 < 0  x < – 2
 4(x + 2) = (4 – x) (6 + x)
 x2 – 2x – 32 = 0
 x=1+ 33 , 1 − 33
 (x = 1 − 33 , 2)  Domain
 x = 1 − 33 , 2 Ans.

16. Find the product of the positive roots of the equation (2008)(x) log2008 x = x 2 .
Sol. Domain : x > 0
(2008) · x log2008 x = x 2
Take log2008 both sides
 log2008 ( )
(2008) · x log2008 x = 2 log 2008 x

 log 2008 2008 + log 2008 x ·log 2008 x = 2·log 2008 x


Put log2008x = t
1
 + t2 = 2t  2t2 – 4t + 1 = 0
2
2 2
 t=
2
2 2
 log2008x =
2
 2+ 2 2− 2 
  x = (2008) 2 & x = (2008) 2   Domain
 
 
 Product of roots = (2008)4/2 = (2008)2 Ans.

121
Maths IIT-JEE ‘Best Approach’ (MCSIR) Logarithm

 4  4  2 2 
17. Find x satisfying the equation log 2 1 +  + log 2 1 −  = 2 log  − 1 .
 x  x+4  x −1 
 4  4  2 2 
Sol. log2 1 +  + log2 1 −  = 2 log  − 1
 x  x+4  x −1 
 x+4 2 x  2  3− x 
 log 2   + log   = 2 log  
 x   x+4  x −1 
 x+4 2 x+4 2  3− x 
 log 2   + log   = 2 log  
 x   x   x −1 
 x+4 2  3− x 
 log 2   = log  
 x   x −1 

x+4  3− x   x+4  3− x 
 log   = log   or log   = − log  
 x   x −1   x   x −1 
x + 4 3− x x + 4 x −1
 = or =
x x −1 x 3− x
 (x + 4) (x – 1) = x (3 – x) or (x + 4) (3 – x) = x(x – 1)
 x2 = 2 or x2 = 6
 (x = 2,– 2 or x= 6 , – 6 ) ; Put is (1)
 x= 2, 6 Ans.

18. Solve : log3 ( x+ x −1 ) = log ( 4


9 x −3+ 4 x −1 )
Sol. log3 ( )
x + x − 1 = log9 4 x − 3 + 4 x − 1 ( )
 log3 ( x+ x −1 = ) 1
2
(
log 3 4 x − 3 + 4 x − 1 )
 2log3 ( x+ x − 1 ) = log ( 4
3 x −3+ 4 x −1 )

( )
2
 x+ x −1 = 4 x = −3 + 4 x − 1

( )
2
 x+ x −1 + 2 x x −1 = 4 x − 3 + 4 x −1

 2x + 1 – 2 x + 2 x | x – 1| = 4 x – 3 + 4 | x – 1|
 2x + 2 x | x – 1| = 6 x – 4 + 4 | x – 1|
Case(i) x – 1  0  x  1
 2x + 2 x ( x – 1) = 6 x – 4 + 4 ( x – 1)
 x–3 x +2=0
 ( x )2 – 3 x + 2 = 0  x = 1, 2  x =  
Case (ii) x –1<0  x<1
 2x – 2 x ( x – 1) = 6 x – 4 – 4( x – 1)
122
Maths IIT-JEE ‘Best Approach’ (MCSIR) Logarithm

 2x – 2x + 2 x = 6 x – 4 – 4 x + 4
 2 x =2 x  0=0
 x  (–, 1)
 x  (–, 1]  {4} Ans.

 
 log 4 ab + log 4 ab − log 4 b + log 4 a  · log b


a b a a b b

a
2 if b  a  1
19. Prove that : 2 =  loga b
2 if 1  b  a

Sol.  log 4 ab + log 4 ab – log 4 b / a + log 4 a / b  loga b


 a b a b 
LHS = 2
1
loga (ab) +logb (ab) − loga (b/a) +log b (a/b)  · log a b
= 22 
1
1+ loga b +1+ log b a − loga b −1+log b a −1 · loga b
= 2
2
a 1
Put logab = t  logab > 0 &
b 1
1 2
t + 2t +1 − t 2 − 2t +1 
t
2   t
= 2
1
[|t +1|−|t −1|]
= 2 2 –1 1
Case(i) t<–1  logab < – 1 Reject
Case(ii) –1t<1  – 1  logab < 1 ; But logab  0
 0 < logab < 1  1<b<a
1
[t +1+ t −1]
 LHS = 2 2 = 2t
LHS = 2loga b ; 1  b  a
Case (iii) t  1  logab  1  ba & a>1
 ba>1
1
[t + t − t +1]
 LHS = 2 2 =2
 LHS = 2 ; b  a > 1

 1  1
20. Solve for x : log2 (4 − x) + log (4 − x) . log  x +  − 2 log2  x +  = 0.
 2  2
4−x  0   1 
Sol. Domain :   x   − ,4
x + 1/ 2  0   2 
 1
Put log(4 – x) = p & log  x +  = q
 2
 p2 + p.q – 2q2 = 0
 p=q or p + 2q = 0
123
Maths IIT-JEE ‘Best Approach’ (MCSIR) Logarithm

 1  1
 log(4 – x) = log  x +  or log(4 – x) + 2log  x +  = 0
 2  2
2
1  1
 4–x= x+ or (4 – x) .  x +  = 1
2  2
7
 x= or x = 0 or 4x2 – 12x – 15 = 0
4
3 + 24 3 − 24
 x= ,
2 2
 7 3 + 24 3 − 24 
  x = 0, , ,   Domain
 4 2 2 

7 3 + 24
 x=0, , Ans.
4 2

124
Maths IIT-JEE ‘Best Approach’ (MCSIR) Logarithm

Logarithm Solved Exercise-V


1
1. Solve the following equations for x & y : log100|x + y| = [REE 96]
2
log10 y – log10|x| = log100 4.
1
Sol. log100|x + y| =  x  y  10 …..(1)
2
2
log10 y – log10|x| = log100 4  log10 2
2
y
  2  y  22 | x | …..(2)
|x|
From (1) & (2)  x  2 | x | 10

x0 x0
 3x  10  x  10
10 10
x ,  x  10, 10
3 3  
 
 
20
y y  20
3
 10 20 
 (x1y)   ,  or  10, 20 
 3 3 

2. Find all real numbers x which satisfy the equation, [REE 96]

2 log2(log2 x) + log1/2 log2 2 2 x = 1 
  3 
Sol. 2 log 2  log 2 x   log 21  log 2  2 2 x    1 ….(1)
 
  
  3 
 log 2  log 2 x   log 2  log 2  2 2 x    1
 
  
 
 2 

 log 2 
 log 2 x    1
  3 
 log 2  2 2 x  
   
 log 2 x   2
2

 ; Put log 2 x  t
3
 log 2 x
22
 t  3  2t  t 2  2t  3  0
 t  3, 1
 log 2 x  3, 1
1
 x  8, (rejected) ; Put in (1)
2
x  8

125
Maths IIT-JEE ‘Best Approach’ (MCSIR) Logarithm

3. log3/4log8 (x2 + 7) + log1/2 log1/4 (x2 + 7)1 =  2. [REE 2000]

Sol. log3/4log8 (x2 + 7) + log1/2 log1/4 (x2 + 7)1 =  2


1  1 
 log 3   log 2  x 2  7    log 2  log 2  x 2  7    2
4 3  2 
Put log 2  x  7   t
2

t t
 log 3    log 2    2
4  3  2
Change base to 2
t t
log 2   log 2  
  3   2   2
3 log 2 2
log 2  
4
log 2 (t)  log 3 2 log 2 t  1
   2
log 2 3  2 1
Put log 2 t   & log 2 3  q
pq
  (p  1)  2
q2
 p  q  (p 1)(q  2)  2(q  2)
 3p  pq  2q  6  0
 (3  q)(p  2)  0
 q  3 or p  2
 log 2 3  3
or log 2 t  2  t  4

 log 2  x 2  7   4
 x 2  7  16  x 2  9
 x  3, 3

4. Number of solutions of log4(x – 1) = log2(x – 3) is


(A) 3 (B) 1 (C) 2 (D) 0
[JEE 2001 (Screening)]
Ans. (B)
Sol. Domain:-
x 1  0
x 3  0   x 3
 log 22 (x  1)  log 2 (x  3)
1
 log 2 (x  1)  log 2 (x  3)
2
log 2  x  1  log 2  x  3
2

x  1   x  3
2

x 2  7x  10  0
 x  2,5  Domain

x  5

126
Maths IIT-JEE ‘Best Approach’ (MCSIR) Logarithm
5. Let (x0, y0) be solution of the following equations
(2x)ln2 = (3y)ln3 [JEE 2011]
3lnx = 2lny
Then x0 is :
1 1 1
(A) (B) (C) (D) 6
6 3 2
Ans. (C)
Sol. (2x)ln2 = (3y)ln3 & 3lnx = 2lny
Take “ln” both sides in both equation.
 ln2  ln(2x)   ln 3  ln(3n) 
 ln 2[ln 2  ln x]  ln 3[ln3  ln y] ….(1)
&
ln x(ln 3)  ln y(ln 3)  ln y(ln 2)
From equation (1) & (2) eliminate lny
ln x
 (ln 2)2  ln 2  ln x  (ln 3) 2  ln 3   ln 3
ln 2
 (ln 3)2  ln x
 (ln 2)2  (ln 3) 2    (ln 3) 2  
 ln 2  ln 2
ln x
 ln 2   ln 3    ln 2    ln 3  
2 2 2 2

ln 2
 ln x   ln 2
1
 x
2

 
 1 1 1 1  is
6. The value of 6 + log 4  4  4  ... [JEE 2012]
3
 3 2 3 2 3 2 3 2 
2
 
Ans. (4)
 
 1 1 1 1
Sol. To find  6  log 3 4 4 4 .... 

2 3 2 3 2 3 2 3 2 
 
 t 
 6  log 3   ….(1)
2 3 2 

1 1 1
 t  4 4 4 .... ; t  0
3 2 3 2 3 2
t
 t  4 , Square both sides
3 2
 3 2t 2  t 12 2  0
3 4 2
 t (reject) or t 
2 3
4 2
 Put t  in equation (1)
3
 1 4 2 4
 To find  6  log 3     6  log 3    6  2  4
 3 
2 3 2 2 9

127
Maths IIT-JEE ‘Best Approach’ (MCSIR) Logarithm
1 1

7. The value of is  (log 2 


9)2 log2 (log2 9)  ( 7) log 4 7
[JEE Advance 2018]
Ans. (8)
1 1

Sol.  (log 2 
9)2 log2 (log2 9)  ( 7) log 4 7

Put log 2 9  t
2
  t2 
log t 2 log7 2
 72

 t 2logt 2  2  t 2logt 2  2
 4 2= 8
8. Consider the statement : "P(n) : n2 – n + 41 is prime,. " then which one of the following is
true ? [Jee main 2019 (10-01-2019-shift-1)]
(A) Both P(3) and P(5) are true (B) P(3) is false but P(5) is true
(C) Both P(3) and P(5) are false (D) P(5) is false but P(3) is true.
Ans. (A)
Sol. p(n) = n2 – n + 41
n(5) = 61
n(3) = 47

9. The number of solutions of the equation log4(x – 1) = log2(x – 3) is :


[Jee main 2021 (26-02-2021-shift-1)]
Ans. (1)
Sol. log4(x – 1) = log2(x – 3)
1
 log2 (x  1)  log2 (x  3)
2
1/2
 (x – 1) = x – 3
 (x – 1) = x2 + 9 – 6x
2
 x – 7x + 10 = 0
 (x – 2) (x – 5) = 0
 x = 2, 5
But x  2 because it is not satisfying the domain of given equation i.e. log2(x – 3)  its
domain x > 3
Finally x is 5
 No. of solutions = 1.

10. The number of solutions of the equation


log(x +1) (2x2 + 7x + 5) + log(2x + 5) (x +1)2 – 4 = 0, x > 0 ,is____.
[Jee main 2021 (20-07-2021-shift-2)]
Ans. (1)
Sol. log(x+1) ((2x + 5) (x + 1)) + log(2x+5) (x + 1)2 = 4
1 + log(x+1) (2x+5) + 2log(2x+5) (x + 1) = 4
Put log(x+1) (2x + 5) = t
2
1+t+ 4
t
t2 + t + 2 = 4t  t2 – 3t + 2 = 0
t = 1, t = 2
For t = 1 For t = 2
2x + 5 = x + 1 2x + 5 = (x + 1)2
 x = – 4 (rejected) x = 2, x = – 2 (rejected)
128
Maths IIT-JEE ‘Best Approach’ (MCSIR) Logarithm

QUESTION BANK
[STRAIGHT OBJECTIVE TYPE]
1−a − b

Q.1 If 60a = 3 and 60b = 5 then the value of 12 2(1−b) equals


(A) 2 (B) 3 (C) 3 (D) 12
Ans. (A)

Sol. a = log60 3
b = log60 5
1−a −b 1−log60 3−log60 5 1−(log60 3+log60 5) log60 60 −(log60 15)
2(1−b) 2(1−log60 5) 2(1−log60 5) 2(log60 60−log60 5)
12 = 12 = 12 = 12
60
log60
15
60 log60 4 1
2log60 log12 12
= 12 5
= 12 log60 144
= 12log144 4 = 4 2 =2

log b + c a + log c − b a
Q.2 Let ABC be a triangle right angled at C. The value of
log b + c a·log c − b a
(b + c  1, c – b  1) equals
(A) 1 (B) 2 (C) 3 (D) ½
Ans. (B)

Sol. a 2 + b2 = c2
 c2 − b 2 = a 2
log b + c a log c −a a 1 1
+  +
log b + c a  log c − b a log b +c a  log c −b a log c −b a log b +c a
log a (c − b) + log a (c + b) = log a (c 2 − b2 ) = log a a 2 = 2

Q.3 The set of values of x satisfying simultaneously the inequalities


( x − 8) ( 2 − x )
 0 and 2x − 3 − 31 > 0 is :
log 0.3 ( 10
7 ( log 2 5 − 1))
(A) a unit set (B) an empty set
(C) an infinite set (D) a set consisting of exactly two elements
Ans. (A)

( x − 8) ( 2 − x )
Sol. 0 and 2x −3 − 31  0
log 0.3 ( 10
7 ( log 2 5 − 1) )
(x − 8)(2 − x)  0 and 2x −3  31
 (x − 8)(2 − x)  0 and (x − 3)log2 2  log2 31
 x [2,8] ….(i) and x − 3  log2 31
we know that when base of log is a x  3 + log2 31
0  a  1, y = loga x x  7 { log2 31  5}

129
Maths IIT-JEE ‘Best Approach’ (MCSIR) Logarithm

y  0, x  1 x  (7, ).......II
y  0, x  1
 10 
log 0.3  ( log 2 5 − 1) 
7 
 10  5 
 log 0.3   log 2 .2 − 1 
 7 2 
 10 
 log 0.3  log 2.5 
7 
 log 0.3 ( No.  1) = no.  0
(x − 8)(2 − x)
0
− ve

 (x − 8)(2 − x)  0
 (x − 8)(2 − x) = 0 [ x  0]
 x = {2,8} …..(ii)
From equation (i) & (ii)
x {2,8}......I
intersection of I and II
x=8

Q.4 If log0.3(x – 1) < log0.09 (x – 1) , then x lies in the interval


(A) (2 , ) (B) (1 , 2) (C) (1, ) (D) none of these
Ans. (A)

Sol. log0.3(x – 1) < log0.09 (x – 1)


Upon applying log rules, we get,
log 0.3 (x − 1)  log 0.3 x − 1 ( )
 x − 1  x − 1 and
x −1  0
Now,
x −1  x −1
( )  ( x − 1)
2
x −1
2

x − 1  x 2 − 2x + 1
x  1 or x  2
Now,
x −1  0; x  1
Overlapping all the intervals, we get,
x2

130
Maths IIT-JEE ‘Best Approach’ (MCSIR) Logarithm

 2x − 2007 
Q.5 Number of integral values of x the inequality log10    0 holds true, is
 x +1 
(A) 1004 (B) 1005 (C) 2007 (D) 2008
Ans. (B)

 2x − 2007 
Sol. log10  0
 x +1 
2x − 2007
 10 Also
x +1
2x − 2007 2x − 2007
1 0
x +1 x +1
2x − 2007  x + 1 x  −1
x  2008 2x − 2007  0
2x  2007
2007
x
2
x  1003.5
Possible values of x are (integers)
1004, 1005, ……., 2008
2008 − 1004
x= +1
1
= 1004 + 1 = 1005

[MULTIPLE OBJECTIVE TYPE]


Q.6 If y = log7–a (2x2 + 2x + a + 3) is defined x  R, then possible integral value(s) of a is/are
(A) – 3 (B) – 2 (C) 4 (D) 5

Ans. (BCD)

Sol. 2x 2 + 2x + a + 3  0, x  R

As D < 0, we have

4 − 4(a + 3)2  0

 1 − 2(a + 3)  0

 1  4a + 6

 −5  2a,a  −5 / 2

 7 − a  0,a  7,7 − a  1,6  a

131
Maths IIT-JEE ‘Best Approach’ (MC SIR) Logarithm
DPP-1
Time : 35 Min.

Straight Objective Type

1. If a4 · b5 = 1 then the value of loga(a5b4) equals


(A) 9/5 (B) 4 (C) 5 (D) 8/5
Ans. (A)
Sol. a4 · b5 = 1
taking log both sides base a
 4  5 log a b  0
now, loga(a5b4)
 5  4 log a b
 4 9
 5  4   
 5 5

2. If log175 5x = log343 7x, then the value of log42 (x4–2x2+7) is


(A) 1 (B) 2 (C) 3 (D) 4
Ans. (A)
Sol. 5x = 175k and 7 x = 343k
k
5  175 
  
7  343 

1
 k=
2
 x4 – 2x2 + 7 = 42
 log42 (x4 –2x2 + 7) = 1

3. The sum of all the solutions to the equation 2 log x – log(2x – 75) = 2
(A) 30 (B) 350 (C) 75 (D) 200
Ans. (D)
Sol. 2 log x – log(2x – 75) = 2
 log x 2  log(2 x  75)  2
x2
  102
2 x  75
 x 2  200 x  7500
 x  50,150
both satisfy this equation
hence ,sum = 200

132
Maths IIT-JEE ‘Best Approach’ (MC SIR) Logarithm
4. 10
log p (log r x ))
= 1 and logq (log r (logp x)) = 0, then 'p' equals
(A) rq/r (B) rq (C) 1 (D) r r/q
Ans. (A)
Sol. 10 =1
log p (log r x ))

 logp (logq (logr x)) = 0


 logq (logrx) = 1
 logr x = q
 x = rq (1)
logq (logr (logp x) = 1
 logpx = r
 x = pr (2)
From (1) and (2) rq = pr  p = rq/r

5. The value of log10  3 5  3 5  is


(A) 1/ 2 (B) 1/4 (C) 3/2 (D) 3/4
Ans. (A)
 62 5  62 5 
 
Sol. Log10  2 
 


   
5 1  5 1 
 
= log10  2 
 

2 5
= log10  2 
 

1
= log10 10 
2

B
6. If log4 A = log6 B = log9 (A +B) then the value of is
A

5 1 5 1
(A) (B)
4 4

5 1 5 1
(C) (D)
2 2
Ans. (D)
Sol. Let log4 A = 10log6 B = log9 (A + B) = x
 A = 4x, B = 6x and A + B = 9x
 4x + 6x = 9x
 22x + 2x . 3x = 32x
 (3/2)2x – (3/2)x – 1 = 0
x
3 B 5 1
    
2 A 2
133
Maths IIT-JEE ‘Best Approach’ (MC SIR) Logarithm
Multiple Objective Type
x
7. If p, q  N satisfy the equation x x
  x then p & q are
(A) relatively prime (B) twin prime
(C) coprime (D) if logqp is defined then logpq is not & vice versa
Ans. (A,C,D)
x
Sol. x x  x  
take log on both sides
 x log x  x log x
1
 x log x  x log x
2
 x
 x log x 1  0
 2 

x
 x  0, log x  0 or 1
2
 x  0,1, 4
 x  0 so x  1, 4
hence , p,q  1, 4
log41 is defind but log14 is not defind
also 1,4 are coprime or relatively prime

8. Which of the following when simplified, reduces to unity ?


2 log 2  log 3 1  64 
(A) log105 · log1020 + log102 2 (B) (C)  log5 log3 5
9 (D) log 3  
log 48  log 4 6 2  27 

Ans. (A,B,C)
Sol. (A) log105 · log1020 + log102 2
 log10 5(1  log10 2)  log10 2 2
 log10 5  log10 2(log10 5  log10 2)
 log10 5  log10 2
 log10 (5.2)  log10 10  1
2 log 2  log 3
(B)
log 48  log 4
log 22.3 log12
  1
 48  log12
log  
 4 
(C)  log5 log3 5
9
1/5 1/2
  log 5 log 3 (9 )
  log 5 log 3 (91/10 )

134
Maths IIT-JEE ‘Best Approach’ (MC SIR) Logarithm
  log 5 log 3 (31/5 )
1
  log 5    (1)  1
5
1  64 
(D) log 3  
6 2  27 
3
1 4
 log 3  
6 4
 3 

1 3  4
   log 3  
6 1 3
  4
2
4
 log 3    1
4 3

1  2 log3 2
9. The number N = 2
 log26 2 when simplified reduces to
1  log3 2
(A) a prime number (B) an irrational number
(C) a real which is less than log3 (D) a real which is greater than log76
Ans. (C, D)
1  2 log3 2
Sol. N= 2
 log26 2
1  log3 2
2
2  log 3 2 
now , log 6 2 
 log 3 6 

2  log32 2 
 log 6 2   
 1  log 3 2 

1  2 log 3 2  log 3 2 2
N 2
1 as Nr & Dr same
1  log 3 2 
also log 7 6  1  log 3 

135
Maths IIT-JEE ‘Best Approach’ (MC SIR) Logarithm
DPP-2
Time : 35 Min.

Straight Objective Type

1. The greatest integer less than or equal to the number log2 15 × log1/6 2×log3 1/6 is
(A) 4 (B) 3 (C) 2 (D) 1
Ans. (C)
Sol. log2 15 × log1/6 2×log3 1/6
log15 log 2 log1/ 6
= log 2. . log1/ 6 . log 3

log15 log 3  log 5


= log 3.  log 3

= 1+ log3 5 > 1 + 1

2. Given that log23 = a, log35 = b, log72 = c, then the value of log140 63 is equal to
2  ac 1  ac
(A) (B)
2c  1  abc c  2  abc

1  2ac 2  ac
(C) (D)
2c  1  abc c  2  abc
Ans. (C)
log 7 7  2 log 7 3
Sol. log140 63 = 2 log 2  log 7  log 5
7 7 7

1  2ac
=
2c  1  abc

3. The real value of x for which the statement log69 – log927 + log8x = log64x – log64 holds true, is
(A) 1/2 (B) 1/4 (C) 1/8 (D) 1/16
Ans. (C)
Sol. log69 – log927 + log8x = log64x – log64
 log 64 x 2  log 64 x  log 9 27  log 6 9  log 6 4

1
 log 64 x  log 33 33  log 6 36  x 
8

 3 2  2  1  0  (3  1)(  1)  0

log 2 x log 2 y log 2 z


4. If   and x2y2z =1 , then k is equla to
4 6 3k
(A) –8 (B) – 4 (C) 0 (D) 4

1
136
Maths IIT-JEE ‘Best Approach’ (MC SIR) Logarithm
Ans. (A)
log 2 x log 2 y log 2 z
Sol.  
4 6 3k

3log 2 x  log 2 y  log 2 z


=
12  12  3k

log 2  x 3 y 2 z 
=
24  3k

0
=
24  3k

 24 + 3k = 0  k = – 94
5. A line x = k intersects the graph of y = log4x and y =log4 (x + 4). The distance between the points of
intersection is 0.5, then the value of k is
(A) 1 (B) 2 (C) 3 (D) 4
Ans. (D)
Sol. We have log4 (k+4) – log4 x = 0.5

 log4 k  4 = 0.5
k

k4
 =2
k
 k=4

3 log 135
3 log 5
6. The value of log 3  log 3 is
15 405

(A) 1 (B) 2 (C) 3 (D) 4


Ans. (C)
log 3 135 log 3 5
Sol. 
log15 3 log 405 3
= (3 + log35) (1 + log35) – log3 5 log3 405
= (3 + log35) (1 + log35) – log3 5 log3 (81 × 5)
= (3 + log35) (1 + log35) – log3 5 (4 +log3 5)
=3

7. There exist positive integers A,B and C with no common factors greater than 1, such that
A log200 5+B log 2002 = C. The sum A+B+C equals
(A) 5 (B) 6 (C) 7 (D) 8
Ans. (B)
Sol. A log200 5+B log 2002 = C
 A log 5 +B log 2 = C log 200 = C log (5223)
 A log 5 + B log 2 = 2C log 5 + 3 C log 2
 A = 2C and B = 3C
For no common factor greater than 1, C = 1
 A = 2 ; B =3
 A + B + C =6

137
Maths IIT-JEE ‘Best Approach’ (MC SIR) Logarithm
8. The value of  2 log 18  .  3log 6 6 3
 is
(A) 5 (B) 6 (C) 7 (D) 8
Ans. (A)
Sol. 2 log 6 18
 .3 
log 6 3

= 2 log66  log63
 .3  log63

=  2  . 3 
log63 log63

3
= 2.6Log 6
6
= 2.3Log 6

= 2×3=6
Multiple Objective Type
log8 (8 x 2 )
9. The equation = 3 has :
(log8 x ) 2
(A) no integral solution (B) one natural solution (C) two real solutions (D) one irrational solution
Ans. (B,C)
log8 (8 x 2 )
Sol. =3
(log8 x ) 2
 log 8 8  log 8 x 2  3(log 8 x ) 2
Let log8 x  
1
   . 1
3
1 1
 log 8 x  .  1  x  2,
3 8

10. The expression, logp logp ...... p p where p  2, p  N, when simplified is


p p p

 
n radical sign
(A) independent of p, but dependent on n (B) independent of n, but dependent on p
(C) dependent on both p & n (D) negative.
Ans. (A,D)
Sol. logp logp p p p
...... p p

 
n radical sign
1/ p ........n
 log p log p  ( p)1/ p 

 log p log p  ( p)1/ p 


n

 1 
 log p  n  log p ( p)  n
p 

138
Maths IIT-JEE ‘Best Approach’ (MC SIR) Logarithm

Match The Column

11. Column-I Column-II


Following expressions simplifies to
log 2 32
(A) log3 243 (P) positive integer

2 log 6
(B) (Q) negative integer
log12  log 3

2
(C) log1/4  1  (R) rational but not integer
 16 

log5 16  log5 4
(D) (S) prime
log5 128
Ans. (A - P S : B - P : C - Q : D - R)
log 2 32 5
 2
Sol. (A) log 3 243 5
2
2 log 6 log 36
(B)  1
log12  log 3 log 36
2
 1 4
(C) log1/4    log1/ 4 (1/ 4)  4
16
 
log 5 16  log 5 4 log 5 4 2
(D)   log128 4 
log 5 128 log 5 128 7

139
Maths IIT-JEE ‘Best Approach’ (MC SIR) Logarithm
DPP-3
Time : 40 Min.

Straight Objective Type

1. Which one of the following is the smallest ?


3
 1   1 
 
(A) log10 (B) log10  2 (C)  
 (D)  
log
 10   log10  
Ans. (B)
Sol. log10 < 1 ....(i)
also log10   log10 

2 1

 2 log10   1.log10 

2
 lo g 1 0   lo g 1 0  ....(ii)

3
 1  3
(C)    (log 10) >1 as log  10 >1
log
 10 

 1  1

(D)    log 10  log 2 10  1
 log10   2 

2. If log 2 log 3 log 4 ( x )   0 and log3 log 4 log 2 ( y)   0 and log 4 log 2 log3 (z)   0 then the sum
of x, y and z is
(A) 89 (B) 58 (C) 105 (D) 50
Ans. (A)
Sol. log 2 log 3 log 4 ( x )   0
 log 3 (log 4 x)  1

 (log 4 x)  3

 x  64
now, log3 log 4 log 2 ( y)   0

 log4 (log2 y)  30 1

 (log 2 y )  4  y  24  16

and log 4 log 2 log3 (z)   0

1
140
Maths IIT-JEE ‘Best Approach’ (MC SIR) Logarithm
 log 2 (log 3 z )  1

 log3 z  2 z 32 9

 log 3 z  2  z  32  9

 x  y  z  64  16  9  89

c
3. If log2 b = 2, logbc = 2 and log3c = 3 + log3a , then the value of is
ab
(A) 1 (B) 3 (C) 9 (D) 27
Ans. (B)
Sol. Log3 c = 3 + log3 a
c
 log3 =3
a
 c = 27 a
logab = 2 and logbc = 2
 logab. logbc = 4
 logac = 4
 c = a4
From (1) and (2) we get
a = 3 and c = 81
 form loga b = 2, we get b = a2 = 9
c
 =3
ab

log e 10
4. If log2 10 = p : log 7 = q and (11)r = 10, then which one of the following expression is equivalent ot
e

log10154 ?
1 pqr pq  qr  rp
(A) pqr (B) pqr (C) pqr (D) pqr

Ans. (D)
Sol. log10154 = log10 2 + log107 + log1011
Now log2 10 = p
1
 log 2  p
10

1
 log102 = p (1)

log e 10
q
log e 7

1
 log 7  q
10

141
Maths IIT-JEE ‘Best Approach’ (MC SIR) Logarithm
1
 log10 7 q

and 11r = 10
 r log 10 11 = 1
1
 rog 1011 =
r

1 1 1 pq  qr  rp
 log10154 = p  q  r  pqr

5. If 3(log37)x =7 (7log73)x , then the value of x will be


1 1
(A) (B)
2 4

1
(C) (D) 1
3
Ans. (A)
Sol. Consider a log a b
b
log b a

 log a b  log b a loga b ( taking log with base 'a')

 1 = log b a  log a b which is true

1
 from 3(log37)x .= 7 (log73)x , x =
2

6. The value of 45log  3 6   6 log  4 2


8 3 2  is

(A) 3 (B) 6 (C) 9 (D) 27


Ans. (C)
Sol. 5log  3 6 6log  3 2 
4 4 2  8 

    
2log2 3 3 2  2log  3 2 
2
= 4    

= 42log2 3

 
4
log 2 3 
= 2  

=9
1 3
9 3

81log5  3 log 6
 log2257 log 6 
7. The value of  7  125  25 
409  

(A) 0 (B) 1 (C) 2 (D) 3


Ans. (B)
1 3
3
log59 log 6
Sol. 81 3  log2257 log 6 
 7  125  25 
409  
3
2 log  6
9log9 5  3
=
409
7 log7 25
 5log5 6
3/ 2


142
Maths IIT-JEE ‘Best Approach’ (MC SIR) Logarithm
25  6 6
= [25  6 6]
409

625  216 409


=  1
409 409

8. The value of log 40.5log 36 is


6 10 10

(A) 200 (B) 216 (C) 432 (D) none of these


Ans. (B)
Sol. We have N = 6 log1040 .5log1036
= log10 N= log10 40 log106 + log1036 log105
= log10 6 [log10 40 + log1025]
= log106 [log10 1000]
= log10 (6)3
 N = 63 = 216

Multiple Objective Type

9. Consider the quadratic equation, (log108)x2 – (log105)x = 2(log210)–1 – x. Which of the following
quantities are irrational.
(A) sum of the roots (B) product of the roots
(C) sum of the coefficients (D) discriminant
Ans. (C,D)
Sol. log108 =3log102 ,log105 = 1 - log102

(log 2 10) 1  log10 2


so equation
3(log102)x2  (1  log102)x = 2(log102)  x
 3(log10 2) x 2  (log10 2) x  2 log10 2  0

1 2
 sum of roots =  ,product of roots = 
3 3
sum of coefficients
= (3 + 1  2)log102
=2log102 = irrational

10. In which of the following case(s) the real number 'm' is greater than the real number 'n'?
(A) m = (log25)2 and n = log220 (B) m = log102 and n = log10 3 10
(C) m = log105 · log1020 + (log102)2 and n=1 (D) m = log1/2 1 3 and n = log1/3 1 2

143
Maths IIT-JEE ‘Best Approach’ (MC SIR) Logarithm
Ans. (A,D)
Sol. (A) m = (log25)2 , n = log220 = 2 + log25

m  n  log 2 2 5  log 2 5  2 = (log25 + 1)(log25  2) = +ve *  ve


so m > n

1
(B) m = log102 = 0.3010 and n = log10 3 10 = =0.333
3
So, m < n
(C) m = log105 · log1020 + (log102)2= log105(log1022.5) + log1022
= log105(2log102 + log105) + log1022
= 2ab + b2 + a2 = (a + b)2
with a = log102 , b =log105 then a + b = 1
so, m = 1 = n

(D) m = log1/2 1 3 = log23 >1 and n = log1/3 1 2 = log32 < 1


So m > n
Match The Column
11. Column-I Column-II
(A) The value of K, (P) 1
where log (log 4) + log (log 25) = log K + log (log 2) + log (log 5) is
(B) Number of values of x  N, for which x4 + 4 is prime, is (Q) 2
(C) If b is a positive real number different from 1, let logbx denotes the base b
logarithm of x. Let n be the number of solutions of x to the equation, (R) 3
logbx = logxb where x is a positive real different from 1. Then n equals
(D) The expression log0.52 8 has the value equal to (S) 4
Ans. (A - S: B - P : C- Q :D - R)
Sol. (A) log (log 4 log 25) = log K + log (log 2 log 5)
log (4log 2 log 5)  log (log 2 log 5) = log K
log 4 = logk
K=4
(B) x4 + 4 = (x2 + 2)2  (2x)2 = (x2  2x +2)(x2 +2x+2) = Prime iff (x2  2x +2) = 1 i.e.
x =1 as x  N
1
(C) logbx = logxb = log x
b

(logbx)2 = 1 or  1

1
x = b or so,no of solution is 2
b

(D) log0.52 8 = 3log 21 2  3  3

144
Maths IIT-JEE ‘Best Approach’ (MC SIR) Logarithm
DPP-4
Time : 35 Min.

Straight Objective Type

1. If x = 2  5 , then log
5 2
10x =  
A  B (log102) + B . The value of (A + B) equals
(A) 7 (B) 9 (C) 11 (D) 13
Ans. (A)
Sol. x= 2   5  then
5 2

log10x =  
A  B (log102) + B

 log10 2  5  
5 2
5 log10 2  2 log10 5

 5 log10 2  2 log10 5  2  2 log10 2  2 log10 2

5 log10 2  2 log10 2  2   
5  2 log10 2  2

A  B  5 2  7

2. Let u = (log2x)2 – 6 log2x + 12 where x is a real number. Then the equation xu = 256 has
(A) no solution for x (B) exactly one solution for x
(C) exactly two distinct solutions for x (D) exactly three distinct solutions for x
Ans. (B)
Sol. u = (log2x)2 – 6 log2x + 12 (i), x u  256 (ii)

u log 2 x  log 2 28    2  6  12   2  8

3  6 2  12  8  0  3  23  3  2(   2)  0
3
    2   0    2  log 2 x  2  k  4

 
1 2 1 1
3. The value of log 9  6 6 6 ..... 

4 2 3 2 3 2 3 2 3 
 

(A) – 2 (B) – 1 (C) – 1/2 (D) none of these


Ans. (C)

1 2 1 1
Sol. 6 6 6 .....
2 3 2 3 2 3 2 3

1
 x = 6x
2 3
 12x2 + x – 6 = 0
 (3x – 2) (4x +3) = 0

145
Maths IIT-JEE ‘Best Approach’ (MC SIR) Logarithm
2
 x =
3

 
 1 2 1 1 
  9
log 6  6  6  .....

4 2 3 2 3 2 3 2 3
 

  2 1
= log 9  3    2
4

4. Number of real values of x satisfying the equation log x 2


 6 x 8
(log log 2x 2
 2x  3
(x2–2x)) = 0 is equal to
(A) 3 (B) 2 (C) 1 (D) 0
Ans. (C)
Sol. log x 2  6 x  8 (log log 2x 2
 2x  3
(x2–2x)) = 0
2
 log 2x  2x  3
(x2 – 2x) = 1
 x2 –2x = 2x2 + 2x+ 3
 (x+1) (x + 3) = 0
 X = –1, – 3
But for x = –3, x2 +6x + 8 < 0
 x=–1

5. If logab + logbc + logca vanishes where a, b and c are positive reals different than unity then the value
of (logab)3 + (logbc)3 + (logca)3 is
(A) an odd prime (B) an even prime
(C) an odd composite (D) an irrational number
Ans. (A)
Sol. logab + logbc + logca  0, a, b, c  0, a  b  c  1
3 3 3
 log a b    log b c    log c a   3log a b  log b c  log c a
 3 1  3 (An odd prime)

Subjective

6. Solve the equation, log( x )  log x 2 (base is 10)

Sol. log( x )  log x 2 ( x  0)


2
log(  x)  log(  x)   log(  x)   log(  x)
log(  x)(log(  x)  1)  0
log(  x)  0, log(  x)  1  x  1, 10

7. If log a  log b  log c , show that aa . bb . cc = 1.


bc ca a b

146
Maths IIT-JEE ‘Best Approach’ (MC SIR) Logarithm

Sol. loga logb logc


  k
b c c a a b
a  10k (b c)  a a  10k (ab ac) .....(i)
b  10 k (c  a )  b b  10 k (bc  ab) .....(ii)
c  10k (a  b)  c c  10k (ac  bc) .....(iii)
i  ii  iii  a a b b c c  10k0  1

8. Find the value of x satisyfing the equations log3 (log2x) + log1/3 (log1/2y) = 1 and xy2 =9
Sol. log3 (log2x) + log1/3 (log1/2y) = 1
 log3 (log2x) – log3 (–log2 y) = 1
 log x 
2
 log3   log y  = 1
 2 

log x
2
  log y = 3
2

 xy3 = 1
Also, xy2 =9
1
 y=
9
 x = 729

3
9. Solve : log4 (x+2)2 + 3 = log4 (4–x)3 + log4 (6 + x)3
2

3
Sol. log4 (x+2)2 + 3 = log4 (4–x)3 + log4 (6 + x)3
2
3
 24  2x  x 2 
 3 = log4  | x  2 | 
 

3
 24  2x  x 2  3
  |x2|   4
 

24  2x  x 2
 4
| x2|
if x + 2 > 0
 x2 + 6x – 16 = 0
 (x –2) (x+8) = 0
 x=2
if x + 2 < 0
x2 – 2x –32 = 0
 x = 1 – 33

147
Maths IIT-JEE ‘Best Approach’ (MC SIR) Logarithm
10. Solve : log3/4 log8 (x2 +7) + log1/2 log1/4 (x2+7)–1 = –2
Sol. log3/4 log8 (x2 +7) + log1/2 log1/4 (x2+7)–1 = –2

1  log 2  x 2  7 
 log 2  x  7    log 2
2
 log3/4  2
3  2
let log2(x2 + 7) = t
t t
 log3/4  log 2  2  0
3 2

t   t
 log3 / 4 3  1   l og 2 2  1  0
 

t t
 log3 / 4  log 2
4 4
 t=4
 log2 (x2 +7) = 4
 x2 + = 16
 x2=9
 x= 3

11. If 5log x – 3 log x–1 = 3log x+1 –5 log x–1 , where the base of logarithm is 10, then find the value of x.
Sol. 5log x – 3 log x–1 = 3log x+1 –5 log x–1
5log x 3log x
 5log x + = 3 . 3log x +
5 3

6.5log x 10.3log x
 
5 3
log x 2
3 3
    
5 5
 log10 x = 2
 x = 100

12. Slove : 4 | x  3 |x 1  3 | x  3 |x  2

Sol. 4
| x  3 |x 1  3 | x  3 |x  2
Taking log on both the sides
x 1 x2
log|x–3| = log |x–3|
4 3

 x 1 x  2 
 log |x–3|  4  3   0
 

 x  1   x  2  
 log |x–3| = 0 or  4    3    0
   
 x = 4, 2 or x = 11

148
Maths IIT-JEE ‘Best Approach’ (MC SIR) Logarithm
13. Solve log x 16 + log2x + 64 = 3
2

Sol. log x 2 16 + log2x + 64 = 3

4 6
 log x 2  log 2x  3
2 2

2 6
 log x  1  log x  3
2 2

Put log2 x t
2 6
  3
t lt
 2 +2t + 6t = 3t +3t2
 3t2 – 5t – 2 = 0
 (3t +1) ( t–2) = 0
1
 t=  ,t=2
3

1
 log2x =   or log2 x = 2  x = 2–1/3 or x = 4
3

149
Maths IIT-JEE ‘Best Approach’ (MC SIR) Logarithm
DPP-5
Time : 40 Min.

Straight Objective Type


1. If logab = 2; logbc = 2 and log3c = 3 + log3a then (a + b + c) equals
(A) 90 (B) 93 (D) 102 (D) 243
Ans. (B)
Sol. logab = 2, logbc = 2
 log a b  log b c  4

 log a c  4iec  a 4

Now log a c  3  log 3 a

 log 3 a 4  3  log 3 a

 4 log 3 a  3  log 3 a

 3log3 a  3

 log 3 a  1 or a  3

 c  a 4  34  81
Also log b  2

b  a2  9
So a  b  c  93

2. Let 3a = 4, 4b = 5, 5c = 6, 6d = 7, 7e = 8 and 8f = 9. The value of the product (abcdef), is


(A) 1 (B) 2 (C) 6 (D) 3
Ans. (B)
Sol. 3a  4, 4b  5,....,8t  9
take log on base 10
a log 3  log 4, b log 4  log 5,....., f log 8  log 9

log 4 log 5 log 9


 a ,b  ......., f 
log 3 log 4 log 8

log 9
 abcdef  2
log 3

1
150
Maths IIT-JEE ‘Best Approach’ (MC SIR) Logarithm
3. If log9x + log4y = 7 2 and log9x – log8y = – 3 2 , then x + y equals
(A) 35 (B) 41 (C) 67 (D) 73
Ans. (C)
Sol. log9x + log4y = 7 2

log9x – log8y = – 3 2
7 3
 log 4 y  log 8 y 
2 2
  log 4  log 4  5
1 1
 log 2 y     5
 2 3
5
 log 2 y    5
6
 log 2 y  6

 y  26  64
Then by (1)

7
log 9 x  log 4 64 
2
1
 log 9 x  so x = 3
2
 x  y  67

4. Let A denotes the real value of x satisfying the equation x3 + 3x2 + 3x + 4 = log12(1728) and

B = 132  132  132......... then (A – B) equals


(A) 1 (B) 13 (C) – 12 (D) – 13
Ans. (D)
Sol. x3 + 3x2 + 3x + 4 = log121728 = 3
 (x  1)3  3  3
 x  1  A
Now B = 132  132  132.........

 B  132  B
Sqauring both sides
B2  132  B
B2  B  132  0
B  12, 11 (rejected)
A  B   1  12   13

2
151
Maths IIT-JEE ‘Best Approach’ (MC SIR) Logarithm
Multiple Objective Type

5. If log45 = x and log56 = y then


1 1
(A) log46 = xy (B) log64 = xy (C) log32 = (D) log23 =
2 xy  1 2 xy  1
Ans. (A,C)
log 5 log 6 log 6
Sol. xy = log45log56 = .   log 4 6 ,true
log 4 log 5 log 4

1
xy = log 4 6  xy  log 2 (2.3)  2 xy  1  log 2 3
2
1
  log3 2
2 xy 1

6. If log1/2 (4–x)  log1/2 f2–log1/2 (x –1) , then x can belong to


(A) (1,2] (B) [1,3] (C) [3,4] (D) [2,3]
Ans. (A,C)
Sol. We have log1/2 (4–x)  log 1/2 2– log1/2 (x–1)
It is defined if 4–x > 0 and x –1 > 0 or 1 < x < 4 (i)
 log 1/2 (4–x) (x–1)  log 1/2 2
 (4 – x) ( x –1)  2
 x2 –5 + 6  0
 x  3 or x  2
9 1
7. The values of x satisfying 2 log 1 (x  5) 4 log 3 3  9   log x 5 (2) is / are
4

(A) (–5,– 4) (B) (–3, –1) (C) (–4, –1) (D) (–5, –2)
Ans. (A,B)
9 1
Sol. we have 2 log 1 (x  5) 4 log 3 3  9   log x 5 (2)
4

9 4 2
 – log2 (x +5) > 4   3   log  x  5 
2

2
 3 > log2 (x+5) + log  x  5 
2

Let log 2 (x +5) = y


2
 3>y+y

y2  2
 3 0
y
y 2  3y  2
 0
y

152
Maths IIT-JEE ‘Best Approach’ (MC SIR) Logarithm
 y  2  y  1
 0
y

Using sing scheme method , we get


y 
 log2 (x+5) (,0),  (1,2)
 (x + 5)  (2–,2°) (2122)
 (x+5)  (0,1)  (2,4)
 x (–5,–4)  (–3,–1)
3
8. If log x – (log3 x)2  log 4, then x can belong to
2 1/ 2 2 
(A) (–, 1/3) (B) (9, )
(C) (1,6) (D) (–,0)
Ans. (A,B)
3
Sol. Log3 x – (log3x)2   log1/ 2 2  4
2
 log3 x –(log3x)2 – 2
 (log3x)2 – log3 x –2 0
 (log3x–2) ( log3 x +1) 0
 x  1/3 or x 9
Match The Column

9. Column-I Column-II
log10 x
(A) If x1 and x2 satisfy the equation x = 100x (P) irrational
then the value of x1x2 equals
(B) Sum of the squares of the roots of the equation log2(9 – 2x) = 3 – x is (Q) rational
(C)   
If log1 8 log1 4 log1 2 x = 1/3 then x is (R) prime

(D) If logba = 3, logbc = – 4 and if the value of x satisfying the equation, a3x =cx – 1 (S) composite
is expressed in the form p/q, where p & q are relatively prime then (p+q) is
Ans. (A)  Q, S; (B)  Q, S; (C)  P; (D)  Q, R
Sol. (A) x log x  100x  (log x) 2  2  log x
Its quad in log x, then

1
log x  1, 2 ie x1 , x 2  ,100
10

x1x 2  10 (a rational & Composite)

(B) log 2 (9  2 x )  3  x

8
 9  2 x  23  x  x
2

153
Maths IIT-JEE ‘Best Approach’ (MC SIR) Logarithm
 9t  t 2  8; t  2x

 t  2x  1,8

 x  0,3
required sum  0  32  9
= rational & Composite
 
(C) log1 8 log1 4 log1 2 x = 1/3 
1
3
1 3 1
 log 1 log 1 x    
4 2 2 2

1
 1 2 1
 log 1 x    
2 4 2

1
 x irrational
2

(D) log b a  3, a  b3

log b c  4, c  b 4

4
 13  4

 c  a   a 3
 

By a 3x  c x 1
4
(x 1)
 a 3x  a 3
4 4
3x  x 
 a 3 3
1

13x 4
 
3 3

4 p
 x 
13 q

 p  q  17 Prime rational

154
Maths IIT-JEE ‘Best Approach’ (MC SIR) Logarithm
Subjective
10. Establish tricotomy in each of this following pairs of numbers
log 27 3 log 2
(i) 3 2 4 and (ii) log45 and log1/16(1/25)
(iii) 4 and log310 + log1081 (iv) log1/5(1/7) and log1/7 (1/5)
log 27 3 1
Sol. 3  33
log 4 2 1
2  22
1 1
Now 33 , 2 2

 32  23  9,8
 9>8
(ii) log45 and log1/16(1/25)

2
Sol. log1/16(1/25)  log 4 5
2

 log 4 5

(iii) 4 and log310 + log1081


Sol. log310 + log1081
 log 3 10  4 log10 3

4
 t2  Say
t2

2
 2
t  4  4
 t

(iv) log1/5(1/7) and log1/7 (1/5)


Sol. log1/51/7 = log 5 7  1

log1/7 1/5 = log 7 5  1


 log1/7 1/5 > log1/7 1/5

11. Compute the value of


1 4
log 5 3 log 9 36 log 7 9
81  27  3

155
Maths IIT-JEE ‘Best Approach’ (MC SIR) Logarithm
1
Sol. (81) log5 3  34.log3 5  54

 625
27 log9 36  33log3 6  63
= 216
4 4 log 7
3 9
log7 9
3  34log9 7  32log 7

 7 2  49
sum = 625  216  49
= 891

12. Let a and b real number greater than 1 for which there exists a positive real numbers c. different from 1,
such that 2(logac +logbc) = 9 logabc. Find the largest possible value of logab.
Sol. a>,b>1
2 (loga c + logbc) = 9 logabc
  log b  log a   log c
 2 log c  log a log b    9 log a  log b
  
 2(log a + log b)2 = 9 (log a) (log b)
 2 (log a)2 + 2 (log b)2 + 4 (log a) (log b)
= 9 (log a) (log b)
 2logb a + 2 loga b + 4 = 9
 2logba + 2logab = 5
1 5
 t  where t = logab
t 2
 2t2 –5t + 2 = 0
 (2t–1) (t–2) = 0
 t=1/t=2
 logab = 1/2 or logab = 2

13. Solve : log3x. log4x. log5x = log3x. log4x + log4x. log5x+ log5x. log3x.
Sol. Solve : log3x. log4x. log5x = log3x. log4x + log4x. log5x+ log5x. log3x.
1 1 1 1
 pqr  pq  qr  pr

 p + q +r =1
 logx3 + logx4 + logx 5 =1
 logx 60 = 1
 x = 60
14. Solve the system of equations :
(logax) (loga (xyz)) = 48
(logay) (loga(xyz)) = 12
(logaz) (loga(xyz)) = 84
Sol. Log a(xyz) [logax + loga y + logaz] = 144

156
Maths IIT-JEE ‘Best Approach’ (MC SIR) Logarithm
 loga (xyz) = (144)1/2 =12
 xyz = a12
From logax loga (xyz) = 48
 (logax) (12) = 48
 logax = 4
 x = a4
Similarly y = a and z = a7

DPP-6
Time : 40 Min.

Straight Objective Type


1. Let B, C, P and L be positive real numbers such that
log (B · L) + log (B · P) = 2; log (P · L) + log (P · C) = 3; log (C · B) + log (C · L) = 4
The value of the product (BCPL) equals (base of the log is 10)
(A) 102 (B) 103 (C) 104 (D) 109
Ans. (B)
Sol. log BL  log BP  2  log10  BL LP   2

log PL  log PC  3  log10  P L LC   3

log CB  log CL  4  log10  C L BL   6

 BL LP  102
P L LC  103
C L BC  10 4
Taking product
B3 L3 P 3C3  109
BLPC  103 Taking cube root

1
2. If x = logkb = logbc = logcd then logkd equals
2

x3
(A) 2x3 (B) (C) 2x8 (D) 6x
2
Ans. (A)
1
Sol. X  log k b  log b c  log c d
2
 b  (k) x  c  (b) x

 d  (c) 2x
3 3
 d  (k) 2x log k (k)2x  2x 3
157
Maths IIT-JEE ‘Best Approach’ (MC SIR) Logarithm
3. If log2 (log2(log2(log2x) = 2 , then the number of digits in x, is (log102 = 0.3010)
(A) 7 (B) 6 (C) 5 (D) 4
Ans. (C)
Sol. log2 (log2(log2(log2x) = 2
 log2 (log2x) = 4
 log2 x = 16
 x = 216
 log10 2 = 16 × 0.3010 = 4.8160
 number of digits =5

4. The number of integers satisfying the inequality log 0.9 log5  


x 2  2  x > 0 is

(A) 6 (B) 7 (C) 8 (D) 9


Ans. (C)

Sol. log 0.9 log5  


x2  2  x > 0

 0 < log5  x2  5  x  1
 1 < (x2 + 5 + x)1/2 < 5
 1 < x2 + 5 + x < 25
 x2 + x – 20 < 0 (as x2 + x + 4 > 4 > 0 for all real x)
 (x + 5) (x – 4) < 0
 x ( –5,4)
1  log 4 x 1
5. The smallest integra x satisfying the inequality 1  log x  2 x is
2

(A) 2 (B) 2 (C) 3 (D) 4


Ans. (B)
Sol. Let log 2x = t
1   t / 2 1
 
1 t 2
2t
 1
1 t

2t
 1  0
1 t

2t  1
 0
t 1

1
 t < – or t 
2

1
`  log2x < –1 or log2x 
2

1
 0<x< or x  2
2
 smallest integes is 2

1
158
Maths IIT-JEE ‘Best Approach’ (MC SIR) Logarithm
6. The number of integral solutions of log9 (x + 1) log2 (x +1) – log9 (x +1) – log 2 (x +1) + 1 < 0 is

(A) 4 (B) 5 (C) 6 (D) 7


Ans. (C)
Sol. Wh have log9 (x + 1) log2 (x +1) – log9 (x +1) – log 2
(x +1) + 1 < 0
 (log9 (x +1) –1) (log2 (x +1) –1) < 0
  x 1   x 1
  log9  9    log 2  2    0
     

 x 1
case I : log9  9   0
 

x 1
 1
9

 x>8

 x 1
 log2  2   0
 

x 1
 0< <1
2
 –1 < x < 1
 form (1) and (2) , we get x d

 x 1
Case II . log9  9  < 0
 

x 1
 0< <1
9

 –1 < x < 8
x 1
 log2 1
2
 x>1
 from (3) and (4), x (1,8)
So integral values are x = 2,3,4,5,6,7

 2  x  2 
7. The number of integers satisfying log log 1   x  1 x  5    1 is
x  
(A) 0 (B) 0 (C) 2 (D) 3
Ans. (A)
Sol. Case I : 1/x > 0 or 0 < x < 1
 2  x  2 
 log 1   x  1 x  5   1
x  

2  x  2 1
  x  1 x  5  x
159
Maths IIT-JEE ‘Best Approach’ (MC SIR) Logarithm
2  x  2 1
  x  1 x  5  x  0

2x  x  2    x  1 x  5 
 0
x  x  1 x  5 

x2  5
 x  x  1 x  5   0

 x  (–1,0)  (5,)
Hence, no solution in this case
1
Case II : 0 < < 1 or x > 1
x
 0 < x < 5
2  x  2
Also  x  1 x  5  0

 x  (1,2)

160
Maths IIT-JEE ‘Best Approach’ (MCSIR) Logarithm

PART TEST – 1
Single Correct Answer Type

 1 
1. Find the value of x satisfying the equation log 3  log 9 x + + 9 x  = 2x is
 2 
1 1 1
(A) (B) (C) (D) none of these
2 3 4
Ans. (B)
 1 
Sol. log3  log 9 x + + 9 x  = 2x ....(1)
 2 
1
 log9x + + 9x = 32x = 9x
2
1 1
 log9x = –  x = 9–1/2 = ; Put in eqn (1)s
2 3
1 1
x= satisfy eqn (1) ;  x =
3 3
log x + 7
2. Number of real values of x satisfying the equation x 4 = 10log x +1 is equal to
(A) 1 (B) 2 (C) 3 (D) 4

Ans. (B)
Sol. Domain : x > 0
log x + 7
x 4 = 10log x +1
Take log both sides.
log x + 7
 × logx = (logx + 1) × log10 ; Put log x = t
4
 t2 + 3t – 4 = 0  t = – 4 , 1
 log x = – 4, 1
 (x = 10 , 10)  Domain
–4

 x = 10–4 , 10 Ans.

x −3 x −3
3. Find the value of x satisfying the equation 2 log3 + 1 = log3 is
x −7 x −1
(A) 5 (B) 4 (C) − 4 (D) − 5
Ans. (D)
x −3 


 0
 x  (−, 3)  (7, )
Sol. Domain : x − 7     x  1 x  7
x −3  x  (– ,1)  (3, )
0 
x −1 
 x −3  x −3
2 log 3   + 1 = log 3  
 x −7  x −1 

161
Maths IIT-JEE ‘Best Approach’ (MCSIR) Logarithm

  x − 3 2 
  
  x −7  (x − 3)2 (x − 1) 1
 log 3 = −1   =
  x −3  (x − 7) 2 (x − 3) 3
  x −1  
   
 3(x – 3) (x – 1) = (x – 7)2
 x2 + x – 20 = 0
 (x = – 5, 4)  Domain
 x=–5

4. If log2 x + log2 y  6, then the least value of x + y is :


(A) 8 (B) 16 (C) 32 (D) 12
Ans. (B)
Sol. log2x + log2y  6 ; x, y > 0
 log2 (xy)  6  xy  26  xy  64
 x & y are positive
 x + y = ( x ) 2 + ( y) 2 = ( x − y) 2 + 2 xy
0

 x + y  2 xy  2 × 8  x + y  16
 least value of x + y = 16

5. Let a and b be real numbers greater than 1 for which there exists a positive real number c,
different from 1, such that 2(logac + logbc) = 9logabc. Find the largest possible value of logab.
(A) 0 (B) 1 (C) 2 (D) 3
Ans. (C)
Sol. a  1, b  1,c  0 (c  1)
2(logac + logbc) = 9logabc
Change base to ‘a’
log a c
 2  log a c + log b c  = 9 
log a ab
 1  1
 2 1 +  = 9 ; Put loga b = t
 log a b  1 + log a b
 1 9
 2 1 +  =  2(t − 1) 2 = 9t
 t  1+ t
1
 2t 2 − 5t + 2 = 0  1t = 2,
2
1
 loga b = 2 or log a b =
2
 Largest value of loga b = 2

162
Maths IIT-JEE ‘Best Approach’ (MCSIR) Logarithm

x
6. If 'x' and 'y' are real numbers such that, 2 log(2y – 3x) = log x + log y, find .
y
4 4 9 3
(A) 1, (B) 1, (C) 1 , (D) 1,
3 9 4 4
Ans. (C)
Sol. Domain :-
x0 

y0 
2y − 3x  0 
2log(2y − 3x) = log x + log y
 (2y − 3x)2 = xy

 4y2 + 9x 2 − 13xy = 0

x
Divide by y 2 & Put =t
y
4
 9t 2 − 13t + 4 = 0  t = 1,
9
x 4
 = 1,
y 9

7. Sum of all values satisfying the equation log(− x) = log x 2 is (when base is 10)
(A) − 9 (B) − 10 (C) − 1 (D) − 11
Ans. (D)
Sol. log(− x) = log x 2 ( x  0)
log(− x) = log(− x)  (log(− x)) 2 = log(− x)
log(− x)(log(− x) − 1) = 0
log(− x) = 0,log(− x) = 1 → x = −1, −10

8. ( )
If log1 8 log1 4 ( log1 2 x ) = 1/3 then x is

(A) rational (B) irrational (C) integer (D) prime


Ans. (B)

Sol. ( )
log1 8 log1 4 ( log1 2 x ) = 1/3

1
3
1 3 1
 log 1 log 1 x =   =
4 2 2 2
1
 1 2 1
 log 1 x =   =
2 4 2

1
 x= irrational
2

163
Maths IIT-JEE ‘Best Approach’ (MCSIR) Logarithm

Integer Type
10 x+ y
9. If x, y > 0, logyx + logxy = and xy = 144, then = N where N is a natural number,
3 2
find the value of N.
Ans. (507)
Sol. Domain :-

x  0
y  0

x 1

y 1
10
log y x + log x y = & xy = 144
3
Put log y x = t

1 10
 t + =  3t 2 − 10t + 3 = 0
t 3
1
 t =3 or t=
3
1
 log y x = 3 or log y x =
3
1
 x = y3 or x = y3
xy = 144 xy = 144
4
 y4 = 144  y 3 = 144
3 1
 x = (12) 2  x = (12) 2

1 3
x + y (12) + (12) 2 2
 =
2 2
1
2
(12)
= (1 + 12)
2
13
= 12 = 507
2
 N = 507

164
Maths IIT-JEE ‘Best Approach’ (MCSIR) Logarithm

10. If logx log18 ( )


2 + 8 =–
1
2
. Then the value of 'x' is equal to

Ans. (4)
log x log18 ( 2 + 8 ) = −
1
Sol.
2
−1
 x 2 = log18 ( 2 + 8 )

= log18 ( 2 + 2 2 )

= log18 ( 3 2 )

= log 18 ( 18 )

1
=
2
1
x −1 = or x=4
4
11. The value of K, where log (log 4) + log (log 25) = log K + log (log 2) + log (log 5) is
Ans. (4)
Sol. log (log 4) + log (log 25) = log K + log (log 2) + log (log 5)
 log(log 4  log 25) = log K + log(log 2  log5)
 log(2log 2)(2log5) − log(log 2  log5) = log K
 log 4 = log K
m
by log m − log n = log
n
so K = 4

log a log b log c


12. If = = then value of aa . bb . cc is equal to
b−c c −a a −b
Ans. (1)
log a log b log c
Sol. = = =k
b−c c −a a −b
a = 10k (b−c ) → aa = 10k ( ab−ac ) (i)
a = 10k (c−a ) → bb = 10k (bc−ab ) (ii)
a = 10k ( a−b) → cc = 10k ( ac−bc ) (iii)
i  ii  iii → aabbcc = 10k0 = 1

165
Maths IIT-JEE ‘Best Approach’ (MCSIR) Logarithm

13. Number of real values of x satisfying the equation log1/3(x2 + 8) = – 2 is equal to

Ans. (2)
Sol. log1/3(x2 + 8) = – 2
−2
1
x + 8 =  
2
3
 x + 8 = 72
2

 x2 = 64
 x2 = 8, − 8

1
14. Let N = log215 · log1/62 · log3   . The greatest integer which is less than or equal to N is .
6
Ans. (2)
1
Sol. log215 · log1/62 · log3  
6
ln15 ln 2
= . .log 3 (1/ 6 )
ln 2 ln (1/ 6 )
ln15
= .log 3 (1 / 6 )
ln (1 / 6 )
A+ B +C
= =
D

. ( log 3 ( 6 ) )  2.46497....
ln15
=
ln 6
hence, [N] = 2
log 64 + log 9
15. Let A = log32 2 8 ; B = 5log5 2+log5 3 ; C = ; D = 2log 2 30 −1
2log 24
find the value of ( A + B + C)  D.
Ans. (1)

A = ( log 2 2 8) = 23 = 8
3
Sol.

B = 5log5 2+log5 3 = 5log5 6 = 6


log 64 + log 9
C=
2log 24
2 log 8 + 2 log 3
=
2 log 24
2log 24
= =1 and D = 2log2 30−1 = (30)−1 2
2log 24
30
= = 15
2
A + B + C = ( 8 + 6 + 1)  15 = 1
166
Maths IIT-JEE ‘Best Approach’ (MCSIR) Logarithm

PART TEST – 2
Single Correct Answer Type
1. Find the value of x satisfying the equation 9log3 (1−2x) = 5x 2 − 5 is
(A) – 2 – 10 (B) – 2 + 10 (C) 2 + 10 (D) 2 – 10
Ans. (A)
1
Sol. Domain : 1 – 2x > 0  x <
2

 9log3 (1−2 x ) = 5x2 − 5


 (1 – 2x)2 = 5x2 – 5
 x2 + 4x – 6 = 0
 (x = – 2 + 10 , – 2 – 10 )  Domain
     
 x = – 2 – 10

2. Given log102 = a and log103 = b. if 3x + 2 = 45. The value of x in terms of a and b is


a −1 1− a 1+ a b
(A) (B) (C) (D)
b b b 1− a
Ans. (B)
Sol. 3x+2 = 45 = 32  51
 3x = 5
x = log3 5

log 5 1 − log 2
= =
log 3 log 3
1− a
=
b

Number of real values of x satisfying the equation log 2 x + 2 log 2 x − 2 = 0 is equal to


2
3.
(A) 1 (B) 2 (C) 3 (D) 4
Ans. (B)
x  0
Sol. Domain :    x  (0, )
x  0
log222x + 2log2 x – 2 = 0
 (log2x)2 + log2x – 2 = 0 ; Put log2x = t
 t2 + t – 2 = 0
 (t + 2) (t – 1) = 0
 t = – 2, 1  log2x = – 2 , 1
 1 
  x = , 2   Domain
 4 
1
x= , 2
4

167
Maths IIT-JEE ‘Best Approach’ (MCSIR) Logarithm

4. Find the value of x satisfying the equation log3(4x – 3) + log3(4x – 1) = 1 is


(A) 0 (B) 1 (C) 2 (D) 3
Ans. (B)
Sol. log3(4x – 3) + log3(4x – 1) = 1 .....(1)
 log3[(4 – 3) (4 – 1)] = 1 ; Put 4 = t
x x x

 (t – 3) (t – 1) = 3  t2 – 4t = 0
 t = 0, 4
 4x = 0, 4  x = 1 ; Put in (1)
x = 1 satisfy eqn (1) ,  x = 1

 a + b  na + nb a b
5. If n  = then + is equal to :
 3  2 b a
(A) 5 (B) 6 (C) 7 (D) 8
Ans. (C)
 a + b  na + nb
Sol. n =
 3  2
a+b
 2n   = n(ab)
 3 
a+b
2

   = ab  a2 + b2 + 2ab = 9ab
 3 
 a2 + b2 = 7ab
a b
 + =7
b a

6. Find the value of x satisfying the equation ln(x – 3) + ln(x – 2) = ln(2x + 24) is
(A) 3 (B) 6 (C) 9 (D) 12
Ans. (C)
x −3  0 

Sol. Domian : x − 2  0    x > 3
2x + 24  0 

n(x – 3) + n(x – 2) = n (2x + 24)
 (x – 3) (x – 2) = 2x + 24
 x2 – 7x – 18 = 0
 (x = 9, – 2)  Domain
 x=9

168
Maths IIT-JEE ‘Best Approach’ (MCSIR) Logarithm

7. The real value of x for which the statement log69 – log927 + log8x = log64x – log64 holds true,
is
(A) 1/2 (B) 1/4 (C) 1/8 (D) 1/16
Ans. (C)
Sol. log69 – log927 + log8x = log64x – log64

log 64 x 2 − log 64 x = log9 27 − log 6 9 − log 6 4


1
log 64 x = log32 33 − log 6 36 → x =
8

8. If x = 2 ( ) (5 ) , then log
5 2
10x = ( )
A − B (log102) + B . The value of (A + B) equals
(A) 7 (B) 9 (C) 11 (D) 13
Ans. (A)
Sol. If x = 2 ( ) (5 ) then
5 2

log10x = ( )
A − B (log102) + B

 log10 ( 2 5 ) (5 2 ) = 5 log10 2 + 2 log10 5

 5 log10 2 + 2 log10 5 + 2 + 2 log10 2 − 2 log10 2

5 log10 2 − 2 log10 2 + 2 = ( 5 − 2 ) log10 2 + 2


A+ B = 5+ 2 = 7
9. If logab = 2; logbc = 2 and log3c = 3 + log3a then (a + b + c) equals
(A) 90 (B) 93 (D) 102 (D) 243
Ans. (B)
Sol. loga b = 2 , logb c = 2
 loga b  logb c = 4

 loga c = 4 ie c = a 4

Now log3 c = 3 + log3 a

 log3 a 4 = 3 + log3 a

 4log3 a = 3 + log3 a

 3log3 a = 3

 log3 a = 1 or a = 3

 c = a 4 = 34 = 81
also log b = 2
b = a2 = 9 so a + b + c = 93

169
Maths IIT-JEE ‘Best Approach’ (MCSIR) Logarithm

10. If log9x + log4y = 7 2 and log9x – log8y = – 3 2 , then x + y equals


(A) 35 (B) 41 (C) 67 (D) 73
Ans. (C)
7
Sol. log 9 x + log 4 y =
2
3
log 9 x − log8 y = −
2
7 3
 log 4 y + log8 y = +
2 2
1 1
 log 2 y + log 2 y = 5
2 3
1 1
 log 2 y  +  = 5
 2 3
5
 log 2 y   = 5
6
 log2y = 6
 y = 26 = 64
Then by (1)
7
log 9 x + log 4 64 =
2
1
 log 9 x = so x = 3
2
 x + y = 67
Integer Type
1 3

+3
.  ( ) 
log 9 log 6 3
81 2

− (125 )
5
log 25 7 log 25 6
11. find the value of 7 
409  
Ans. (1)
81log9 5 + 33.log3 6 
Sol.  ( 7) 2.log7 25 − (125)log25 6 
409
92.log9 5 + 33log3 6  log7 25 3log52 6 
=  7 −5
409  
25 + ( 6) 3
=  (25 − (6)3/2 )
409
(25)2 − 63 625 − 216
= = =1
409 409

170
Maths IIT-JEE ‘Best Approach’ (MCSIR) Logarithm

12. Let 'L' denotes the antilog of 0.4 to the base 1024.
and 'M' denotes the number of digits in 610 (Given log102 = 0.3010, log103 = 0.4771)
and 'N' denotes the number of positive integers which have the characteristic 2, when base of
the logarithm is 6.
Find the value of LMN.
Ans. (23040)
Sol. log10242 = 0.4  L = (1024)0.4 = (210)0.4 = 16
 L = 16
= Let y = 610 , Take log10 both sides
 log10y = 10 × log106 = 6 × (log102 + log103)
Log10y = 10 × (0.3010 + 0.4771) = 7.781
 Characteristic = No. of digits – 1
 7 = No. of digits – 1
 No. of digits = 8
 M=8
Characteristic of log6x = 2
 x  [36, 216)
 No. of +ve integers = 180
 N = 180
 LMN = 16 × 8 × 180
= 23040
1 4

+ +
log 5 3 log 9 36 log 7 9
13. Find the value of 81 27 3
Ans. (891)
1

Sol. (81) log5 3


= 34log3 5 = 54
= 625
27log9 36 = 33log3 6 = 63 = 216
4

3 log7 9
= 34log9 7 = 32log 7
= 72 = 49
Sum = 625 + 216 + 49 = 891

14. If log8a + log8b = (log8a)(log8b) and logab = 3, then the value of 'a' is
Ans. (16)
Sol. logab = 3 i.e. b = a3
Then from given
log8 a + 3log8 a = ( log8 a )( 3log 8 a )

log8 a  0 so 4 = 3log8 a
4
a = 8 3 = 24 = 16

171
Maths IIT-JEE ‘Best Approach’ (MCSIR) Logarithm

15. If a = log1218 & b = log2454 then find the value of ab + 5 (a − b).


Ans. (1)
Sol. a = log1218 & b = log2454
change base to 3
log 3 18 log 3 54
a= and b =
log 3 12 log 3 24

log3 ( 32  2 ) log3 ( 33  2 )
a = and b =
log3 ( 22  3) log3 ( 23  3)

2 + log 3 2 3 + log 3 2
a= and b =
2 log 3 2 + 1 3log 3 2 + 1

 2a log3 2 + a = 2 + log3 2 and 3blog3 2 + b = 3 + log3 2


2−a 3− b
 log 3 2 = and log 3 2 =
2a − 1 3b − 1
2−a 3− b
 log 3 2 = and log 3 2 =
2a − 1 3b − 1
eliminate log32
2−a 3− b
 =
2a − 1 3b − 1
 6b − 2 − 3ab + a = 6a − 3 − 2ab + b
 ab + 5(a − b) = 1

172
Maths IIT-JEE ‘Best Approach’ (MCSIR) Logarithm

BRAHMASTRA
FINAL REVISION MODULE BEFORE EXAMINATION

173
Maths IIT-JEE ‘Best Approach’ (MCSIR) Logarithm

LOGARITHM
1. LOGARITHM OF A NUMBER
The logarithm of the number N to the base ‘a’ is the exponent indicating the power to which
the base ‘a’ must be raised to obtain the number N. This number is designated as loga N.
logaN = x  ax = N , a > 0 , a  1 & N > 0
From the definition of the logarithm of the number N to the base 'a' , we have an identity :

a loga N = N , a > 0 , a  1 & N > 0


FUNDAMENTAL LOGARITHMIC IDENTITY

loga1 = 0 (a > 0 , a  1)
loga a = 1 (a > 0 , a  1) and
log1/a a = − 1 (a > 0 , a  1)

2. THE PRINCIPAL PROPERTIES OF LOGARITHMS

Let M & N are arbitrary positive numbers , a > 0 , a  1 , b > 0 , b  1 and  is any real
number then ;
(i) loga (M . N) = loga M + loga N (ii) loga (M/N) = loga M − loga N
log a M
(iii) loga M =  . loga M (iv) logb M =
log a b
NOTE :  logba . logab = 1  logba = 1/logab.  logba . logcb . logac = 1
x
 logy x . logz y . loga z = logax.  eln a = ax

3. PROPERTIES OF MONOTONICITY OF LOGARITHM

(i) For a > 1 the inequality 0 < x < y & loga x < loga y are equivalent.
(ii) For 0 < a < 1 the inequality 0 < x < y & loga x > loga y are equivalent.
(iii) If a > 1 then loga x < p  0 < x < ap
(iv) If a > 1 then logax > p  x > ap
(v) If 0 < a < 1 then loga x < p  x > ap
(vi) If 0 < a < 1 then logax > p  0 < x < ap

174
Maths IIT-JEE ‘Best Approach’ (MCSIR) Logarithm

SOLVED EXAMPLES
log 2 (9 − 2x )
1. Find the value of x satisfying the equation = 1 is
3− x
(A)1 (B)2 (C)3 (D)0

Ans. (D)
log 2 (9 − 2x )
Sol. = 1 ....(1)
3− x
 log2(9 – 2x) = 3 – x
 9 – 2x = 23–x = ; Put 2x = t
 t2 – 9t + 8 = 0  t = 1, 8
 2x = 1, 8  x = 0 , 3 ; Put in equation (1)
 x = 3 does not satisfy equation (1)
 x=0

2. If a = log1218 & b = log2454 then find the value of ab + 5 (a − b).

Ans. (1)
Sol. a = log1218 and b = log2454
Change base to 3
log 3 18 log3 54
 a= and b=
log 3 12 log 3 24
log 3 (32  2) log 3 (33  2)
 a= and b=
log 3 (22  3) log 3 (23  3)
2 + log 3 2 3 + log 3 2
a= and b=
2 log 3 2 + 1 3log 3 2 + 1
 2alog32 + a = 2 + log32 and 3blog32 + b = 3 + log32
2−a 3−b
 log3 2 = and log 3 2 =
2a − 1 3b − 1
Eliminate log3 2
2− a 3−b
 =
2a − 1 3b − 1
 (2 − a)(3b −1) = (3 − b)(2a −1)
 6b − 2 − 3ab + a = 6a − 3 − 2ab + b
 ab + 5a − 5b = 3 − 2
 ab + 5(a − b) = 1

175
Maths IIT-JEE ‘Best Approach’ (MCSIR) Logarithm

3. Find the value of x satisfying the equation log1/3 x − 3 log1/3 x + 2 = 0 is

1 1 
Ans.  , 
 3 81 
x0   x  0
Sol. Domain    x  (0, 1]
log1/3 x  0   x  1 
log1/3x – 3 log1/3 x + 2 = 0
Put log1/3x = t2
 t2 – 3 |t| + 2 = 0
 |t|2 – 3|t| + 2 = 0
 |t| = 1, 2
 log1/3x = t2 = 1, 4
 1 1
  x = ,   Domain
 3 81 
1 1
 x= ,
3 81
4. Solve for 'x' in the equation: ln(x – 3) + ln(x – 2) = ln(2x + 24) :

Ans. (9)
x −3  0


Sol. Domian : x − 2  0    x > 3
2x + 24  0 
n(x – 3) + n(x – 2) = n (2x + 24)
 (x – 3) (x – 2) = 2x + 24
 x2 – 7x – 18 = 0
 (x = 9, – 2)  Domain
 x=9
1 3

+3
.  ( ) 
log 9 log 6 3
81 2

− (125 )
5
log 25 7 log 25 6
5. Simplify: 7 
409  

Ans. (1)
81log9 5 + 33.log3 6 
Sol.  ( 7) 2.log7 25 − (125)log25 6 
409
92.log9 5 + 33log3 6  log7 25 3log52 6 
=  7 −5
409  
25 + ( 6) 3
(25) − 63 625 − 216
2
=  (25 − (6)3/2 ) = = =1
409 409 409

176
Maths IIT-JEE ‘Best Approach’ (MCSIR) Logarithm

6. Number of solutions of log4(x – 1) = log2(x – 3) is [JEE 2001]


(A) 3 (B) 1 (C) 2 (D) 0

Ans. (B)
Sol. Domain: -
x −1  0
x −3  0 
 x 3
 log 22 (x − 1) = log 2 (x − 3)
1
 log 2 (x − 1) = log 2 (x − 3)
2
log 2 ( x − 1) = log 2 ( x − 3)
2

x − 1 = ( x − 3)
2

x 2 − 7x + 10 = 0
( x = 2,5)  Domain

x = 5
7. Let (x0, y0) be solution of the following equations
(2x)ln2 = (3y)ln3 [JEE 2011]
3lnx = 2lny
Then x0 is :
1 1 1
(A) (B) (C) (D) 6
6 3 2
Ans. (C)
Sol. (2x)ln2 = (3y)ln3 & 3lnx = 2lny
Take “ln” both sides in both equation.
 ln2 ( ln(2x) ) = ln 3 ( ln(3y) )
 ln 2[ln 2 + ln x] = ln 3[ln3 + ln y] ….(1)
&
ln x(ln 3) = ln y(ln 2) ….(2)
From equation (1) & (2) eliminate lny
ln x
 (ln 2) 2 + ln 2  ln x = (ln 3) 2 + ln 3   ln 3
ln 2
 (ln 3) 2  ln x
 (ln 2)2 − (ln 3) 2 =  − (ln 2)  
 ln 2  ln 2
ln x
( ln 2 ) − ( ln 3) = − ( ln 2 ) − ( ln 3)  
2 2 2 2

ln 2
 ln x = − ln 2
1
 x=
2

177
Maths IIT-JEE ‘Best Approach’ (MCSIR) Logarithm

 
 1 1 1 1
8. The value of 6 + log 3 4− 4− 4− ...  is [JEE 2012]
 3 2 3 2 3 2 3 2 
2
 
Ans. (4)
 
 1 1 1 1
Sol. To find = 6 + log 3 4− 4− 4− .... 

2 3 2 3 2 3 2 3 2 
 
 t 
= 6 + log 3   ….(1)
2 3 2 

1 1 1
 t = 4− 4− 4− .... ; t  0
3 2 3 2 3 2
t
 t = 4− , Square both sides
3 2
 3 2t 2 + t − 12 2 = 0
3 4 2
 t=− (reject) or t =
2 3
4 2
 Put t = in equation (1)
3
 1 4 2
 To find = 6 + log 3   
 3 
2 3 2

4
= 6 + log 3   = 6 − 2 = 4
2 9

1 1

9. The value of is ( (log 2 )


9)2 log2 (log2 9)  ( 7) log 4 7
[JEE Advance 2018]

Ans. (8)
1 1

Sol. ( (log 2 9) )
2 log (log 9)
2 2  ( 7) log 4 7

Put log 2 9 = t
2
= (t )
log7 2
2 log t 2
7 2

= t 2logt 2  2 = t 2logt 2  2
= 4 2 = 8

178
Maths IIT-JEE ‘Best Approach’ (MCSIR) Logarithm

 ab + (ab)2 − 4(a + b)   ab − (ab)2 − 4(a + b) 


10. Let A denotes the value of log10   + log10  
 2   2 
   
when a = 43 and b = 57 and B denotes the value of the expression ( 2 log 18
6
)·( 3 ) .
6 log 3

Find the value of (A · B).

Ans. (12)
 ab + (ab)2 – 4(a + b)   ab – (ab)2 – 4(a + b) 
Sol. A = log10   + log10  
 2   2 
   
 (ab) − ((ab) − 4(a + b)) 
2 2
 A = log10  
 4 
= log10(a + b) = log10100 = 2 { a = 43 ; b = 57}

B = 2log6 18 ·3log6 3 = 2log6 (63) ·3log6 3


= 21+log6 3 ·3log6 3
= 2 · 2log6 3 ·3log6 3 = 2  (2·3)log6 3
= 2 × (6)log6 3 = 2  3 = 6
 A × B = 2 × 6 = 12

179

You might also like